Saverese PRACTICE EXAM QUESTIONS- Mine

¡Supera tus tareas y exámenes ahora con Quizwiz!

During your evaluation of a patient complaining of neck pain you find a decrease in flexion and extension. What structure typically has the greatest movement in flexion and extension? A) Occipital condyles B) Atlanto-axial joint C) C3 D) C5 E) C7

A

Manipulation of which spinal segments may decrease blood pressure by decreasing fluid retention? A) C3-C4 B) C5-C7 C) T5-T9 D) T10-T11 E) T12 - L1

D

Abnormal visceral afferent input from cardiac muscle is most likely to manifest somatic changes in paraspinal musculature adjacent to which vertebral level. A) C5 - C7 B) T3 - T4 C) T7 - T9 D) T9-T12 E) L1 - L3

B

A patient with a history of muscular dystrophy has weakness of the pelvic-girdle muscles, increased lordosis, pot-bellied posture and appears to roll from side to side when he walks.

A waddling gait can be described as rolling from side to side. The pelvic rotation and pelvic tilt are increased. This has been described as a penguin walk. Muscular dystrophy with weakness of hips, exaggerated lordosis, and pot-bellied posture can produce this gait.

Which of the following manipulative techniques would be contraindicated in this patient? A) Pedal pump B) Thoracic inlet release C) Rib raising D) CV4 bulb decompression E) Indirect myofascial release

A The pedal pump utilizes the intermittent movement of the abdominal contents up against the diaphragm. This movement may disrupt recent abdominal surgery and therefore it is contraindicated. Thoracic inlet release would be indicated in this patient to improve lymphatic return from the lower extremities and the lungs. Answer B Rib raising is also indicated in this patient as described above. Rib raising is contraindicated in recent spinal surgery and rib or spinal fractures. Answer C Craniosacral techniques as well as indirect myofascial release techniques are typically well tolerated in the post-operative patient, and therefore not contraindicated in this scenario. Answers D and E

A patient is found to have a posterior transverse process on the right at L2. Flexion of the lumbar spine increases the asymmetry at this segment, whereas extension restores symmetry. Which of the following is the correct muscle energy (direct) treatment position. A) Flexed rotated left sidebent left B) Flexed rotated right sidebent right C) Extended rotated right sidebent right D) Extended rotated left sidebent left E) Neutral rotated right sidebent left

A

Which of the following soft tissue dysfunctions has been specifically associated with ureterolithiasis? A) Psoas spasm B) Abdominal rectus spasm C) Piriformis tenderpoint D) Paraspinal spasm E) Quadratus lumborum spasm

A Due to the close proximity of the ureter to the psoas muscle, ureterospasm has been associated with iliopsoas dysfunction and psoas syndrome. Abdominal rectus spasm and a piriformis tenderpoint are not likely due to the fact that these muscles are not local to the site of irritation. Answers B and D Paraspinal muscle spasm may be seen in patients with ureteral stones, especially patient's with costovertebral angle tenderness, however paraspinal muscle spasm can occur in many different dysfunctions and has not been specifically associated with ureterospasm. Answer C The quadratus lumborum lies posterior to the iliopsoas and would less likely be affected with a ureteral stone. Answer E

A patient has a sacral torsion that resulted in a right anterior sacral base and a negative lumbosacral spring test. Based on the information given, what is the expected somatic dysfunction at L5? A) L5 NSlRr B) L5 NSrRl C) L5 ERrSl D) L5 FRlSl E) L5 FRrSl

A L5 NSlRr If a sacral torsion is present, a right sacral base anterior (a deep right sulcus) and a negative lumbosacral spring test indicate a left on left torsion. Using the rules of L5 on the sacrum and Fryette's principles one can figure out the dysfunction of L5. Rules of L5: 1) When L5 is rotated, the sacrum rotates in the opposite direction. - Sacrum rotated left, L5 must be rotated right. 2) When L5 is sidebent, a sacral oblique axis is engaged on the same side as the sidebending. - Sacrum has a left oblique axis, L5 must be sidebent left. Fryette's principle I: If L5 is rotated right and sidebent left, L5 must be in the neutral plane (L5 NSlRr).

A 63-year-old-male with a right peroneal compression neuropathy has a complete foot drop. When he ambulates he flexes his right hip more than his left in order to clear his right toes

A high steppage gait will typically result from a foot drop. In this case the effected side is raised higher (hip is flexed more) in order to clear the foot.

During the evaluation of a patient's upper back pain, you notice that your right thumb is more posterior upon palpation of the transverse processes of T4. T4 returns to neutral position with flexion of the thoracic spine, whereas extension of the thoracic spine increases the asymmetry. Which of the following is the most correct assessment of T4? A) Sidebent left, rotated left B) Sidebent right, rotated right C) Sidebent right, rotated left D) Sidebent left, rotated right E) Rotated left, no sidebending component

B

The most common injured ligament in the ankle is: A) Spring ligament B) Anterior talofibular ligament C) Posterior talofibular ligament D) Deltoid ligament E) Calcaneofibular ligament

B

At which spinal level would you expect to find palpatory changes related to a viscerosomatic reflex? A) T4 B) T6 C) T10 D) T12 E) L2

B This patient has gastric cancer. This would cause viscerosomatic changes to occur at the T5 - T9 spinal levels. T4 typically does not receive sympathetic innervation from structures below the diaphragm. Answer A T10, T12 and L2 receive sympathetic innervation from the middle and lower GI tract. Answers C, D and E

Increases fluid flow from middle ear structures via the eustachian tube

B. GALBREATH TECHNIQUE The Galbreath Technique is a mandibular drainage technique that relaxes the medial pterygoid muscle, which enables the tensor veli palatini muscle to functionally open the eustachian tube. This technique is useful in acute otitis media.

Helps decongest the medial pterygoid plexus

B. GALBREATH TECHNIQUE. Relaxing the medial pterygoid muscle helps decongest the rich medial pterygoid plexus.

A patient with back pain received osteopathic manipulative treatment. The following was done: HVLA to C3, C5, T4, L1, L5 and right innominate Muscle energy to the sacrum, Counterstrain to the piriformis muscle Based on the above information what is the maximum number of treated areas that should be filled out on the Osteopathic Soap Note Form? A) 1-2 areas B) 3-4areas C) 5-6 areas D) 7-8 areas E) 9-1 0 areas

C

Chronic pyrosis and regurgitation would most likely cause paraspinal tissue texture changes from a viscerosomatic reflex at which spinal level? A) C3-C6 B) T1-T4 C) T3-T8 D) T10-T11 E) L1-L3

C Hypersympathetic activity from GERD will result in tissue texture changes at T2-T8 because these segments receive sympathetic innervation from the esophagus, so T3-T8 is the best choice. C3-C6 have no sympathetic fibers associated with the esophagus. Answer A Tl-T4 receive sympathetic innervation from the head and neck. Answer B T10-Tll receive sympathetic innervation from the middle GI tract. Answer D Ll-L2 have no sympathetic fibers associated with the esophagus. Answer E

A patient complains of mid-thoracic back pain for three days. While evaluating T5 you find that your right thumb is more anterior than your left, and symmetry is not restored after flexion or extension. What is the diagnosis for this somatic dysfunction?

C T5 NRlSr

Which of the following somatic dysfunctions has been linked with suckling difficulties in the newborn? A) Upper thoracic dysfunction B) Hyperparasympathetic tone C) Maxillae dysfunction D) Cranial nerve dysfunction E) TMJ dysfunction

D Suckling difficulties have been associated with condylar compression and/or restrictions at the jugular foramen. Condylar compression causes cranial nerve XII dysfunction. Other possible causes of suckling dysfunctions have been associated with restrictions at the jugular foramen causing cranial nerve IX and X dysfunction

52-year-old man comes to your office with low back pain of acute onset. Examination reveals a positive standing flexion test on the left. The right anterior superior iliac spine (ASIS) is more anterior and inferior than the left. The most likely diagnosis is: A) Right superior pubic shear B) Right posterior innominate C) Right anterior innominate D) Left posterior innominate E) Left anterior innominate

D

A 30-year-old male runner presents with left-sided low back pain and left hip pain. The pain started yesterday after an 8 mile run. It is sharp but does not radiate into the lower extremities. On examination, you notice tenderness over the left SI joint, a positive seated flexion test on the left, the sacral sulcus on the left is anterior, while the right ILA is posterior and inferior. Based on the information given what is the most likely diagnosis? A} Left sacral rotation on a left oblique axis (L on L) B} Left sacral rotation on a right oblique axis (L on R) C) Right sacral rotation on a left oblique axis (R on L) D} Right sacral rotation on a right oblique axis (R on R) E} Unilateral sacral flexion on the right (USFR).

D

A tender point is located 2cm lateral to the medial end of the left clavicle at the attachment of the inserting neck muscle. What is the counterstrain treatment position for this anterior cervical tender point? A) Head and neck rotated left, sidebent right B) Head and neck rotated right, sidebent right C) Head and neck rotated left, sidebent left D) Head and neck rotated right, sidebent left E) Head and neck rotated right, and flexed

D The position describes the left seventh anterior cervical tender point. This corresponds to the left sternocleidomastoid muscle (SCM) and it is considered a maverick point. The appropriate treatment position is to rotate away (right), and side bend toward (left).

34-year-old female complains of dysuria and odd smelling urine. Physical examination reveals suprapubic tenderness. At what spinal levels would you expect to find the tissue texture changes related to a viscerosomatic reflex that would confirm the diagnosis? A) T2-T4 B) T5-T7 C) T8-T10 D) T11-L1 E) L2-L5

D This patient most likely has a urinary tract infection, and would have tissue texture changes from a viscerosomatic reflex at Tll-Ll. T2-T4 receives sympathetic innervation from the head and neck. Answer A T5-T7 receives sympathetic innervation from the upper GI tract. Answer B T8-Tl0 receives sympathetic innervation from the upper and middle GI tract. Answer C The spinal cord stops at L2; therefore the lowest possible segment to receive sympathetic innervation is L2. Answer E

Contraindicated in infectious mononucleosis

D. SPLENIC PUMP The splenic pump improves immune function and is indicated in infection and fever, however, it is contraindicated in splenomegaly. This is a common finding in infectious mononucleosis.

The most appropriate management of the patient would be: A) GI consultation B) Psychiatric consultation C) Immediate surgical consultation D) SI joint injection E) Sacrococcygeal joint mobilization

E

Which structures/locations are least likely to be influenced in a patient with cardiac dysfunction? A) The intercostal space between rib 2 and 3 at the sternal junction B) The cranial base C) The second cervical vertebrae D) The second thoracic vertebrae E) The seventh thoracic vertebrae

E

It is recommended that the above tenderpoint is treated for: A) 15 seconds B) 30 seconds C) 60 seconds D) 90 seconds E) 2 minutes

E In order to allow extra time for the patient to relax, it is recommended that rib tender points be treated for 2 minutes. All other body counterstrain points are typically treated for 90 seconds. Answer D There are no other intervals of time used for counterstrain. Answers A, B, and C

A 16-year-old-female with spastic diplegic cerebral palsy walks with her legs adducted, crossing alternately in front of one another.

In scissor gait the legs are adducted, crossing alternately in front of one another. Both adductors typically have spasticity. This is a common type of gait in children with cerebral palsy.

In a patient with low back gluteal pain physical examination shows the following: Positive seated flexion test on the right, left ILA posterior/inferior, and right sacral base is anterior. Lumbosacral spring test is negative and lumbar curve is convex to the right. Assuming the patient has a sacral torsion what would be the expected L5 dysfunction?

L5 NSlRr

Which of the following correctly describes the treatment position when treating the above somatic dysfunction with facilitated positional release? A) Marked extension of the thoracic spine, right rotation, right sidebending B) Marked flexion of the thoracic spine, right rotation, right sidebending C) Flattened thoracic kyphosis, right rotation, right sidebending D) Marked flexion of the thoracic spine, left rotation left sidebending E) Marked extension of the thoracic spine, left rotation, left sidebending.

C Although facilitated positional release is an indirect technique, somatic dysfunctions of the spine are treated with the spine in the neutral position. This entails flattening the thoracic kyphosis, and flattening the lordosis of the lumbar and cervical spine. The segment is then positioned away from the restrictive barrier. In this case sidebent and rotated right

Which one of the following statements regarding the transverse processes of the cervical spine is true? A) They are located posterior to the articular pillars. B) They are typically used as palpatory landmarks for osteopathic physicians to induce translation at the cervical spine. C) The posterior tubercle of the transverse process serves as the origin of the anterior scalene. D) The vertebral artery courses through the transverse processes C1 - C7. E) They are located immediately posterior to the zygapophyseal joints.

C Transverse processes have an anterior and posterior tubercle. The posterior tubercles serve as the origin of the scalenes. They are located anterior to the articular pillars. Answer A. The articular pillars are typically used as palpatory landmarks for osteopathic physicians to induce translation at the cervical spine. Answer B The vertebral artery courses through the transverse processes C1 - C6. Small accessory vertebral veins go through the foramen transversarium of C7. Answer D They are located immediately anterior to the zygapophyseal joints. Answer E

Contraindicated in deep venous thrombosis

A. DALRYMPLE PUMP The pedal (a.k.a. Dalrymple) pump is a venous and lymphatic drainage technique applied through the lower extremities. It is contraindicated in patients with deep venous thrombosis.

A patient has a severely painful bunion in his right foot that results in a short stance phase on the right and a rapidly executed swing phase of the left leg.

Antalgic gait is characterized by a short stance phase while the patient is standing on the painful extremity. During this short stance phase there is a rapidly executed swing phase of the opposite leg. The patient tries to avoid standing on a painful extremity.

A patient with diverticulosis is likely to have chronic changes related to a Chapman's point in which of the following areas? A) Periumbilically B) Iliotibial band on the left C) Paraspinal muscles at T9 D) Spinous process of T11 E) Paraspinal muscles at L4

B

Which of the following anatomic sacral somatic dysfunctions is often seen in the postpartum patient? A) Bilateral sacral extension B) Bilateral sacral flexion C) Forward sacral torsion D) Backward sacral torsion E) Unilateral sacral flexion

B

At what spinal level would you expect to find tissue texture changes related to this patient's COPD? A) CO-C2 B) T2-T6 C) T7-T10 D) T11-L2 E) L3-L5

B T2-T7 is the spinal level for the sympathetic reflex of the lungs. Answers A and E have no sympathetic innervation. CO is another term for the occiput. T7-T10 is the sympathetic innervation for the upper to middle GI tract. Answer C T11-L2 is the sympathetic innervation for the middle to lower GI tract. Answer D

A patient comes to your office after sustaining a fall. Structural exam reveals a negative seated flexion test and a positive standing flexion test. The left ASIS is 5 inches from midline and the right is 4 inches from midline. AP compression of the right ASIS demonstrates adequate resiliency, whereas the left side is restricted. What is the most likely diagnosis A) A left anterior innominate rotation B) A right innominate inflare C) A left innominate outflare D) A right superior innominate shear E) A left superior innominate shear

C

Given the above findings, what is the sacral diagnosis? A) Left sacral rotation on a right oblique axis B) Left sacral rotation of a left oblique axis C) Right sacral torsion on a left oblique axis D) Right sacral torsion on a right oblique axis E) Bilateral sacral extension

C

Indicated for pneumonia

C. THORACIC PUMP The thoracic pump is a technique that consists of intermittent compression of the thoracic cage, which is indicated for pneumonia and other pulmonary conditions

A patient with renal failure for 8 months would be expected to have: A) Acute tissue texture changes at T7 B) Chronic tissue texture changes at T9 C) Acute tissue texture changes at T11 D) Chronic tissue texture changes at T11 E) Acute tissue texture changes at T9

D

36-year-old female complains of abdominal pain that is associated with anorexia, fatigue, and weakness. Physical exam reveals an area of hyperpigmentation in the palmar creases, and sparse axillary hair. Lab results reveal low sodium, and elevated potassium, and calcium levels. At what spinal level would you expect to find tissue texture changes due to a viscerosomatic response from the above condition? A) T2 B) T4 C) T8 D) T10 E) L2

D This patient has Addison's disease, which is a disease of the adrenal gland and would cause a viscerosomatic response at T10. T2 and T4 receive sympathetic innervation from the head and neck, heart esophagus and lungs, not the adrenal medulla. Answers A and B T8 would receive sympathetic innervation for the upper GI tract. Answer C L2 would receive innervation from the lower GI tract, uterus and cervix, penis, clitoris and legs. Answer E

You approach your osteopathic professor one day with a very serious question. "A friend of yours" is having difficulty with premature ejaculation. As a concerned friend you ask the professor if there are any specific GMT techniques that may help this type of ejaculatory dysfunction. The professor explains that GMT may help alter visceral input from facilitated segments. Based on your knowledge of the autonomic nervous system and visceral innervation, you come to the conclusion that treatment of which structure(s) may help "your friends" premature ejaculation? A) Occipito-atlantal joint B) Sacrum C) T4 - T6 D) T8 - T10 E) T12 - L2

E

Spasm of the muscle in question 79 could result in which one of the following conditions? A) 1st rib dysfunction B) Left cervical rotation C) Second rib elevation D) Thoracic outlet syndrome E) Torticollis

E Torticollis results from a shortening of the sternocleidomastoid (SCM) muscle. A first rib dysfunction or second rib elevation would not likely occur with spasm of the SCM since it does not attach to these structures. Answers A and C The SCM rotates the head in the opposite direction, therefore it would cause right cervical rotation. Answer B Spasm of the SCM is not associated with thoracic outlet syndrome. Answer D

In a patient with low back pain, the dysfunctional T12 segment is found to have restriction in a transverse plane and around a transverse axis. Which of the following dysfunctions best describes the position of T12? A) T12 is flexed B) T12 is sidebent right C) T12 is rotated left D) T12 is neutral sidebent right, rotated left E) T12 is extended rotated right, sidebent right

E Vertebral motion around a transverse axis is flexion or extension. So therefore T12 must be either flexed or extended. Vertebral motion in a transverse plane is rotation. So therefore T12 must also be rotated. Understanding Fryette's principles one must conclude if a vertebrae is flexed or extended and rotated it must be sidebent toward the same side. T12 has restricted motion around a transverse axis, therefore it is flexed or extended, not neutral. Answer D Answers A, B and C only comment on one plane of motion.

The mother of a 3 year-old female complains that her child limps when she walks. Originally, she thought that her daughter had foot pain because she had worn new shoes. After a thorough inspection of the feet, the mother found that the pain was located in her left hip. Her daughter has also had a runny nose for seven days without a fever. Physical examination reveals a playful child with a limp that favors the right side. Hips have full range of motion bilaterally but the child guards all motion of the left hip. There are no neurologic deficits noted. Laboratory tests and X- rays of the hip are normal What is the most likely diagnosis? A) Septic arthritis B) Legg-Calve Perthes disease C) Slipped capital femoral epiphysis D) Developmental dislocation of the hip E) Transient synovitis of the hip

E Transient synovitis of the hip (also known as toxic synovitis) is a nonspecific, common, unilateral (5% bilateral) inflammatory arthritis involving the hip joint, which occurs in children under 10 years of age (typically 3-6 years of age). It is the most common cause of limp with hip pain in children. The male to female ratio is 3-5: 1. There may be a history of a preceding upper respiratory tract infection. Pain may be present in the hip, antero-medial aspect of the thigh and the knee. Occasionally, a low-grade fever of 100-101F may be present. X-ray of the hip is normal. Septic arthritis is a serious pyogenic infection of the joint space. It occurs most often in children less than 3 years old. The joint is swollen, and effusion, erythema, tenderness, pain and warmth are evident. The child may have a history of a recent bacterial infection. Septic arthritis is usually accompanied by a fever. The knee is the most commonly affected joint followed by the hip, elbow, and ankle. Answer A Legg-calve-Perthes disease is a juvenile idiopathic avascular necrosis of the femoral head. The onset is insidious taking weeks to months, which does not fit the case history. Answer B Slipped capital femoral epiphysis represents a displacement of the femoral head from the femoral neck due to a stress fracture through the femoral capital epiphyseal growth plate. It is classically seen in obese adolescent males. This would have been diagnosed by the X-ray of the hip. Answer C Developmental dislocation of the hip encompasses the severity spectrum from mild acetabular dysplasia to frank dislocation. Hip X-ray would have demonstrated a shallow acetabulum with a completely or partially subluxed femoral head. Answer D

A patient has a tender point at the right articular process of C5. What is the typical treatment position for the head and neck in order to treat this tender point using counterstrain? A) Flexed, sidebent to the left, and rotated to the left. B) Extended, sidebent to the left, and rotated to the left. C) Flexed, sidebent to the right, and rotated to the right. D) Extended, sidebent to the right, and rotated to the right. E) Flexed, sidebent to the left, and rotated to the right.

A

A 68-year-old male complains of a headache and blurred vision for several months. Physical examination reveals an abnormal ophthalmic tonometer. At what spinal levels would you expect to find the tissue texture changes related to a viscerosomatic reflex? A) T1-T3 B) T5-T6 C) T7-T9 D) T10-T12 E) T12-L2

A

Which treatment will help decrease the symptoms associated with the above condition? A) Antibiotics B) Corticosteroids C) Non-steroidal anti-inflammatory drugs D) Surgical fixation E) Hip manipulation

C Most children can be treated symptomatically with bed rest at home and non-steroidal anti-inflammatory medication to decrease pain and inflammation. Antibiotics would be indicated for septic arthritis. Answer A Corticosteroids are not indicated for any of these conditions. Answer B Surgical fixation is indicated for slipped capital femoral epiphysis. Answer D Hip manipulation has not been shown to improve the symptoms or shorten the duration of transient synovitis of the hip. Answer E

Which of the following is true regarding this patient's hip musculature? A) Right adductors are restricted B) Left adductors are restricted C) Right gluteus medius and tensor fascia lata are restricted D) Left gluteus medius and tensor fascia lata are restricted E) Right gluteus medius and tensor fascia lata have increased flexibility

C The patient's right leg is limited in adduction indicating that the right abductors (gluteus medius and tensor fascia lata) are restricted. If the right adductors were restricted, right leg abduction would be decreased. Answer A If the left adductors were restricted, left leg abduction would be decreased. Answer B If the left abductors were restricted, left leg adduction would be decreased. Answer D If the right abductors had increased flexibility, the right leg would have increased adduction. Answer E

What is the primary reason for treating the thoracoLumbar junction in the above patient? A) To decrease this patients pain B) To increase function of the contralateral kidney C) To decrease ureterospasm and increase glomerular filtration rate D) To decrease bladder spasm E) To decrease this patient's blood pressure

C Ureterolithiasis is associated with ureterospasm. In addition, patients with unilateral renal disease associated with ureteral stones have a decreased glomerular filtration rate. This is probably because hypersympathetic tone to the kidneys will decrease GFR. Treating the thoracic spine will decrease sympathetic tone to the kidneys (Tl0 - Tll) and treating the lumbar region will decrease sympathetic tone to the ureters associated with ureterospasm. Although controlling the patient's pain is important, pain management is usually controlled with medication; osteopathic treatment should be limited to the reduction of sympathetic tone. Paraspinal inhibition and rib raising have been suggested techniques. Answer A Treatment to the thoraco-Iumbar junction cannot increase function of the normally functioning contralateral kidney. Answer B Since the obstruction is proximal to the bladder, spasm is not the primary concern in the above patient; therefore it is not the primary reason for treating the thoraco-Iumbar junction. Answer D Although the patient does have some high blood pressure, it is a result of the patient's diagnosis. Treating the thoraco-Iumbar junction to decrease the blood pressure is treating a symptom, not the cause. Answer E

A 56-year-old male is complaining of low back pain. On examination you notice transverse process of L5 is posterior on the right. Extending L5 worsens the asymmetry while flexion restores rotational symmetry. Which one of the following is the best statement regarding this patient's somatic dysfunction? A) Due to the non-neutral L5 somatic dysfunction, a group dysfunction of the high lumbar region will be present. B) This patient has a right-sided psoas syndrome. C) L5 will resist left rotation in the extended position. D) When employing a muscle energy (post-isometric relaxation) technique the patient would be asked to rotate his torso to the left against isometric contraction. E) Extending L5 will greatly limit its sidebending to the right

C. L5 will resist left rotation in the extended position The patient in the above question has L5 flexed, rotated right, and sidebent right. If the spine is extended symmetry is worsened, and L5 will resist left rotation. A non-neutral L5 dysfunction does not always cause a high lumbar group dysfunction or a right-sided psoas syndrome. Answers A and B Post-isometric relaxation muscle energy technique is an example of a direct technique in which the patient is positioned toward the barrier and is asked to turn away from the barrier. In this case the patient would be positioned with his torso rotated to the left and he would be asked to rotate his torso to the right. Answer D Extending L5 will limit its sidebending to the left, not right. Answer E

A patient presents with a neurological deficit resulting in an absent triceps tendon reflex and weakness in the extensors of the hand, wrist and elbow. Which nerve is most likely involved? A) Musculocutaneous B) Median C) Radial D) Ulnar E) Axillary

C. Radial N The radial nerve innervates the extensors of the upper extremity [elbow (triceps), wrist and hand]. An absent triceps (C7) reflex and wrist drop will indicate radial nerve pathology. The musculocutaneous nerve innervates the biceps, brachialis, and coracobrachialis. A patient with a musculocutaneous nerve injury presents with a loss of the biceps reflex (C5, C6). Answer A The median nerve innervates forearm pronators, wrist and finger flexors and thenar muscles. Answer B The ulnar nerve innervates flexor carpi ulnaris, hypothenar muscles and the adductor pollicis. Answer D The axillary nerve innervates the teres minor and deltoid muscles. Injury to the axillary nerve results in atrophy of the deltoid and weakness with shoulder abduction and flexion. Answer E

What is the rationale for using rib raising on this patient? A) It will increase sympathetic tone thus promoting cranial vasoconstriction B) It will decrease sympathetic tone thus promoting cranial vasoconstriction C) It will increase sympathetic tone thus limiting cranial vasoconstriction D) It will decrease sympathetic tone thus limiting cranial vasoconstriction E) It will be of no benefit to this patient

D Increased sympathetic tone to innervated blood vessels of the cranium will cause vasoconstriction. The purpose of rib raising is to reduce hypersympathetic tone thus limiting vasoconstriction. Other somatic dysfunctions, such as cranial dysfunctions and axial dysfunctions may produce fascial strains that are transmitted to the head and may playa role in the origin of a migraine. Since classic migraines are thought to be caused by a vasoconstriction of innervated blood vessels and a vasodilatation of non-innervated blood vessels, techniques aimed at reducing sympathetic tone will decrease initial vasoconstriction. Answer E

In this patient. treatment of which area would have the greatest effect on renal function? A) Occipito-atlantal B) T5 C) T8 D) T11 E) The sacrum

D The kidneys receive sympathetic innervation from segments Ti0 - Til. Therefore, treatment of these segments would be most likely to decrease excessive sympathetic tone to the kidneys. This will improve the glomerular filtration rate and increase urinary output. Although the occipito-atlantal joint will influence the vagus nerve, it is not known how the parasympathetic nervous system affects the kidney. Answer A The kidney does not receive sympathetic innervation from T5 or T8. Answers B and C Although treatment of the sacrum will influence the pelvic splanchnic nerve, the kidney does not receive innervation from the pelvic splanchnic nerve. Answer E

A 72-year-old male comes to your office with chronic low back pain. He has a 20+year history of diabetes and was recently diagnosed with prostate cancer. His back pain is a dull ache that radiates into the right buttock and thigh. The pain is made worse with standing or walking. Pain is relieved with sitting. On examination, pulses are strong in the lower extremities and capillary refill is adequate. Range of motion of his lumbar spine is decreased. Sensation is decreased in both feet in a stocking like distribution. Muscle testing is 5/5 in both lower extremities. Reflex testing is 2+ at the patella bilaterally, and absent at the Achilles bilaterally. Considering this patient's symptoms, what is the most likely finding this patient will have on radiological studies? A) A herniated nucleus pulposus on magnetic resonance imaging B) Lytic lesions involving the lumbar vertebrae C) Spondylolisthesis of L5 on S1. D) Spondylosis of the lumbar spine E) Compression fracture of the lumbar vertebrae.

D The patient's aggravating factors and alleviating factors along with the physical findings should indicate to the test taker that this patient most likely has lumbar spinal stenosis. Lumbar stenosis is narrowing of the spinal canal and/or intervertebral foramina. As the patient extends his Lumbar spine (by standing or walking) the intervertebral foramen physiologically becomes more narrowed. If a patient has narrowing of this area neural impingement can occur. Patient's symptoms will usually improve with flexion of the lumbar spine (sitting). Radiographically, degenerative changes are present in patients with spinal stenosis. Spondylosis is a radiographic term for general degenerative changes. It is virtually a universal finding in elderly patients (even patients without spinal stenosis). Therefore out of the choices given, this is the most likely finding. A herniated disc is possible, however it is less likely because the pain is made better with lumbar flexion (sitting). The absent Achilles reflexes is likely due to diabetic peripheral neuropathy. Answer A Although we do not know the extent of this patient's prostate cancer, the likelihood of him having vertebral metastasis (usually seen as lytic lesions) is far less than the likelihood of spondylosis. Answer B Spondylolisthesis is a much less frequent a finding than spondylosis in the elderly population. Answer C Compression fractures of the lumbar spine are far less common than spondylosis. In addition, spondyltic changes are usually seen with and often occur before compression fractures. Answer E

Contraindicated in acute hepatitis

E. LIVER PUMP The liver pump is a lymphatic technique that involves patient cooperation (exhaling and inhaling) upon command, while the practitioner's hand leans on the thoracic cage applying a vibratory motion. This results in a pumping action in the liver. This is indicated in several conditions, such as congestive heart failure, congestion of the liver, etc. But it is contraindicated in acute hepatitis

Given the above finding what is the diagnosis at L5? A) Flexed, rotated left, sidebent left B) Extended, rotated left sidebent left C) Extended, rotated right, sidebent right D) Neutral, rotated right, sidebent left E) Neutral, rotated left sidebent right

A

Which of the following is associated with a right on left sacral torsion? A) A non-neutral dysfunction of L5 that is rotated left and sidebent left; and a positive seated flexion test on the right B) A non-neutral dysfunction of L5 that is rotated right and sidebent right; and a positive seated flexion test on the left C) A neutral dysfunction of L5 that is rotated right and sidebent left; and a positive seated flexion test on the right D) A neutral dysfunction of L5 that is rotated left and sidebent right; and a positive seated flexion test on the left E) A flexed dysfunction of L5 that is rotated right and sidebent right; and a positive seated flexion test on the right

A A right on left sacral torsion would have an L5 that is non-neutral (meaning either flexed or extended), rotated left, sidebent left with a seated flexion test positive on the right (opposite the axis of the torsion). A non-neutral dysfunction of L5 that is rotated right and sidebent right and a positive seated flexion test on the left would be associated with a left on right sacral torsion. Answer B A neutral dysfunction of L5 that is rotated right and sidebent left and a positive seated flexion test on the right would be associated with a left on left sacral torsion. Answer C A neutral dysfunction of L5 that is rotated left and sidebent right and a positive seated flexion test on the left would be associated with a right on right sacral torsion. Answer D A flexed dysfunction of L5 that is rotated right and sidebent right and a positive seated flexion test on the right could not be associated with any sacral torsion. Answer E

A 33-year-old female comes to your office with a three-day history of a severe headache. The pain originates from the base of the occiput and radiates into the right temporal region. The patient reports that she started experiencing nausea, vomiting and vertigo today. Pain intensifies in brightly lit rooms. She smokes one pack of cigarettes a day. Her allergies include ragweed. Osteopathic structural examination reveals tenderness in the right trapezius and right temporalis muscles. Palpation of these tender areas does not refer pain. There is a tenderpoint at the articular pillar of C2 on the right. Which of the following is true regarding her headache? A) It is likely due to an alteration of the intracranial blood supply B) It is likely due to hypertonicity of the trapezius muscle. C) Treatment may consist of myofascial stretch using vapocoolant spray D) Pain will resolve with counterstrain to the temporalis muscle E) It is likely due to degeneration of the joints of Luschka in the lower cervical spine.

A Classic migraines are thought to be caused by a vasoconstriction and vasodilatation of the blood supply to the cranium. Symptoms of a classic migraine includes photophobia. Nausea and vomiting are not uncommon. These symptoms can be preceded by an aura. Tension type headaches can be due to tension in the upper back or cervical spine musculature. Tension headaches are not usually associated with nausea and vomiting. Pain is often worse at the end of a workday due to muscle fatigue. Answer B Trigger points are taut myofascial bands that refer pain when compressed. They can be treated with "spray and stretch" technique using vapocoolant spray. The above patient does not have any trigger points. Answer C Counterstrain alone will not likely completely resolve the pain. Other treatments that would be useful would include techniques to reduce sympathetic tone to the head and neck structures to limit vasoconstriction. Answer D Degeneration of the joints of Luschka will likely result in axial cervical pain, or if there is compression of a nerve root, pain will likely radiate into the upper extremity. Answer E

A patient is diagnosed with a first-degree ankle sprain. What is the most likely injured ligament? A) Anterior talofibular B) Posterior talofibular C) Calcaneofibular D) Tibiocalcaneal E) Deltoid

A The most injured ligament of the foot is the anterior talofibular ligament. The posterior talofibular and calcaneofibular ligaments are less frequently injured and are associated with a grade II and III ankle sprains. Answers Band C The tibiocalcaneal ligament is a portion of the deltoid ligament. It is rarely injured. Answer D Since the ankle is stable in dorsiflexion and the deltoid ligament very strong, injury to this ligament is rare. Answer E

Which of the following physical signs may be observed in the above patient if there was compression of the S1 nerve root? A) Decreased ability for the patient to walk on his toes B) Decreased ability for the patient to walk on his heels C) Absent patellar tendon reflex D) Decreased sensation on the dorsum of the foot E) Sustained clonus with ankle dorsiflexion

A Decreased ability of the patient to walk on their toes. Although the S1 nerve root innervates several parts of the lower extremity, it is intimately involved in plantar flexion of the foot. Heel walking tests the strength of the main ankle dorsiflexor, which is the anterior tibialis. This muscle is innervated by L4 and L5 nerve roots. Answer B An absent patella reflex may indicate pathology of the L4 nerve root. Answers C Sensation at the dorsum of the foot is supplied by L5. Answer D Sustained ankle clonus is associated with an upper motor neuron lesion; a herniated disc at L5-S1 would result in nerve root (lower motor neuron) compression. Answer E

C3 is extended, sidebent left and rotated left. If you decide to use a direct muscle energy technique to correct this dysfunction how would you position C3? A) In flexion, sidebent right, rotated right B) In flexion, sidebent left, rotated right C) In extension, sidebent left, rotated left D) In extension, sidebent right, rotated right E) In extension, sidebent left, rotated right

A Direct muscle energy treatment would reverse all three planes of C3 (Le. C3 would be placed in such a way that the segment is against its restrictive barrier in all three planes). In order to correctly perform typical muscle energy the segment must be flexed, sidebent right, and rotated right . Flexing, sidebending left and rotating right would not reverses all 3 planes of motion. Answer B Extending, sidebending left, and rotating left will place C3 away from the restrictive barrier. This position would be used for an indirect treatment. Answer C Extending would not place the segment against its restrictive barrier. Answers D and E

Injury to the C5 and C6 nerve roots of the brachial plexus is most likely to produce which of the following effects? A) Waiters tip deformity B) Wrist drop C) Claw hand D) Benediction sign E) Klumpke's palsy

A Erb-Duchenne's palsy is an injury to the upper portion of the brachial plexus. This often involves the C5, C6 nerve roots, but can extend in some cases to involve the C7 nerve root. Classically Erb's palsy will result in a waiters tip deformity Wrist drop is due to injury to the radial nerve usually distal to the brachial plexus (C6, C7 & C8 nerve roots). Answer B Claw hand is due to injury to the median and ulnar nerves. Answer C Benediction sign (also known as bishops deformity) typically results from injury to the ulnar nerve distal to the brachial plexus (C8). Answer D Klumpe's palsy is due to injury to the lower brachial plexus. This typically involves the T1 and C8 nerve roots but may extend to include C7. Answer E

Which of the following is the best statement regarding typical muscle energy treatment for this innominate dysfunction? A) Hip flexors are isometrically contracted to provide the necessary force to correct this dysfunction. B) The patient is best treated in a seated position. C) The patient is treated with her hip flexed and slightly abducted D) Hamstrings are activated by the patient to correct the dysfunction E) Muscle energy treatment is relatively contraindicated in the above patient

A Hip flexors are used to correct the dysfunction. The treatment position to correct a left innominate posterior is as follows: With the patient supine, drop the left leg off the table to engage the restrictive barrier. Instruct the patient to flex their hip against your counterforce for 3-5 seconds. In the typical muscle energy treatment, for this dysfunction the patient should be supine, not seated. Answer B Hamstrings are hip extensors and would be used to correct an anterior innominate dysfunction. Answer D Since the patient is not critically ill, muscle energy is not contraindicated in this patient. Answer E

In this patient, which of the following choices is the most appropriate regarding treatment for the somatic dysfunction of ribs 5-7 using muscle energy? A) Treat rib 5 using the pectoralis minor muscle B) Treat rib 5 using the serratus anterior muscle C) Treat rib 6 using the pectoralis minor muscle D) Treat rib 7 using the serratus anterior muscle E) Treat rib 7 using the pectoralis minor muscle

A In a group exhalation dysfunction the key rib is the top most rib of the group, which is rib 5. The correct muscle to use for a muscle energy treatment of rib 5 is the pectoralis minor muscle. Rib 5 is the key rib, but it is not treated with the serratus anterior muscle. Answer B Ribs 6 and 7 are not the key ribs, therefore these answers can be easily eliminated. Answers C, D, and E

Muscle energy treatment of the somatic dysfunction of ribs 5-7 was discontinued because the patient complained of pain. Using counterstrain, where would you locate the tender point to direct your treatment for this group dysfunction? A) On the mid-axillary line on rib 5 B) On the mid-axillary line on rib 6 C) On the mid-axillary line on rib 7 D) On the rib angle of rib 5 E) On the rib angle of rib 7

A In a group exhalation dysfunction the key rib is the top most rib of the group, which is rib 5. Anterior tender points are associated with depressed ribs (exhalation dysfunctions). The tender point for rib 5 is located in the mid-axillary line. Ribs 6 and 7 should not be treated because neither are the key rib. Answers B, C and E Posterior tender points are associated with elevated ribs (inhalation dysfunctions). Answers D and E

20-year-old male presents with low back pain following a fall onto a concrete floor. The patient gives a history of episodic aching of the lumbar region prior to the fall. Examination of the patient in the prone position reveals a deep sacral sulcus on the left, a posterior/inferior ILA on the right when compared to the opposite side, and a lumbosacral junction that springs freely upon compression. The most likely diagnosis is? A) A forward sacral torsion on a right oblique axis B) A forward sacral torsion on a left oblique axis C) A backward sacral torsion on a left oblique axis D) A left unilateral sacral flexion E) Bilateral sacral extension

A In a patient with a deep left sulcus and a negative lumbosacral spring test the left portion of the sacrum has moved anterior. Since the above patient has a posterior/inferior ILA on the right this indicates that this portion of the sacrum has moved posteriorly. A forward sacral torsion on a right oblique axis (Le. a right rotation on a right oblique axis) is the only answer that would be consistent with the above findings. In a forward sacral torsion on a left oblique axis (left on left) the right sulcus would be deeper and the left ILA would be posterior/inferior. Answer B In a backward sacral torsion or a bilateral sacral extension, the lumbosacral spring test would be positive (Le. the lumbosacral junction would not spring). Answers C and E In a unilateral sacral flexion on the left, the left ILA would be posterior and significantly inferior. Answer D

In a patient with neck pain, the second cervical segment is found to resist translation to the left with the head in the flexed position. The segment translates equally to the left and right in the extended position. Given the above diagnosis, which is the correct muscle energy (direct) technique? A) C2 should be placed in flexion, right sidebending and right rotation and the patient would be asked to rotate his head to the left against resistance. B) C2 should be placed in flexion, left sidebending and left rotation and the patient would be asked to rotate his head to the right against resistance. C) C2 should be placed in extension, right sidebending and right rotation and the patient would be asked to rotate his head to the left against resistance. D) C2 should be placed in extension, left sidebending and left rotation and the patient would be asked to rotate his head to the right against resistance. E) C2 should be placed in extension, left sidebending and left rotation and the patient would be asked to rotate his head to the left against resistance.

A In the above question, C2 resists translation to the left with the head in the flexed position. Left translation induces right sidebending. Therefore, the cervical spine resists right sidebending. If it resists right sidebending then it must be sidebent left. Due to facet orientation, C2 always sidebends and rotates to the same side (type II mechanics). Therefore, if it is sidebent left, it must be rotated left. This resistance is worse when the head is in the flexed position, and eases with the head in the extended position. Since you name the dysfunction for where the segment wants to go C2 must be extended, sidebent left and rotated left. The correct muscle energy technique calls for a reversal of all planes of motion, therefore the segment should be placed in flexion, right rotation, right sidebending. And the patient would then be asked to rotate his head to the left.

The first cervical segment has the greatest degree of freedom in which plane(s)? A) Transverse B) Coronal C) Sagittal D) Oblique E) Transverse and Coronal

A Motion in the transverse plane is rotation. Segmental motion is conventionally defined as the specified segment on the one below. Since rotation is the main motion of the first cervical segment (the AA joint-the atlas on the axis) it has the greatest degree of freedom in the transverse plane. Motion in the coronal plane is sidebending. Answer B Motion in the sagittal plane is flexion/extension. Answer C The oblique plane is not a true description of a plane. Answer D Vertebral motion in the transverse and coronal planes refer to rotation and sidebending respectively. There is a slight amount of flexion allowed at the atlas, but no sidebending. The AA joint essentially allows rotation only. Answer E

Patient complains of hip pain for one day. Range of motion testing reveals: Left Right EXTERNAL ROTATION LEFT: 60 EXTERNAL ROTATION RIGHT: 60 INTERNAL ROTATION LEFT: 20 INTERNAL ROTATION RIGHT: 35 The most likely diagnosis is. A) Tight left external rotators B) Tight left internal rotators C) Lax left external rotators D) Tight right internal rotators E) Lax right internal rotators

A Normal values for range of motion varies due to age, gender and flexibility. Therefore it is more important to focus on asymmetry. The patient in the question has asymmetry of her internal rotators. The patient could have a problem on the right or left, however out of the answers given the only one that is possible is tight left external rotators. Tight left external rotators would decrease left internal rotation. Another possible answer is lax right external rotators. Lax right external rotators will increase right internal rotation, however this choice is not given. Tight left internal rotators would decrease left external rotation. Answer B Lax left external rotators would increase left internal rotation. Answer C Tight right internal rotators will decrease right external rotation. Answer D Lax right internal rotators will increase right external rotation. Answer E

Decreasing myofascial restrictions in which muscle would most likely result in greatest reduction in this patient's symptoms? A) Vastus lateralis B) Semimembranosus C) Semitendinosus D) Vastus medialis E) Quadriceps

A Patello-femoral syndrome is caused by a mal-tracking of the patella. Typically the patella tracks too far laterally, this is usually due to a weak vastus medialis and a tight vastus lateralis. Decreasing myofascial restrictions in the vastus lateralis would have the greatest effect on this patient's symptoms. Surgical treatment for patellofemoral syndrome consists of a lateral patella release. Decreasing myofascial restrictions in the vastus medialis could theoretically worsen this condition. Answer D Decreasing myofascial restrictions in other muscles may help, but considering the biomechanics of the problem, it will not have the greatest effect. Answers B, C and E

A female patient underwent a right-sided radical mastectomy 3 years ago for breast cancer. As a result, she has residual swelling in her right upper extremity. The patient completed a course of chemotherapy and was informed last year that she is cancer free. The patient wishes to have osteopathic treatment to decrease her edema. The physical exam reveals pitting edema in the right upper extremity. Which of the following is the most accurate statement regarding lymphatic treatment? A) Proper lymphatic treatment includes, opening the thoracic duct, then releasing the thoracoabdominal diaphragm then applying the posterior axillary fold technique B) Proper lymphatic treatment includes. a thoraco-abdominal diaphragm release. posterior axillary fold technique and pedal pump C) Lymphatic treatment is absolutely contraindicated in this patient because of the history of cancer. D) Lymphatic techniques will be of no use since the anatomy has been altered. E) Lymphatic treatment should be postponed until re-occurrence of cancer is ruled out.

A Proper lymphatic treatment includes, opening the thoracic duct, then releasing the thoracoabdominal diaphragm then applying the posterior axillary fold technique. Although variations in treatments vary, this is not the best choice because the patient should have her thoracic inlet released. The lymph from the right upper extremity will drain into the right lymphatic duct, which traverses the thoracic inlet. Answer B Lymphatic treatment is not absolutely contraindicated in a patient with a history of cancer. Answer C Lymphatic techniques have been shown to help in patients with post-surgical edema. Answer D Residual limb edema following radical mastectomy is not uncommon. As long as the edema did not develop recently or there was no increase in her current edema, the chance of cancer recurrence of cancer is low. Therefore the practitioner can proceed with lymphatic treatment. Answer E

A patient who recently had a parathyroid adenoma removed should have what spinal level examined for a viscerosomatic reflex? A) T1-T3 B) T5-T6 C) T7-T9 D) T10-T12 E) T12-L2

A The parathyroid glands are structures in the neck, and could have caused viscerosomatic reflex changes at the spinal level T1-T4. T5-T6 spinal levels are associated with the viscerosomatic reflex from the heart, lungs and esophagus. Answer B T7-T9 spinal levels are associated with the viscerosomatic reflex from the upper GI tract. Answer C T10-T12 spinal levels are associated with the viscerosomatic reflex from the middle GI tract and the gonads. Answer D T12-L2 spinal levels are associated with the lower GI tract. Answer E

A 46-year-old male presents to your office complaining of chronic lumbosacral pain. There is a negative standing flexion test and a positive seated flexion test. Structural examination reveals a deep sacral sulcus on the left, posterior/inferior ILA on the right, and the lumbosacral junction springs freely upon compression. The most likely diagnosis is: A) Right sacral rotation on a right oblique axis B) Left sacral rotation on a left oblique axis C) Right sacral shear D) Right sacral rotation on a left oblique axis E) Left sacral rotation on a right oblique axis

A Right sacral rotation on a right oblique axis. A right sacral rotation on a right oblique axis will result in a deep sacral sulcus on the left, a posterior/inferior ILA on the right, and the lumbosacral junction springs freely upon compression. A left rotation on a left oblique will result in a deep sacral sulcus on the right and a posterior/inferior ILA on the left. Answer B A right sacral shear can either be a right unilateral sacral flexion or a right unilateral sacral extension. A right unilateral sacral extension will result in a positive lumbosacral spring test. A right unilateral sacral flexion will result in a deep sacral sulcus on the right (not left). Answer C A backward sacral torsion (right-on-Ieft or a left-on-right) will result in a positive lumbosacral spring test (Le. the lumbosacral junction will not spring freely). Answers D and E

A patient has a tenderpoint at the right articular process of C5. What is the typical treatment position for the head and neck in order to treat this tenderpoint using counterstrain? A) Flexed, sidebent to the left, and rotated to the left. B) Extended, sidebent to the left, and rotated to the left. C) Flexed, sidebent to the right, and rotated to the right. D) Extended, sidebent to the right, and rotated to the right. E) Flexed, sidebent to the left, and rotated to the right.

A Tender points on articular processes are anterior tender points. Tender points on spinous processes (or just lateral to them) are posterior tender points. Anterior tender points are treated in flexion. Treatment position for most cervical tender points (including C5) would be to sidebend and rotate away from the side of the tenderpoint. Thus, putting all the information together, a right anterior cervical tenderpoint should be treated in flexion, sidebending to the left and rotating to the left. Answers B, C, D, and E

A mildly obese patient comes to your office complaining of buttock pain. She fell on her buttocks about one week ago and she still has some residual pain. Inspection of her lumbar spine and gluteal region reveals a decreased lumbar lordosis. Standing flexion test and seated flexion test show no evidence of asymmetry. Sacral sulci are equal in depth. The lumbosacral junction does not spring. Based on the information given what is the most likely diagnosis? A) Bilateral sacral extension B) Forward sacral torsion on a left oblique axis C) Anterior sacral base D) Unilateral extended sacrum on the right E) Unilateral extended sacrum on the left

A The above patient has a bilateral sacral extension (a.k.a. sacral base posterior or a posterior sacral base). The examiner will notice that the sacral sulci are equal in depth and there is no springing at the lumbosacral junction (i.e. the lumbosacral spring test is positive). The seated flexion test will be FALSELY negative - Why? Since both SI joints are restricted, asymmetry will not be appreciated. In a forward sacral torsion (Ieft-on-Ieft or right-on-right) the seated flexion test would be positive, the sulci would appear asymmetric, and the lumbosacral spring test would be negative. Answer B In an anterior sacral base (a.k.a. bilateral sacral flexion or sacral base anterior), the findings will be the same, except that the lumbosacral junction would spring (lumbosacral spring test is negative). Answer C In a unilateral extended sacrum on the right the seated flexion test will be positive on the right, the right sulcus will appear shallow (posterior) and the lumbosacral spring test will be positive. Answer D In a unilateral extended sacrum on the left, the seated flexion test will be positive on the left, the left sulcus will appear shallow (posterior) and the lumbosacral spring test will be positive. Answer E

A patient is found to have a posterior transverse process on the right at L2. Flexion of the lumbar spine increases the asymmetry at this segment, whereas extension restores symmetry. Which of the following is the correct muscle energy (direct) treatment position. A) Flexed rotated left sidebent left B) Flexed rotated right sidebent right C) Extended rotated right sidebent right 0) Extended rotated left sidebent left E) Neutral rotated right sidebent left

A The above patient has a posterior transverse process on the right at L2. Symmetry is restored in extension. Therefore the somatic dysfunction is extended rotated right, sidebent right. A muscle energy treatment is a direct treatment that entails positioning the segment against its restrictive barrier (Le. reversing all planes of motion). Thus, the correct treatment position in the above patient is flexed, rotated left and sidebent left (FRLSL).

While examining a patient, you notice that a child has a somatic dysfunction of rib 5 on the right side. The rib seems to lag with inhalation and move easily into exhalation. Which of the following is the best statement concerning the effected rib? A) This patient will have limited cephalad motion of rib 5 at the mid-axillary line B) This patient will have a tenderpoint in the pectoralis minor muscle C) This patient will display a winging of his scapula D) The fourth thoracic vertebrae is non-neutral sidebent right and rotated right E) The fourth thoracic vertebrae is non-neutral sidebent left and rotated left

A The above patient has an exhalation dysfunction of rib 5 (you name the dysfunction in the direction it wants to go). Although rib 5 primarily moves in a pump-handle fashion, there still is some bucket-handle movement associated with it. Bucket-handle movement is most easily palpated in the mid-axillary line. Therefore, there will be some restriction of cephalad motion of rib 5 in the mid-axillary line. The pectoralis minor originates at the coracoid process and inserts onto ribs 3 -5. A tender point in this muscle could be due to a somatic dysfunction of ribs 3 - 5, but not always. The tender point that is more specifically related to rib 5 would be either in the mid-axillary line (anterior) or on the rib angle (posterior) Answer B Scapular winging is related to weakness in the serratus anterior and can be due to injury to the long thoracic nerve. It is not necessarily related to a somatic dysfunction of rib 5. Answer C Although a rib dysfunction and thoracic dysfunction can easily be seen together, specific thoracic dysfunctions are not associated with specific rib dysfunction. Answers D and E

A 54-year-old nurse complains of paresthesias in her left hand. She describes intermittent numbness that worsens while sleeping. Ibuprofen improves her symptoms. She has difficulty handling small objects and buttoning shirts. On examination, she has decreased sensation on the palmar aspect of her second and third digit when compared to the fifth digit. There is mild atrophy of the left thenar eminence. Tender Points are present in the left anterior scalene. The first rib on the left has limited exhalation movement. The lower cervical segments are restricted in left sidebending. The primary reason for treating the cervical somatic dysfunction in the above patient is? A) To remove possible sites of additional neurologic compression and improve brachial plexus function B) To improve lymphatic drainage from the upper extremity thus decreasing distal nerve compression C) To improve vascular drainage from the upper extremity thus decreasing distal nerve compression D) To decrease sympathetic tone in the upper extremity E) Due to the distal site of injury in this patient, treatment of the axial spine will not further decrease this patient's symptoms.

A The above patient has carpal tunnel syndrome. Osteopathic treatment for carpal tunnel syndrome includes: 1. Treating rib and upper thoracic somatic dysfunctions to decrease sympathetic tone in the upper extremity. 2. Treating cervical somatic dysfunctions and myofascial restrictions to enhance brachial plexus function and remove potential sites of additional compression. 3. Treating the carpal tunnel with direct release techniques to increase the space in the carpal tunnel.

A 16-year-old female is experiencing low, midline, wave-like cramping pelvic pain that occurs with menses. The pain has been present for 3 months and has been getting progressively worse. Pain frequently radiates to her lumbar spine and is associated with frequent nausea. In addition to appropriate oral medications, which osteopathic manipulative technique will most effectively decrease the patient's symptoms by altering sympathetic tone? A) Rib raising and paraspinal inhibition to the thoracoLumbar junction B) Celiac ganglion release C) Pelvic diaphragm release D) Muscle energy to T1 0 E) Sacral inhibition

A The above patient has signs and symptoms of dysmenorrhea. Pain is produced by uterine vasoconstriction, anoxia, and sustained contractions mediated by prostaglandins. The uterus receives sympathetic innervation from T10 - L2 segments. Rib raising and paraspinal inhibition will decrease sympathetic tone, therefore these techniques will most effectively enhance blood flow and relax the uterus, thus decreasing pain. The purpose of the celiac ganglion release is to calm sympathetics from T5 - T9. This technique will not affect autonomic tone to the uterus. Answer B The pelvic diaphragm release will not effectively normalize sympathetic tone in the uterus. Answer C Although manipulative techniques at vertebral levels could alter autonomic function, muscle energy at T10 will not greatly affect sympathetic tone in the uterus because treatment of one segment is less likely to be as effective as compared to treatment of a few levels. Answer D Parasympathetic flow to the uterus originates in the pelvic splanchnic nerves (S2-S4). Treatment directed at the sacrum will influence parasympathetic tone, not sympathetic tone. Answer E

Which of the following is the most appropriate technique for treating the cause of the bibasilar crackles? A) Rib raising B) Condylar decompression C) Pedal pump D) Liver pump E) Abdominal pump

A The bibasilar crackles are due to limited chest wall motion. This patient has recently undergone a major surgery. Deep breathing is likely to cause him abdominal pain and pain medication may limit respiratory effort. Therefore, any manipulative treatment that will enhance respiration will be beneficial to this patient. Rib raising will increase chest wall motion, enhance diaphragmatic excursion and enhance lymphatic return. In addition, rib raising will help normalize sympathetic influence to the viscera. Condylar decompression may influence the vagus nerve, but this will not increase chest wall motion to remove the bibasilar crackles. Answer B The pedal pump, abdominal pump and liver pump are contraindicated in patients where there is a nearby incision or who have had recent abdominal surgery. Answers C, D and E

A posterior fibular head dysfunction can cause direct compression on which of the following peripheral nerves? A) Common fibular B) Obturator C) Sciatic D) Tibial E) Sural

A The common fibular nerve (a.k.a. common peroneal nerve) courses behind (posterior to) the fibular head. A posterior fibular head dysfunction can exert pressure directly on the common fibular nerve resulting in foot drop. Fibular head dysfunction will not directly compress the obturator or sciatic nerves. Answers B and C The tibial nerve courses through the popliteal fossa, it is not affected by a fibular head dysfunction. Answer D The sural nerve is made up of fibers from the common peroneal and tibial. It would not be compressed in fibular head dysfunction since these branches arise proximal to the fibular head. Answer E

The most likely etiology for the above condition is: A) Idiopathic B) Bacterial C) Developmental D) Viral E) Obesity

A The etiology of transient synovitis of the hip is idiopathic, but mild trauma at an age when the socket or acetabulum is not fully developed is a proposed theory. Bacterial would be the etiology of septic arthritis. Answer B Transient synovitis of the hip is not a developmental disorder. Answer C Although one theory proposes a viral etiology for transient synovitis of the hip, Virologic cultures of the effusion have not identified organisms. Therefore viral is not the best answer. Answer D Slipped capital femoral epiphysis is associated with obese children. Answer E

A patient is diagnosed with a right anterior innominate rotation. About which sacral axis does this rotation occur? A) Inferior transverse B) Oblique C) Middle transverse D) Sagittal E) Superior transverse

A The innominates rotate about an inferior transverse sacral axis with ambulation, and in posterior and anterior innominate somatic dysfunctions. Sacral rotation occurs about 2 oblique axes with ambulation and with sacral torsions. Answer B Postural motion occurs about a middle transverse sacral axis. Answer C Sacral margin somatic dysfunctions rotate about a vertical axis. Answer D Respiration and craniosacral motion occur about the superior transverse sacral axis. Answer E

Manipulation of which somatic dysfunction is considered essential for the patient's primary cause of low back pain? A) L1 FRLSL B) L5 FRLSL C) Sacrum D) Left innominate E) Right innominate

A The key somatic dysfunction is usually found at L1 or L2. Treatment of this "key" somatic dysfunction is essential for the patient's comfort and for effective, long lasting effects. Although treatment of other somatic dysfunctions may restore symmetry, these dysfunctions are not considered the key somatic dysfunction. Answers B, C, D, E

This condition could be prevented by which of the following? A) Gain weight, cushion on chair, frequent breaks B) Lose weight, hard chair, sit straight C) Gain weight, cushion on chair, no breaks D) Lose weight, cushion on chair, frequent breaks E) Gain weight, hard chair, frequent breaks

A The patient is cachectic, and doesn't have enough physical padding to prevent the mechanical irritation of the piriformis. He should take more frequent breaks from sitting and add a cushion to his chair to reduce the trauma of sitting and eventually receive nutritional counseling to help him regain his normal weight to prevent this condition. Losing weight and sitting in a hard chair would make the condition worse, and encourage constant irritation of the piriformis muscle. Answer B Gaining weight takes time and would help, but taking frequent breaks from sitting and moving around offers help immediately. While the cushion on the chair will help sitting for 15 hours a day, not taking sufficient breaks would mostly still aggravate an already irritated piriformis muscle. Answer C If the patient loses weight the condition would still occur and is therefore not the best combination. Answer D Gaining weight takes time and a hard chair would aggravate his condition, even with frequent breaks. Answer E

What is the theorized neurophysiologic mechanism by which the correct answer in the above question achieves its effect? A) The Golgi tendon organs senses a change in tension from the isometric muscle contraction, this causes a reflex relaxation of the agonist muscle fibers. Therefore, by reflex relaxation of the agonist muscle, the physician is then able to passively stretch the patient to the new restrictive barrier. B) Contracting the agonist muscle causes local prostaglandin release; this produces a localized muscle relaxation and analgesia. C) Contracting the antagonistic muscle, signals are transmitted to the spinal cord and through the reciprocal inhibition reflex arc; the agonist muscle is then forced to relax. D) An HVLA thrust is thought to forcefully stretch a contracted muscle producing a barrage of afferent impulses from the muscle spindles to the central nervous system. The central nervous system reflexively sends inhibitory impulses to the muscle spindle to relax the muscle. E) An HVLA thrust is thought to forcefully stretch the contracted muscle pulling on its tendon, activating the Golgi tendon receptors and reflexively relaxing the muscle.

A This correctly describes the neurophysiologic mechanism of the post-isometric relaxation type of muscle energy. Prostaglandins are not released with active muscle contraction and do not result in local muscle relaxation. Prostaglandins typically cause pain and inflammation; some anti-inflammatory medications block the release of prostaglandins. Answer B This correctly describes the neurophysiologic mechanism of the reciprocal inhibition type of muscle energy, not the post-isometric relaxation type of muscle energy. Answer C There are two theories by which HVLA is thought to achieve its effects. They are described by choices D and E. Answers D and E

A 21 year old complains of a headache, severe facial pain, greenish purulent nasal discharge and nasal congestion. At what spinal levels would you expect to find the tissue texture changes related to a viscerosomatic reflex? A) T1-T3 B) T4-T6 C) T7-T9 D) T10-T12 E) T12-L2

A This patient most likely has sinusitis, and would have viscerosomatic reflex changes at T1-T3 spinal levels. T4-T6 spinal levels are associated with the viscerosomatic reflexes from the lungs and esophagus. Answer B T7-T9 spinal levels are associated with the viscerosomatic reflexes from the upper GI tract. Answer C T10-T12 spinal levels are associated with the viscerosomatic reflexes from the middle GI tract and the gonads. Answer D T12-L2 spinal levels are associated with the lower GI tract. Answer E

Which of the following statements concerning treatment of rib dysfunctions is true? A) If a thoracic dysfunctions is present it should be treated before rib dysfunctions B) Exhalation dysfunctions cannot be treated with HVLA C) In general, posterior tender points are treated by rotating and sidebending the thorax toward the dysfunction D) Muscle energy techniques are contraindicated in elderly patients with COPD E) An inhalation dysfunction of ribs 6-9 can be corrected by contracting the serratus anterior muscle

A Thoracic dysfunctions should be treated before rib dysfunctions. Rib dysfunctions may be caused by corresponding thoracic dysfunctions. If a thoracic dysfunction is present treat the thoracic spine before treating the specific rib dysfunction. Rib exhalation dysfunctions can be treated using the Kirksville Krunch technique. The technique is the same except that the thenar eminence is placed under the corresponding rib angle. Answer B In general, posterior rib tender points are treated by rotating and sidebending the thorax away from the dysfunction. Answer C Although muscle energy techniques should not be performed on patients with low vitality who could be compromised by adding muscular exertion, elderly patients with COPD can typically manage active muscular contraction without exacerbating their COPD. Answer D An exhalation dysfunction of ribs 6-9 can be corrected with muscle energy by contracting the serratus anterior. Answer E

A 67-year-old obese diabetic male comes to the emergency room with severe shortness of breath. He appears slightly anxious. He denies any chest pain. Dyspnea worsens when he lies flat on his back. Dyspnea improves when he is placed in a semi-Fowlers position. He admits to a recent 20lbs weight gain, he has developed swelling of the legs, and has nocturia. He has a history of two myocardial infarctions. On examination, he is 5 foot 8 inches tall and weighs 290 pounds. BP 150/80, HR 105, RR 32 and shallow, Temp. 98.60. On auscultation, there is an 83 gallop rhythm, rales are present in the lower lung fields, abdominal examination reveals a soft and enlarged liver and grade 4 pretibial edema. EKG demonstrates normal sinus rhythm with no changes when compared to his previous hospital admission's EKG 2 months ago. Which of the following is the most appropriate for initial management of this patient's symptoms in the emergency room? A) Intravenous furosemide B) Oxygen, intravenous fluids and cardiac monitoring C) Oxygen and digitalis D) Intravenous tissue plasminogen activator E) Placement of a 8wan-Ganz Catheter

A Treatment of acute congestive heart failure includes oxygen and IV furosemide. Nitroglycerine, which also can be used, is a venodilator that can potentiate the effect of furosemide. Other agents include morphine (reduces anxiety and dilates pulmonary and systemic veins) and nitroprusside (a useful adjunct in the treatment of CHF due to acute valvular regurgitation or hypertension). Although oxygen and cardiac monitoring would be wise choices, additional intravenous fluids will worsen his congestive heart failure, the goal of initial treatment would be to remove excess fluid. Answer B Digitalis is useful in the long-term management of heart failure. It is not typically used in the treatment of acute congestive heart failure. Answer C Thrombolytic therapy (tissue plasminogen activator) is not used in congestive heart failure. Answer D Placing a Swan-Ganz catheter may be helpful in cases in which a prompt response to therapy does not occur. Answer E

A 67-year-old male with a history of congestive heart failure presents to the ER in a very anxious state with shortness of breath and bilateral lower extremity edema. After appropriate diuresis, the patient stabilizes. You decide to perform OMT to re-establish homeostasis and enhance lymphatic drainage. Which of the following describes the most appropriate and proper sequencing of treatment? A) Thoracic inlet release, rib raising, redome the diaphragm, pelvic diaphragm release, pedal pump. B) Pedal pump, pelVic diaphragm release, redome the diaphragm, thoracic inlet release, pectoral lift C) Pedal pump, pelvic diaphragm release, redome the diaphragm, rib raising D) Rib raising, CV4 technique, pectoral lift, pedal pump E) Redome the diaphragm, pelvic diaphragm release, pectoral lift

A Treatment to improve lymphatic return should start with removal of all central restrictions (release the diaphragm and rib raising), followed by release of the periphery (lymphatic pumps). The basic lymphatic treatment program includes: 1) Releasing the diaphragms: Restrictions within the thoracic inlet will obstruct lymph drainage from anywhere in the body. Redoming the diaphragm will produce effective pressure gradients and enhance lymph return. Restrictions within the pelvic diaphragm will decrease lower extremity lymphatic drainage. 2) Rib raising to reduce hypersympathetic activity (larger lymph channels receive sympathetic innervation and mobilizing ribs will also enhance respiration). 3) Lymphatic pumps: A pedal pump will promote further lymph return. There is a lot of debate about which diaphragm to release first; most say release the thoracic inlet first, but this answer is not fully established. What you should remember for the boards is to release the diaphragm and perform rib raising before starting lymphatic pumps. As a general rule, diaphragms are first released before applying lymphatic techniques. This ensures the least amount of lymphatic resistance and maximizes the performance of the lymphatic technique (in this case pedal pump). Answers B and C Although rib raising, pectoral lift and pedal pumps will enhance lymphatic drainage, this answer does not list any diaphragm releases and therefore it is not the best answer. Answer D Answer E is not the best answer because it does not address the lower extremity edema. Answer E

Given the patient's history and physical findings which of the following treatments would be inappropriate? A) Ultrasound to the shoulder B) Left upper extremity sling C) Ice to the shoulder region D) Fascial release of the shoulder E) Cervical HVLA

A Ultrasound, which is a form of deep heat, increases the inflammatory response and will worsen the patient's edema. Ice will help decrease edema following a musculoskeletal injury. It is indicated for the initial management of the injury. Answer B Although it is not recommended that the patient wear a sling for more than a week, it can be used initially to provide relative rest depending on the severity of the injury and patient's activities of daily living. Answer C A fascial release of the shoulder may be attempted to decrease myofascial restrictions and improve lymphatic return to reduce edema and promote healing. Answer D HVLA to the cervical spine is not contraindicated in this patient. It will not directly affect the glenohumeral joint. Answer E

Dysfunction of which cranial bone is most associated with tinnitus and vertigo? A) Temporal B) Parietal C) Occiput D) Ethmoid E) Sphenoid

A Vertigo and tinnitus is often associated with cranial nerve VIII dysfunction. Although somatic dysfunction of the sphenoid, occiput and temporal bones can affect cranial nerve VIII, vertigo and tinnitus is most associated with the temporal bone dysfunction. The parietal bone is not associated with any specific symptom. Answer B The occiput is associated with dysfunctions in cranial nerves VIII - XII. Although dysfunction of cranial nerve VIII can be due to occiput dysfunction, specifically tinnitus and vertigo is attributed to temporal bone dysfunction. Answer C The ethmoid is associated with CN I dysfunction, which will produce an altered sense of smell. Answer D Dysfunction of the sphenoid can cause dysfunction of several cranial nerves (CN 1- CN VIII) and result in a variety of symptoms. However, sphenoid dysfunction is not specifically related to tinnitus or vertigo. Answer E

A female patient complains of progressively worsening low back pain. The pain is located at the lumbosacral junction. Two days ago, she vaginally delivered a 9lbs. 20z baby boy. On examination the seated flexion test is negative. The most likely diagnosis is: A) Left on left sacral torsion B) Bilateral sacral flexion C) Right innominate anterior D) Bilateral sacral extension E) Right inferior pubic shear

B A bilateral sacral flexion occurs during the delivery phase of childbirth when the sacrum nutates (sacral base moves anteriorly) to allow more space for the fetus to pass through the pelvic outlet into the birth canal. If the sacrum fails to counter-nutate (sacral base moves posteriorly), a bilaterally sacral flexion dysfunction remains. Due to birth mechanics, bilateral sacral flexion is a common dysfunction in the postpartum patient. Since the sacrum is bilaterally flexed it would cause a falsely negative seated flexion test. A left on left sacral torsion would have a positive seated flexion test on the right. Answer A Right innominate anterior and a right inferior pubic shear are not specifically associated with childbirth. Answers C and E A bilateral sacral extension would have a falsely negative seated flexion test. However, it has not been specifically associated with childbirth. Answer D

The patient returns in two weeks for follow-up. He states that the pain has not improved. AP and lateral x-rays in your office showed a decreased subacromial space with a high riding humerus. What is the best radiographic study to determine the extent of this patient's injury? A) Computed tomography B) Magnetic resonance imaging C) Bone scan D) Axillary view radiograph E) Pet Scan

B A decreased subacromial space and a high riding humerus raises the likelihood of a complete rotator cuff tear. An MRI is the best imaging study to identify the extent of the injury. A CT scan (computed tomography scan) does not show the detail of soft tissue as would an MRI. Answer A A bone scan and PET scan would show general inflammation in the area of the shoulder but would not identify the extent of soft tissue injury. Answers C and E An axillary radiograph is the best view to determine if there is a shoulder dislocation. It will not identify a rotator cuff tear. Answer D

A patient was in their car, turned to the left, and reached for the seat belt when a sharp pain was felt in the low back area. The patient has never had back pain before and has no radiating numbness or tingling in the lower extremities. The structural examination reveals no lumbar dysfunction. The right sulcus is deep and the left ILA is posterior. The left superior sulcus does not spring as compared to the right, and the left ILA does not spring. The most likely diagnosis is: A) Left unilateral sacral extension B) Left sacral margin posterior C) Right sacral margin posterior D) Bilateral sacral flexion E) Bilateral sacral extension

B A left sacral margin posterior is the diagnosis. The left sulcus and the left ILA would be posterior and will resist springing. A left unilateral sacral extension would have the static findings of a shallow left sulcus, a superior and slightly anterior left ILA and a positive left seated flexion test. Answer A In a right sacral margin posterior both the right sulcus and the right ILA would be posterior and resist springing. Answer C A bilateral sacral flexion has equally deep right and left sulci with equally shallow ILA's that resist springing. Answer D A bilateral sacral extension has equally shallow right and left sulci, which resist springing and has equally deep ILA's. Answer E

A 34-year-old male presents to your office with severe low back pain. The pain started yesterday while he was working on his car and is now radiating into his lower extremities. What additional information in this patient's history would most impact the immediate diagnosis and treatment? A) Absent unilateral superficial abdominal reflex B) Presence of incontinence C) Loss of Achilles reflex D) Down-going plantar response with Babinski's testing E) Unilateral loss of cremasteric reflex

B A patient with low back pain radiating into BOTH lower extremities may be of neurogenic or musculoskeletal origin. Of all the neurogenic or musculoskeletal possibilities the most important entity to rule out is Cauda Equina Syndrome (CES). CES is an entrapment of terminal nerve roots of the spinal cord. This can be due to a central disc herniation. If S2-S4 nerve roots are involved incontinence can result. If this occurs, immediate surgical decompression is indicated. If decompression is delayed, this may result in irreversible incontinence. Unilateral loss of the superficial abdominal reflex indicates a lower motor neuron lesion from T7 - L2, depending on where the absence is noted. If a lower motor neuron lesion was suspected, such as a herniated disc, it may impact his diagnosis, but this treatment is still likely to be conservative management. Answer A If the patient had an absent Achilles reflex, this would indicate a lower motor neuron lesion at S1 If this were present, the treatment would still likely be conservative. Answer C A downgoing plantar response when trying to elicit a Babinski reflex is a normal adult response. In other words, there is no Babinski reflex therefore, there is no suspected upper motor neuron lesion. However, the patient may still have a LMN lesion present. Answer D A unilateral loss of the cremasteric reflex suggests a lower motor neuron lesion between L1 and L2. This finding may impact diagnosis and treatment, however it would not impact the diagnosis and treatment as much as it would if it were cauda equina syndrome. Answer E

What is the innominate somatic dysfunction? A) Left innominate anterior B) Left innominate posterior C) Right innominate anterior D) Right innominate posterior E) Not enough information to determine

B A positive standing flexion test on the left indicates a left innominate problem. The left ASIS is cephalad and the left PSIS is caudad, these findings are indicative of a posterior innominate on the left. In a left innominate anterior, the left ASIS would be caudad and the left PSIS cephalad. Answer A In a right innominate anterior or a right innominate posterior, the standing flexion test will be positive on the right. Answers C and D

13-year-old male comes to the emergency room with right wrist and elbow pain following a fall. He states that he fell forward on his outstretched arm. Based on the history, what is the most likely diagnosis at the elbow? A) Adducted ulna B) Posterior radial head C) Abducted ulna D) Cubital tunnel syndrome E) Medial epicondylitis

B A posterior radial head may present as a sharp pain at the wrist or elbow and may result from a fall on an outstretched arm. On examination, the posterior radial head will resist anterior glide at the elbow and the forearm will be restricted with supination. The other answers listed are possible, however none are associated specifically with a fall on an outstretched arm. Answers A, C, D and E

The innominate diagnosis is: A) Left anterior rotation B) Right anterior rotation C) Left posterior rotation D) Right posterior rotation E) Indiscernible from the information given

B A right anterior innominate rotation has a lower right ASIS than the left, a right PSIS higher than the left, and a positive standing flexion test on the right. A left anterior and left posterior innominate rotation would have positive standing flexion tests on the left. Answers A and C A right posterior rotation would have the same standing flexion test, but would have a higher ASIS on the right than the left, and the right PSIS would be lower than the left. Answer D

A 56-year-old obese male is admitted to the hospital with severe right upper quadrant pain radiating to the tip of his right scapula. The pain started suddenly after eating a fatty meal. He has had one episode of vomiting. Examination demonstrates pain and guarding of the right upper quadrant. Lab values are as follows: Alkaline Phosphatase = 220U/L Alanine aminotransferase =17U/L Aspartate aminotransferase =20U/L Lactate dehydrogenase = 250U/L Bilirubin, serum Total =1.3mg/dl Normal = 0.1 - 1.Omg/dl Direct =0.7mg/dl Normal =0.0 - 0.3mg/dl The tenderness that he is experiencing at the tip of his scapula could be most accurately described as: A) Somatic-visceral reflex B) Viscero-somatic reflex C) Chapman's point D) Trigger point E) Counterstrain tenderpoint

B A viscero-somatic reflex occurs when localized visceral stimuli (in this case, from the gallbladder) produce patterns of reflex response in segmentally related somatic structures (in this case, at the tip of the scapula). A somato-visceral reflex occurs when abnormal somatic stimuli enters the spinal cord and results in altered function of the visceral structure. Answer A Although a Chapman's reflex point does represent a viscero-somatic reflex, it typically is described as a smooth firm, discretely palpable nodule approximately 2 -3 mm in diameter located within the deep fascia or on the periostium of the bone. It is not typically described as generalized tenderness. In addition, the posterior Chapman's point for the gallbladder is located at the T6 spinous or transverse process on the right. Answer C A trigger point is "a hypersensitive focus, usually within a taut band of skeletal muscle or in the muscle fascia". Again the above question does not describe discrete tenderness, therefore it cannot be assumed that the patient has a trigger point. Answer D A Jones tenderpoint is a small, tense area usually in the belly of a muscle that is used to diagnose and treat somatic dysfunctions. The question does not describe a discrete tense area, therefore, it cannot be assumed that this patient has a counterstrain tenderpoint Answer E

The correct typical counterstrain position to treat the above tenderpoint is: A) Lumbar flexion B) Lumbar extension C) Left leg adduction D) Left leg abduction E) Left lumbar sidebending

B According to Yates the typical treatment position for a tender point at the transverse process of L5 is lumbar extension, sidebending away with rotation either toward or away. Jones just specifies lumbar extension

4-year-old child with asthma is brought to the ED by his father. The child developed an upper respiratory tract infection 48 hours ago. Over the past hour his breathing has become increasingly labored and he has started to develop respiratory distress. His father has administered one albuterol treatment via a nebulizer one hour ago; there was slight improvement in the patient's symptoms. On examination, respiratory rate =50. Inspection of the chest wall reveals subcostal retractions and evidence of accessory muscle use. Which of the following can be done initially to improve the patient's symptoms? A) Oxygen, oral albuterol, intravenous corticosteroids, and thoracic manipulation to stimulate sympathetics. B) Oxygen, inhaled albuterol, intravenous corticosteroids and seated thoracic pump technique. C) Oxygen, inhaled albuterol, zafirlukast, and manipulation to the cervical spine to normalize vagal tone. D) Oxygen, oral corticosteroids, and HVLA to rib 3. E) Oral albuterol, cromolyn sodium, and CV4 technique.

B Acute management of asthma includes supplemental oxygen, inhaled bronchodilators (albuterol) and intravenous corticosteroids. Osteopathic techniques that have been used successfully for asthma include the seated thoracic pump technique, CV4 technique, and cervical spine manipulation to normalize the vagus. Although the patient did not respond to one treatment of inhaled albuterol, additional doses can be administered every 20 minutes. Oral albuterol is not recommended if inhaled medications can be administered. Answer A Zafirlukast is an oral leukotriene receptor agonist. It is useful for patients with mild to moderate symptoms, however it's onset of action is delayed for approximately 2 weeks. Answer C Oral corticosteroids may be effective, however the preferred route of administration is IV. The asthmatic patient will usually present with a rib 3 or 4 dysfunction, however, as with most viscero-somatic reflexes, it responds poorly to HVLA. Answer D Cromolyn sodium is a medication that is useful in patients with mild asthma, especially allergic asthma. 34 However, onset of action is delayed several weeks. Answer E

The patient returns to your office for follow-up in three days. She reports that her headache has resolved and she was able to get some rest after your OMT treatment. Which of the following best indicates follow-up care? A) Since symptoms have resolved, there is no reason to examine the patient and no need for further OMT. B) Re-evaluate your patient today and determine if further intervention is necessary. C) Repeat the same OMT treatment today and have the patient return in one week if her symptoms return. D) Repeat the same OMT treatment two more times to insure adequate resolution of somatic dysfunction. E) Refer to physical therapy for initiating ultrasound therapy to the trapezius.

B Although the patient's symptoms have resolved, the patient still may have some somatic dysfunction. Somatic dysfunctions, such as cranial dysfunctions and axial dysfunctions may produce fascial strains that are transmitted to the head and may play a role in the origin of a migraine. If somatic dysfunction is found, treatment may prevent further headaches. Therefore, it is best to evaluate this patient again. Although the patient's symptoms have resolved, the patient still may have some trapezius tenderness and some somatic dysfunction, therefore the patient should be examined. Answer A All OMT treatments should be individualized to the patient's findings. Therefore it is recommended that the patient be examined and OMT should be directed specifically toward her dysfunction. It would be a disservice to the patient just to repeat the same treatment. Answers C and D Although ultrasound is a form of deep heat and can help loosen tight muscles, this is not the best choice for two reasons. Ultrasound was not used initially; therefore you do not know if it the patient will benefit from it. Second, ultrasound should be used in addition to cervical stretching and strengthening in physical therapy. Answer E

A patient was snowboarding and fell several times. The patient now complains of left arm pain. The findings of the structural exam are: a decreased carrying angle of the left arm, the olecranon process is restricted in medial glide, and the wrist and hand are abducted. The most likely diagnosis is? A) Posterior radial head dysfunction B) Adduction dysfunction of the ulna C) Abduction dysfunction of the ulna D) Anterior radial head dysfunction E) Cervical radiculopathy

B An adducted dysfunction of the ulna would have a decreased carrying angle, the hand and wrist would be abducted and the olecranon process would prefer lateral glide. A posterior radial head dysfunction would cause the forearm to prefer pronation, and the radial head would resist anterior glide. Falling on a pronated forearm causes this dysfunction. The carrying angle is not affected in radial head dysfunctions. Answer A An abducted ulna would have a increased carrying angle; the olecranon process would be restricted in medial glide, and the wrist and hand would be adducted. Answer C An anterior radial head dysfunction would cause the forearm to prefer supination and the radial head would resist posterior glide. Falling backward on a supinated forearm can cause this dysfunction. The carrying angle is not affected in radial head dysfunctions. Answer D Cervical radiculopathy is nerve root compression. It can occur with trauma, usually to the neck, not to the upper extremity and typically numbness and tingling in the extremity would be present. Answer E

The above patient's condition has been most closely associated with: A) Genu recurvatum B) Increased Q angle C) Decreased Q angle D) Coxa valgus E) Coxa varus

B An increased a angle has a major effect on the tracking of the patella. As the a angle increases, the patient's knees become more knock-kneed (genu valgus), causing an abnormal tracking of the patella, predisposing the patella to irregular or accelerated wear. The patella may even sublux laterally with these biomechanical forces, especially with dysfunction or weakness of the vastus medialis muscle. Genu recurvatum is a hyperextension of the knee due to ligament laxity, also called back knee, and has not been associated with patella-femoral syndrome. Answer A A decreased Q-angle is not associated with patellofemoral syndrome; see explanation for answer B. Answer C Coxa valgus occurs when the angle between the neck and shaft of the femur is greater than 135 degrees. This has no association with patellofemoral syndrome. Answer D Coxa vara has not been associated with patellofemoral syndrome. Answer E

Which of the following is typically seen in golfers elbow. A) A history of overuse of the wrist extensor muscles. B) Pain with resisted wrist flexion C) Positive Tinel's sign at the cubital tunnel D) Posterior radial head dysfunction. E) Pain at the olecranon process of the ulna.

B Golfers elbow (a.k.a. medial epicondylitis) can be due to an overuse injury of the forearm pronators and wrist flexors. Patients typically have pain with resisted wrist flexion. The patient with medial epicondylitis usually has a history of overuse of the wrist flexors muscles, not wrist extensors. Answer A The ulnar nerve runs through the cubital tunnel. Entrapment of this nerve at this location can result in a positive Tinel's sign. It is not associated with medial epicondylitis. Answer C Radial head dysfunction has not been linked to epicondylitis. Answer D Pain at the olecranon process may been seen in triceps tendonitis or olecranon bursitis. Answer E

A patient with an upper respiratory tract infection is having difficulty clearing his secretions. Which of the following treatments would most effectively thin this patient's secretions?

B Hypersympathetic activity to the respiratory epithelium increases the number of goblet cells in the respiratory tract. This encourages the production of profuse, thick secretions. Sphenopalatine ganglion treatment stimulates parasympathetic activity and reduces the number of goblet cells and increases the proportion of ciliated columnar cells. Rib raising to the upper thoracic region will also help normalize sympathetic tone and help reduce the number of goblet cells, thus thinning out secretions. Frontonasal suture spread encourages the ethmoid bone to move freely. This encourages natural pumping action through the craniosacral mechanism and improves sinus drainage. However, it will not change the consistency of the secretions. Answer A CV4 decompression technique primarily enhances the amplitude of the CRI. It will not effectively thin this patient's secretions. Answer C Redoming the diaphragm improves lymphatic drainage, venous return and improves immune function. It certainly has a role in treatment of the upper respiratory tract infection, however it will not change the consistency of the secretions. Answer D Thoracic inlet release will improve lymphatic return from the structures form the head and neck. However, it will not change the consistency of the secretions. Answer E

Which of the following statements is true of a left cranial torsion of the sphenobasilar synchondrosis? A) The sphenoid rotates about an AP axis so that the left greater wing is inferior B) The sphenoid and occiput will rotate in the opposite directions about an AP axis C) The sphenoid will rotate left about a vertical axis D) The occiput will rotate left about a vertical axis E) The sphenoid will rotate anterior about a transverse axis

B In a cranial torsion of the SBS (which is the same as a cranial torsion- just a different way of saying it) the sphenoid and occiput rotate in opposite directions about an AP axis. Specifically in a left cranial torsion, the sphenoid will rotate so that the left greater wing is superior. In a left cranial torsion, the sphenoid will rotate so that the left greater wing of the sphenoid is superior, not inferior. Answer A The sphenoid and occiput rotate about vertical axes in lateral strains. Answers C and D The sphenoid will rotate about a transverse axis with flexion and extension phases of the CRI. Answer E

Complications from the adverse effect in the previous question can be avoided by? A) Yearly bone densiometry B) Concomitant use of misoprostol C) Yearly colonscopy D) Frequent monitoring of liver enzymes E) Frequent monitoring of blood urea nitrogen and serum creatinine

B In a double blind, randomized, placebo-controlled study 51 misoprostol decreased serious gastrointestinal complications by 40%. Since NSAID use has not been linked to osteoporosis or diverticulitis, yearly bone densitometry or yearly colonoscopy will not prevent NSAID induced complications. Answers A and C NSAIDS have been associated with reversible (not irreversible) hepatocellular toxicity and renal failure is uncommon; therefore these are not the best answers. However, it is recommended that a complete blood count, liver function tests, renal function as well as stool guiac testing be done every 4 months in patients on long term Answers D and E

In order to correct the somatic dysfunction, treatment should be directed at which rib? A) Rib 2 B) Rib 3 C) Rib 4 D) Rib 5 E) Rib 6

B In a group dysfunction usually one "key" rib is responsible in causing the dysfunction. In exhalation dysfunctions the "key" rib is the uppermost rib of the dysfunction, in this case rib 3. If the patient were to have an inhalation dysfunction, treatment would be directed at rib 5

A 33-year-old patient with new onset left-sided low back pain comes into your office. Pain stems from the low back and radiates to the left knee. Structural examination reveals the following: - A tender point located at the right transverse process of L5 - L5 neutral, rotated right and sidebent left - Positive seated and standing flexion test on the right - Deep right sacral sulcus, Left inferior lateral angle is posterior and inferior - Springing at the lumbosacral junction is present - PSIS on the right is caudad compared to the left PSIS with the patient in the prone position - Apparent shorter leg on the right What is the most likely innominate diagnosis? A) Right innominate anterior B) Right innominate posterior C) Right superior innominate subluxation D) Right inferior innominate subluxation E) There is not enough information in the question to differentiate from the above

B In a right posterior innominate the standing flexion test is positive on the right, the right ASIS is cephalad, the right PSIS is caudad and there is a relative short leg on the right. In an anterior rotated innominate the PSIS on the right would be cephalad compared to the left PSIS. Answer A In a right superior innominate subluxation (a.k.a. innominate upslip, superior innominate shear) the right leg would be shorter and the right PSIS would be cephalad. Answer C In a right inferior innominate subluxation (a.k.a. innominate downslip, inferior innominate shear) the right PSIS would be caudad and the right leg would appear longer. Answer D There is enough information provided to choose one of the above diagnoses. Answer E

You are examining a 29-year-old male presenting to the emergency room complaining of right-sided flank pain. The pain is intermittent, excruciating and radiates to the right testicle. Physical exam reveals: Vitals: Temp: 98.6F, Heart Rate: 105 Respiratory Rate: 20 Blood pressure 140/90 Pain: 10/10 Abdomen: There is no rebound tenderness or signs of peritoneal irritation. There is costovertebral angle tenderness on the right. Laboratory data: Urinalysis: RBC's: too numerous to count, small leukocytes, nitrite negative. The CBC is normal. Radiological data: KUB demonstrates a 6 mm well circumscribed radio-opaque mass approximately 10 cm superior to the middle of the right pubic ramus. IVP confirms the presence and location of the stone with severe hydronephrosis Which of the following is true regarding osteopathic treatment for the above patient? A) All osteopathic treatment is absolutely contraindicated in the above patient, until the patient's pain is controlled. B) Due to the patient's excruciating pain, osteopathic treatment should be limited to gentle techniques such as rib raising and paraspinal inhibition only. C) High velocity thrust to the thoracoLumbar junction will decrease ureterospasm via a CNS inhibitory reflex. D) Pain would decrease with the treatment of a posterior tender point. E) Post-isometric relaxation muscle energy is indicated

B Initial management of ureterolithiasis includes rib raising and paraspinal inhibition to decrease hypersympathetic tone. Other OMT should be delayed until at least partial pain management has been achieved. Not all osteopathic treatment is contraindicated in the above patient provided standard medical protocols are followed. Answer A High velocity thrust and post isometric relaxation muscle energy is contraindicated in the above patient, due to the nature of the illness and level of patient's pain. Answers C and E Treatment of a posterior tenderpoint is likely to worsen this patient's pain, because it typically requires the patient to be in extension. In addition, treating the cause of the pain (ureteral stone) would decrease the patient's pain, not just counterstrain. Answer D

In a patient with low back pain structural examination reveals: A deep sacral sulcus on the right. A posterior ILA on the left. A negative lumbosacral spring test. A positive seated flexion test on the right. Given these findings, which is the most likely diagnosis? A) Right sacral rotation on a right oblique axis B) Left sacral rotation on a left oblique axis C) Left sacral rotation on a right oblique axis D) Right sacral rotation on a left oblique axis E) Unilateral sacral flexion on the right

B Left sacral rotation on a left oblique axis. The patient has a positive seated flexion test on the right. In sacral rotation on an oblique axis, the . seated flexion test is positive on the opposite side of the axis. Therefore we can eliminate nght rotation on a right oblique axis. Answer A The patient has a negative lumbosacral spring test. A lumbosacral spring test will be positive (no spring at the lumbosacral junction) if part of or the entire sacral base has moved posteriorly. The sacral base will move posterior a with backward sacral torsion (a right rotation on a left oblique axis or left rotation on a right oblique axis. Answers C and D In a unilateral sacral flexion on the right the patient will have a posterior and inferior ILA on the right (not the left). Answer E

Which of the following is the best treatment for this patient's scoliosis? A) Only OMT would be needed to completely straighten the scoliotic curve B) Physical therapy, OMT and Konstancin exercises C) Physical therapy, OMT, Konstancin exercises, and spinal bracing D) Physical therapy, OMT, Konstancin exercises, spinal bracing and botulinum toxin A injections into the paraspinals E) Neurosurgical referral

B Mild scoliosis (Cobb angle of 5 - 15) is typically treated with conservative measures. This includes: physical therapy, Konstancin exercises and OMT. Konstancin exercises are a series of specific exercises that has been proven to improve the patient with scoliosis postural decompensation. OMT is not intended to completely straighten scoliotic curves. Answer A Conservative therapy and spinal bracing are indicated in patients with moderate scoliotic curves Answer C Botulinum toxin A (BOTOX) injections along with conservative management and bracing have been proven to be helpful in some small pilot studies in patients with moderate scoliosis. However, it is not considered the standard of care. Answer D Neurosurgical referral is indicated for patients with severe scoliosis. Answer E

In addition to osteopathic treatment, patient education, and a short course of physical therapy, you decide to prescribe a non-steroidal anti-inflammatory medication. A significant concern regarding long-term use of this medication is: A) Osteoporosis B) Upper gastrointestinal bleed C) Diverticulitis D) Irreversible hepatocellular toxicity E) Renal failure

B NSAID-associated gastropathy is one of the most prevalent serious drug toxicities in the United States. Approximately 1 - 2% of patients using NSAIDS for three months and approximately 2 - 5% of patients using NSAIDS for one year have gastrointestinal ulcers, bleeding or perforation. NSAID use has not been linked to osteoporosis or diverticulitis. Although long term use of corticosteroids will lead to osteoporosis. Answers A and C NSAIDS have been linked with reversible hepatocellular toxicity. Liver enzymes usually revert to normal after the drug is discontinued. As a result, it is recommended that liver enzymes are checked periodically in patients on long term NSAIDS. Answer D NSAIDS may increase serum creatinine concentrations in some patients. However, renal failure is uncommon in patients without underlying renal impairment. Answer E

A 29-year-old female in her 36th week of pregnancy is having some leg edema. The most effective initial treatment is A) Pedal pump B) Thoracic inlet release C) Rib raising to the thoracoLumbar junction D) Place the patient in the Trendelenburg position E) Muscle energy to L1 - L3

B Prior to lymphatic pumping techniques, the thoracic inlet should be released to remove any impedance into the thoracic duct and ultimately the central circulation. The thoracic inlet should be released first before a lymphatic pump is attempted. Answer A Edema in the lower extremities is due to the mechanical effect of the uterus and abdominal contents pushing against the inferior vena cava. Rib raising to the thoracoabdominal junction will normalize sympathetics, but in the above case sympatheticotonia (increased sympathetic tone) is not the cause of the edema. Answer C Placing the patient in the Trendelenburg position will increase the pressure against the diaphragm and may make the patient short of breath. Therefore, it is not appropriate. Answer D Muscle energy will do little to improve this patient's lower extremity edema. Answer E

In a patient with lumbosacral pain, there is a positive seated flexion test on the left. The left sacral sulcus is deep and moves freely with anterior glide. The right ILA resists anterior glide. Based on the information given what is the most likely diagnosis? A) Unilateral sacral flexion on the right B) Right sacral torsion on a right oblique axis (R on R) C) Left sacral torsion on a left oblique axis (L on L) D) Left sacral torsion on a right oblique axis (L on R) E) Right sacral torsion on a left oblique axis (R on L)

B Right sacral torsion on a right oblique axis. In a sacral torsion, the seated flexion test (the test that checks for sacral dysfunction) is positive on the opposite side of the axis. A left deep sulcus that moves freely with anterior glide (springing) and a right ILA that resists springing indicates right rotation. In a left sacral torsion on a left oblique axis, a right sacral torsion on a left oblique axis and in a unilateral sacral flexion on the right, the seated flexion test would be positive on the right. Answers A, C and E In a left sacral torsion on a right oblique axis, the left sacral sulcus would be posterior (shallow) and it would resist anterior glide. In addition the right ILA would be anterior and move freely with anterior glide. Answer D

In addition to osteopathic treatment the most appropriate oral medication to treat this patient's pain is: A) Ibuprofen B) Rofecoxib C) Indomethacin D) Depomedrol E) Hydrocodone

B Rofecoxib is a non-steroidal anti-inflammatory agent that achieves its effects by inhibiting cyclooxygenase 2 (COX-2). By selectively inhibiting COX- 2 it is less likely to cause a GI bleed. Since the patient is on Coumadin (Warfarin) the patient will be at a greater risk if a GI bleed develops. Another medicine that would be indicated for this patient is acetaminophen. Ibuprofen and indomethacin are NSAID's that would put the patient at an increased risk for GI bleed. Answers A and C Rotator cuff tears are not treated with oral steroids due to the numerous adverse effects of this class of medications. The same beneficial effects can be achieved with non-steroidal anti-inflammatory agents. Answer D Narcotics are reserved for pain that does not respond to non-narcotic medication. Answer E

You are assessing a newborn in the hospital nursery. The child was born yesterday, delivered at 41 weeks by spontaneous vaginal delivery complicated by shoulder dystocia. At birth the child was 10 lbs 20z. in weight and measured 21 inches from head to toe. The Apgars were 9 and 10. The mother has been attempting to breast-feed, however the child is having difficulty latching on to the breast. On examination, the infant appears to be nervous and irritable. Flexing and abducting the hips while exerting pressure on the greater trochanters demonstrates that the child has a right hip click. What is the most likely cause of the shoulder dystocia? A) Gynecoid pelvic shape B) The child was macrosomic C) The mother did not receive prenatal care D) The obstetrician probably used forceps during delivery E) Premature rupture of membranes

B Shoulder dystocia is the inability of the child's shoulders to fit comfortable through the mother's pelvis. Shoulder dystocia is often associated with macrosomia. Any neonate weighing over 4000g is considered macrosomic. The gynecoid pelvis typically has adequate space along the birth canal when compared to other pelvic types. Therefore it is less likely cause of shoulder dystocia. Answer A Maternal prenatal care and forceps delivery are not associated with shoulder dystocia. Answers C and D Premature rupture of membranes is not associated with shoulder dystocia. Answer E

The patient's wrist is splinted and he is told to follow-up in three weeks. At the next follow-up visit, the patient's pain has not improved. Which of the following would be the appropriate course of action? A) Prescribe a more effective pain medication and have the child follow up in a week B) Order a bone scan C) Immobilize the wrist for 2 weeks with a cast D) Reassure the parent and the child that the inflammation and pain are typical of wrist sprains E) None of the above are correct

B Since the patient had pain at the anatomic snuff box and symptoms have not improved in 3 weeks time, the diagnosis of fracture should be considered. Although the X-rays of the wrist are normal, the patient may still have a small non-displaced scaphoid fracture. The scaphoid bone is the most common bone fractured in the wrist and often presents with pain in the anatomic snuffbox. X-rays may be normal for a week or longer. If a fracture is suspected, the practitioner may obtain a second set of x-rays, or a bone scan can be ordered. A bone scan will accurately identify a fractured scaphoid. Putting the child on a pain medication without immobilizing the wrist is ill advised, especially with the likelihood of a fracture. Answer A Appropriate management for a scaphoid wrist fracture is immobilization of the wrist for 3 months. Due to the lack of a rich blood supply, the healing of a fractured scaphoid is characteristically slow. Immobilizing the wrist for two weeks will not allow the fracture to fully heal. Answer C Wrist sprains typically improve within a week's time. If the swelling and pain are increased, the diagnosis of a fracture should be considered. Answer D

Which of the following is common in lumbar stenosis with neurogenic claudication, but is not typical in peripheral vascular disease with vascular claudication? A) Pain relief with sitting B) Increased pain when treating a typical posterior lumbar tender point C) Absence of lower extremity weakness D) Increased lower extremity pain with walking E) Posterior calf pain

B Spinal extension decreases the size of the intervertebral foramina and it decreases the AP diameter of the spinal column. Spinal extension is necessary when treating a typical posterior lumbar tender point. Extension increases pain associated with spinal stenosis, but will not affect the patient with vascular claudication since the patient is at rest. Sitting flexes the spinal column, thus decreases pain associated with neurogenic claudication. Sitting (resting) will also decrease the symptoms associated with vascular claudication. Answer A Lower extremity weakness is seen in patients with neurogenic claudication, but not in patients with vascular claudication. Answer C Symptoms of lumbar spinal stenosis (pain, numbness and sometimes weakness in the lower extremities) typically increased with extension of the spinal column (standing, walking). Symptoms of peripheral vascular disease, (leg pain and cramping) increases with activity and decreases with rest. Walking produces symptoms in both vascular and neurogenic claudication. Answer D Posterior calf pain is common in both neurogenic and vascular claudication. Answer E

An 88-year-old male with chronic peptic ulcer disease may have changes related to viscerosomatic reflexes in which of the following areas? A) T1 - T4 B) T5 - T7 C) T9 - T11 D) T11 - L2 E) L3 - L5

B Sympathetic control for the upper GI tract is via TS - T9 visceral efferents. Abnormal stimuli from visceral afferents enter the spinal cord, sensitize interneurons and produce an exaggerated motor response (a process called facilitation) at these segments. Head and neck =T1 - T4. Answer A T9 - L2 = left colon and pelvic structures. Answers C and D L3 - L5 = Nothing!!! Answer E

A 53-year-old male comes to the emergency department complaining of low back pain. He states that the pain he is experiencing now is similar to his back pain that he had when he herniated a disc two years ago. He brings in x-rays and an MRI from two years ago. X-rays show spondylosis at L5/S1. The MRI demonstrated a left posterior lateral herniated disc at this level. The back pain radiates from the lumbar spine into his left foot. Which of the following maneuvers would be expected to decrease this patient's pain? A) Active lumbar flexion in a standing position B) Passive lumbar extension in the prone position C) Valsalva maneuver D) Straight leg raising E) Ambulation

B Symptoms from a herniated disc typically improve with lumbar extension. Therapeutic exercises called the McKenzie program has been specifically designed to reduce low back pain caused by herniated discs. Most of the McKenzie exercises are extension-based maneuvers. The theory behind these extension techniques is that lumbar extension draws the nucleus back into the center of the intervertebral disc. Active lumbar flexion will push the disc posteriorly, this typically worsens the patient's pain. Answer A Valsalva maneuvers will typically increase pain associated with a herniated disc. Answer C Straight leg raising test will stretch the sciatic nerve, this typically causes the nerve to be pulled over the herniated disc, thus causing pain. Answer D Ambulation increases axial load and compresses the intervertebral disc resulting in increased pain. Answer E

A 47-year-old female comes in with low back and buttock pain. On structural examination she has a positive seated flexion test on the left. The right sulcus is deeper and the lumbosacral spring test is positive. Range of motion testing reveals left leg adduction to 35°, right leg adduction to 15°. What is the most likely sacral diagnosis? A) Right on right torsion B) Left on right torsion C) Right on left torsion D) Left unilateral sacral flexion E) Right unilateral sacral extension

B The above patient has a left on right torsion. In sacral torsions, the seated flexion test is positive on the opposite side of the axis. In this case, the patient has a positive seated flexion test on the left, which this implies a right oblique axis. The lumbosacral spring test is positive; therefore the sacrum has rotated backward. This will also make the right sulcus deeper. In a right on right torsion, the lumbosacral spring test is negative and the left sulcus is deeper. Answer A In a right on left torsion, the seated flexion test would be positive on the right and the left sulcus is deeper. Answer C In a left unilateral sacral flexion the left sulcus would be deeper, and the lumbosacral spring test would be negative. Answer D In a right unilateral sacral extension, the left sulcus would be deeper. Answer E

A 17-year-old male has a fibular head somatic dysfunction. On examination, the affected ankle tends to plantarflex and invert more than the unaffected ankle and the proximal fibular head resists anterior glide. What is the biomechanical mechanism that resulted in this type of fibular head somatic dysfunction? A) Ankle eversion and external rotation forces the talus to push the lateral malleolus posterior, consequently the proximal fibular head will move anterior due to the natural reciprocal biomechanics of the fibula. B) Ankle supination pulls the distal fibular head anterior, consequently the proximal fibular head will move posterior due to the natural reciprocal biomechanics of the fibula. C) Ankle eversion and external rotation stretches the gastrocnemius/soleus complex thus pulling the proximal fibular head posterior D) Ankle supination stretches the anterior tibialis thus pulling the proximal fibular head anterior. E) Ankle plantar-flexion and inversion stretches the peroneus longus tendon thus pulling the proximal fibular head posterior.

B The above patient has a posterior fibular head somatic dysfunction. This somatic dysfunction is associated with ankle sprains in which the foot is inverted and plantar-flexed and internally rotated. The biomechanical mechanism is: Ankle supination (inversion + plantar-flexion and internal rotation) tightens the anterior talofibular ligament. This taut ligament pulls on the distal fibular head anterior and the natural occurring reciprocal motion of fibula moves the proximal fibular head posterior. The proposed mechanism for an anterior fibular head is: Ankle pronation (eversion, dorsiflexion and external rotation) forces the talus and tissues to push directly against the distal fibular head, thus pushing it posterior. Consequently the proximal fibular head will move anterior due to the natural reciprocal biomechanics of the fibula. Answer A Ankle eversion and external rotation (components of pronation) will not cause a posterior fibular head dysfunction. Answer C Ankle supination will not cause an anterior fibular head somatic dysfunction. Answer D Although ankle plantar-flexion and inversion stretches the peroneus longus tendon, it is not the proposed mechanism by which a posterior fibular head occurs. Answer E

You are asked to evaluate a 23-year-old male with severe unrelenting low back pain. He is unable to move without excruciating pain and rates the pain 10 out of 10. The pain is localized and there is no numbness or tingling in the lower extremities. On examination, there are no neurological deficits. Straight leg raising test is negative. X-rays are negative for fracture, but demonstrates a straightening of the lumbar lordosis. Structural examination reveals an anterior rotated innominate on the right; a left on left sacral torsion and L2 is FRRSR. Tenderpoints are noted one inch medial to the right ASIS. The paraspinal tissues are edematous, erythematous, and boggy with increased moisture and hypertonic muscles. Given the above information, what osteopathic treatment is best suited for this patient initially to improve his symptoms? A) Muscle energy to the innominates. B) Counterstrain to iliopsoas tender points. C) Muscle energy to sacrum. D) High-velocity low amplitude techniques to the lumbar spine. E) All osteopathic treatment is contraindicated due to the acute nature of his injury.

B The above patient has an acute lumbar strain. The patient has no evidence of neurologic impingement. According to the Foundations of Osteopathic Medicine, second edition, acute strains/sprains are often better treated with indirect techniques to prevent further strain. Counterstrain is an indirect passive treatment that will allow the patient's musculature to relax. The patient will unlikely tolerate the positioning for muscle energy due to his severe pain. Answers A and C HVLA may actually worsen the patient's pain if it is done initially, given the acute nature of his injury. Answer D Osteopathic treatment is not contraindicated in acute lumbar strains. Answer E

A 74-year-old female is complaining of severe left-sided neck pain. The pain is located at the base of the neck and radiates into her left upper extremity. She demonstrates limited neck range of motion especially with extension. Symptoms and radiation of pain are worsened with passive extension and left sidebending of the neck. Flexion of the neck decreases the pain. Neurological examination is significant for decreased sensation from the lateral elbow to the thumb and left biceps weakness. Reflexes are 1+ in the left biceps; all other reflexes in the upper and lower extremities are normal. Structural examination reveals C4 to be flexed and sidebent left. Given the above information, which of the following is the best statement? A) A left posterior-lateral disc herniation at C5 impinging the C5 and C6 nerve roots is the most likely cause of this patient's symptoms. B) Degenerative changes and hypertrophy involving the Joints of Luschka and zygapophyseal joints is the most likely cause of this patient's nerve root compression symptoms. C) AP x-ray of the cervical spine will most likely show joint space and foraminal narrowing. D) The somatic dysfunction at C4 is the most likely cause of this patient's symptoms E) Cervical spinal canal stenosis is the most likely cause of this patient's symptoms.

B The above patient is likely to have cervical stenosis. Cervical stenosis can involve the spinal canal, the intervertebral foramen or both. Since she has decreased sensation at C5 (lateral elbow) and C6 (thumb), weak biceps and decreased biceps reflex, she is most likely to have intervertebral foraminal stenosis and nerve root compression at the C5 and C6 levels. The most common cause of nerve root compression in the neck are degenerative changes involving the joints of Luschka and facet (zygapophyseal) joint hypertrophy. Since this patient's symptoms decreased with neck flexion she may not have a herniated disc. In addition, a herniated disc involving one level usually affects one nerve root not two. Also, the nucleus pulposus tends to desiccate by the age of 50 and it is therefore, it is less likely to cause of her symptoms. Answer A Even though this patient is likely to have joint space and foraminal narrowing, an AP x-ray does not show the intervertebral foramen. Intervertebral foramen are best seen on oblique views. Answer C Somatic dysfunction does not usually cause neurologic deficits. Answer D Cervical spinal canal stenosis will result in lower extremity neurological deficits. Usually the patient will be hyper-reflexic in the lower extremities. Answer E

Which of the following correctly describes counterstrain treatment for the above dysfunction? A) Minimal thoracic flexion, left rotation, right sidebending. B) Minimal thoracic flexion, left rotation, left sidebending. C) Marked thoracic flexion, left rotation, left sidebending. D) Neutral thoracic spine, right rotation, left sidebending. E) Neutral thoracic spine, left rotation, right sidebending

B The anterior tenderpoints for ribs 3-6 are all treated by sidebending and rotating toward the side of the tenderpoint and encouraging slight flexion. As a general rule, rib tenderpoints are treated for 2 minutes as opposed to 90 seconds. Dr. Jones felt that the positioning for rib dysfunctions could be uncomfortable for the patient and an extra 30 seconds were needed for the patient to relax.

A 28-year-old medical student comes to your OMT clinic with complaints of low back and buttock pain. The pain originates from the lumbar spine and radiates into the right groin. Upon inspection the pelvis appears to be shifted to the left. Structural examination reveals L2 and L5 FSRRR . Three anterior tenderpoints are noted: one approximately 1 em lateral to the pubic symphysis on the right superior ramus, one at the right AilS and the other approximately one inch medial to the ASIS on the right. Range of motion of the hip is normal except for right hip extension and internal rotation, which is decreased. The lumbosacral junction springs freely. She has a positive seated and standing flexion tests on the right. Her left ASIS is more superior when compared to the right. Which of the following statements is true? A) This patient has a right sacral torsion on a left oblique axis. B) This patient has a shortened right iliopsoas. C} The tender point near the pubic symphysis identifies a somatic dysfunction at L2. D} The left lateral Sim's position is the treatment position to correct the sacral dysfunction. E} This patient has a posteriorly rotated innominate on the right.

B The iliopsoas is a hip flexor and leg external rotator. Spasm of the iliopsoas will limit hip extension and internal rotation. Additional indicators of a right psoas dysfunction is a pelvis that is shifted to the left, the tender point one inch medial to the right ASIS and a high (L2) lumbar type II somatic dysfunction. Since the lumbosacral junction springs freely, the patient could not have a backward sacral torsion (right on left or left on right). As a matter of fact, there is not enough information in the question to diagnose a torsion. Answer A The tender point at the AIlS correlates to the L2 somatic dysfunction. Answer C The left lateral Sim's position will correct a sacral torsion. There is not enough information in the above question to diagnose a torsion. Answer D There is not enough information to diagnose an innominate dysfunction. In order to differentiate an innominate rotation vs. an innominate shear in the above question, the location of the PSIS's must be given. Answer E

A45-year-old female with new onset left upper extremity weakness, miosis, anhydrosis and ptosis was seen by her primary care physician two weeks ago. Diagnostic work-up revealed a Pancoast tumor in the left upper lobe. The tumor appears to be invading the inferior trunk of the brachial plexus. This is the likely etiology of her upper extremity weakness. Based on the tumor's location in the brachial plexus, what physical exam findings would you expect this patient to have? A) Winging of the scapula B) Weakness with finger abduction C) Weakness with arm abduction D) Weakness with elbow flexion E) Numbness of the lateral aspect of the forearm

B The inferior trunk is formed by the C8 and T1 nerve roots. As a result, this will produce weakness with the intrinsic muscles of the hand (interossi) and deep finger flexors. It will also likely produce numbness in the little finger (C8) and medial elbow (T1). The winging of the scapula is due to injury to the long thoracic nerve, which is formed by the C5 - C7 nerve roots. Answer A The deltoid and supraspinatus is primarily responsible for arm abduction. The supraspinatus (suprascapular nerve) receives innervation from the superior trunk. The deltoid (axillary nerve) receives innervation from posterior cord, specifically from C5 and C6 nerve roots. The axillary nerve does not receive contributions from the inferior portion of the brachial plexus. Answer C Numbness at the lateral aspect of the forearm and elbow flexion (musculocutaneous nerve) would not be affected since it stems from the superior trunk of the brachial plexus. Answers D and E

To confirm your diagnosis you would find an anterior Chapman's point just lateral to the sternum at the interspace of: A) Ribs 1 and 2 B) Ribs 2 and 3 C) Ribs 3 and 4 D) Ribs 4 and 5 E) Ribs 5 and 6

B The patient has hyperthyroidism, and the Chapman's reflex point associated with thyroid dysfunction is found at both interspaces of ribs 2 and 3, just lateral to the sternum. Next to the sternum at the interspace of ribs 1 and 2 is the Chapman's point for the tonsils. Answer A Next to the sternum at the interspace of ribs 3 and 4 is the Chapman's point for the upper lung. Answer C Next to the sternum at the interspace of ribs 4 and 5 is the Chapman's point for the lower lung. Answer D Next to the sternum at the interspace of ribs 5 and 6 on the left is the Chapman's point for stomach acidity, and on the right is the Chapman's point for the liver. Answer E

A patient has a CRI of 8 cycles/min with severely decreased excursion of the flexion and extension mechanism. The technique that will most effectively restore normal amplitude of the CRI is? A) Temporal rocking B) CV4 decompression C) V-spread D) Venous sinus technique E) Frontal Lift

B The purpose of the CV4 decompression technique is to enhance the amplitude of the CRI. This is accomplished by inducing a "still point" then allowing restoration of normal flexion and extension. Temporal rocking can help eliminate restrictions in the temporal bone or nearby areas; it does not have as a significant effect on the amplitude of the CRI. Answer A The V-spread is used to free restrictions of impacted sutures. It is not specifically designed to restore normal amplitude of the CRI. Answer C The goal of the venous sinus technique is to enhance venous drainage through the jugular foramen. Answer D The frontal lift aids in the balance of frontal bone membranous tension. Answer E

When palpating the head for the cranial rhythmic impulse using the vault hold, the index fingers should be in contact with which structure? A) The frontal bone B) The sphenoid bone C) The parietal bone D) The temporal bone E) The occipital bone

B The purpose of the vault hold is to address strains at the SBS. The index fingers contact the greater wings of the sphenoid. Fingers do not contact the frontal bone in a vault hold. Answer A The middle fingers contact the zygomatic processes of the temporal bones. Answer C The ring fingers contact the mastoid processes of the temporal bones. Answer D The little fingers contact the squamous portions of the occiput. Answer E

With the patient in the prone position you passively flex the right knee. He is able to touch his right heel to his right buttock. However, when doing the same to the left leg, his left heel remains 6 inches away with maximal knee flexion and you notice that his left hip flexes slightly with this maneuver. Restriction is most likely present in which of the following muscles A) Sartorius B) Rectus femoris C) Vastus medialis D) Gracilis E) Hamstrings

B The rectus femoris originates from the AIIS and inserts onto the quadriceps tendon. This muscle (along with the quadriceps) extends the knee. In addition, since it does attach above the hip it will also flex the hip to a small extent. To test restriction in this muscle the knee is passively flexed when the patient is in the prone position; if the knee cannot touch the buttocks and the hip starts to flex, the test is positive. This test is called Ely's test. Although the Sartorius muscle flexes the hip, it inserts with the gracilis and semitendinosus via the pes anserine tendon and flexes the knee. Answer A Stretching the vastus medialis will not flex the hip since it does not attach above the hip. Answer C The gracilis muscle adducts and flexes the knee. Answer D The hamstrings flex the knee and extend the hip. Answer E

The rationale for having a positive seated flexion test in a patient with a left on left sacral torsion is: A) Posterior rotation of the right innominate produces right iIiosacral dysfunction B) The anterior glide of the right sacral base produces right sacroilial dysfunction. C) Posterior glide of the left ILA produces left sacroilial dysfunction D) Anterior rotation of the right innominate produces sacroilial dysfunction E) Anterior glide of the left sacral sulcus produces left iIiosacral dysfunction

B The seated flexion test assesses sacral motion on the ilium. In other words, it assesses sacroilial motion. If there is an alteration (dysfunction) of sacral motion on the ilium, then there is sacroilial dysfunction. In a left on left sacral torsion, the right sacral base of the sacrum (sacral sulcus) will move anteriorly. This will produce an alteration of sacral motion in the right SI joint (right SI joint dysfunction). Posterior rotation of the right innominate produces right iIiosacral dysfunction and thus will produce a positive standing flexion test on the right. lIiosacral dysfunction is movement of the innominate on the sacrum. Answer A Posterior glide of the left ILA does not produce left sacroilial dysfunction. Remember the seated flexion test is positive on the opposite side of the oblique axis in a sacral torsion. Answer C Anterior rotation of the right innominate produces right iIiosacral dysfunction and thus will produce a positive standing flexion test on the right. Answer D In a left on left torsion the left sacral sulcus does not glide anteriorly (the right glides anteriorly). Answer E

If treating this patient with HVLA in a supine position, which of the following is correct regarding hand placement and direction of force? A) The thenar eminence is placed under the right transverse process of T5, force is perpendicular to the table B) The thenar eminence is placed under the right transverse process of T4, force is perpendicular to the table C) The thenar eminence is placed under the right transverse process of T3, force is 45° caudad D) The thenar eminence is placed under the right transverse process of T4, force is 45° cephalad E) The thenar eminence is placed under the right transverse process of T5, force is 45° cephalad

B The thenar eminence is placed under the right transverse process of T4, force is directed perpendicular into the table. Using the Kirksville Krunch technique, to reverse all planes of a T4 FRRSR the physician's thenar eminence is placed under the right transverse process of T4 (to induce left rotation of the spine). The downward projection of the spinous process at T4 will induce extension at this joint when HVLA thrust is applied. When treating a flexed somatic dysfunction the thrust is directed at the dysfunctional segment, not the one below. Answer A Treating T3 will not have the greatest effect on the somatic dysfunction at T4. Answer C When treating an extended somatic dysfunction of the thoracic spine, the thrust is directed at the segment below the dysfunctional segment at a 45° angle cephalad.

What would be the typical counterstrain treatment position for the patient's tenderpoint? A. Supine, lower extremity flexed, abducted and internally rotated B. Prone, lower extremity flexed, abducted and externally rotated C. Supine, lower extremity flexed, adducted and externally rotated D. Prone, lower extremity extended, abducted and internally rotated E. Supine, lower extremity extended, adducted and externally rotated

B The typical counterstrain treatment position for the piriformis muscle is with the patient prone, the hip and knee flexed and the thigh abducted and slightly externally rotated

Which of the following is the best answer regarding pelvic biomechanics? A) During ambulation, weight bearing on the left leg will induce sacral rotation around a right oblique axis. B) The stance phase of gait is associated with anterior rotation of the innominate. C) As a person inhales the lumbosacral angle increases D) The sacrum nutates with craniosacral flexion E) A positive seated flexion test on the right will always indicate sacral dysfunction on a left oblique axis

B There are two main phases of gait. 1) The swing phase and 2) the stance phase. The swing phase starts at toe off and terminates at heel strike. The stance phase starts at heel strike and terminates at toe off. In the swing phase the innominate rotates posteriorly around the inferior transverse axis of the sacrum. In the stance phase the innominate rotates anteriorly around the inferior axis of the sacrum. When a person is ambulating the innominates rotate about a transverse axis (as above) while the sacrum rotates about two oblique axes. Weight bearing on the left leg will cause a left oblique axis to be engaged, and the opposite is true for the right leg. Sacral rotation will then occur about these two axes. Answer A As a person inhales the diaphragm pushes downward, decreasing the lumbar lordosis causing the sacral base to move posterior. This motion will decrease the lumbosacral angle. Answer C Nutation and counternutation are terms used for motion at the sacral base associated with craniosacral motion. Flexion of the cranial bones will cause the sacral base to extend (move posterior). This is called counternutation. Extension of the cranial bones will cause the sacral base to flex (move anterior). This is called nutation. Answer D Sacral dysfunction on a left oblique axis will yield a positive seated flexion test on the right, however other sacral dysfunctions are also seen with a positive seated flexion test on the right (e.g. right unilateral sacral flexion or extension). Therefore a positive seated flexion test on the right will not always indicate a sacral dysfunction on a left oblique axis. Answer E

A football player gets hit on the side of his right hip. The patient complains of right hip pain. Structural examination reveals no sacral dysfunction. The left ASIS is 6 inches from the midline, while the right ASIS is 5 inches from the midline. The left ASIS moves easily with posterior compression as compared to the right. What is the most likely diagnosis? A) A left anterior innominate rotation B) A right innominate inflare C) A left innominate outflare D) A right anterior innominate rotation E) A left superior innominate shear

B This patient has a right inflared innominate caused by the hit to the lateral side of the hip. The laterality of the dysfunction was determined by the ASIS compression test. A positive test occurs if one ASIS resists posterior compression, indicating the side of the dysfunction. The static findings indicate that the distance from the right ASIS to the umbilicus is shorter than the left. This indicates an innominate inflare. Left anterior innominate rotation, a left innominate outflare and a left superior innominate shear would have a positive ASIS compression test on the left, not the right. Answers A, C, and E A right anterior innominate rotation would have a positive ASIS compression test on the right. The static findings would show that the ASIS on the right is inferior and the right PSIS is higher on the right compared to the left. The distance from the ASIS to the umbilicus would be equal. Answer D

53-year-old male complains of chronic pyrosis and regurgitation after meals for several years. The patient has been taking antacids and over-the-counter famotidine, but they are no longer relieving his symptoms. The patient admits to drinking five to six beers and four cups of coffee a day, and smoking half a pack of cigarettes a day. These symptoms are typical of: A) Dyspepsia B) GERD C) Peptic ulcer disease D) Zollinger-Ellison syndrome E) Mallory Weiss syndrome

B This patient has gastroesophageal reflux disease. The typical symptom is heartburn; in this case the disease is refractory to antacids and famotidine. Reclining after eating, chocolate, coffee, caffeine, cigarettes, fatty meals, peppermint, and alcohol worsen the condition. Dyspepsia is defined as pain or discomfort centered in the upper abdomen. Heartburn (retrosternal burning) should be distinguished from dyspepsia. Patients with dyspepsia often have heartburn as an additional complaint. When heartburn (pyrosis) coupled with regurgitation is the dominant complaint, gastroesophageal reflux disease is nearly always present and should be distinguished from dyspepsia. Answer A Epigastric pain (dyspepsia) is the hallmark of peptic ulcer disease, which is usually relieved by eating or antacids, with a recurrence of pain two to four hours later. Answer C Zollinger-Ellison syndrome is caused by gastrin-secreting neuroendocrine tumors (gastrinomas), which result in hypergastrinemia and acid secretion. Over 90% of patients with this syndrome develop peptic ulcers. In most cases, the two conditions are indistinguishable and present as peptic ulcer disease. Answer D Mallory-Weiss syndrome is characterized by a non-penetrating mucosal tear at the gastroesophageal junction, which is associated with hematemesis. Answer E.

A 34-year-old female complains of dyspnea, fatigue, weakness, weight loss, and increased appetite. On physical exam you note a goiter, warm sweaty skin, and hand tremor. You would expect to find tissue texture changes due to a viscerosomatic reflex at which spinal level? A) C3-C5 B) T1-T4 C) T5-T9 D) T9-T12 E) L1-L2

B This patient has signs and symptoms consistent with hyperthyroidism. T1-T4 receives sympathetic innervation from the thyroid. Although C3-C5 may have tissue texture changes from local effects, it is less likely to be involved in a viscerosomatic reflex since the sympathetic nervous system stems from T1 - L2. Answer A T5-T9 receives sympathetic innervation from the upper GI tract. Answer C T9-T12 and L1 - L2 receive sympathetic innervation from below the diaphragm. Answers D and E

A 17-year-old lacrosse player comes to the athletic trainer's office with low back pain. As the team physician he asks to see you. The athlete states that the pain started several weeks ago, it is localized to the lumbosacral spine and worsens with backward bending. Pain was mild at first, now it is severe enough that he can no longer play. On examination, there is tenderness of the lumbar paraspinals. The spinous processes of the lumbar vertebrae do not appear prominent unlike the L5 vertebra and sacrum. Examination of the lower extremities reveals normal muscle strength and sensation. On structural examination of the lumbo-sacral region, there is a positive seated flexion test on the right, a deep sacral sulcus on the left, and L5 is rotated to the left. The L5 findings are exaggerated upon backward bending, symmetry is restored with forward flexion. The sacrum does not spring at the lumbosacral junction. The most likely etiology of this patient's acute low back pain is? A) Herniated nucleus pulposus B) Spondylolisthesis C) Osteoarthritis of the lumbar spine. D) Spinal stenosis E) Facet tropism

B This patient most likely has spondylolisthesis. This is not an uncommon cause of back pain for young athletes; especially those athletes that are involved with extension based athletic activities, such as lacrosse players. Pain is usually localized to the lumb-sacral spine. Neurological examination is normal. There may be a positive "step-off sign" (upon palpating the spinous processes there is an obvious forward displacement at the area of the Iisthesis). In this case the spinous process of L5 is prominent, whereas the spinous processes of remainder of the lumbar spine are not. Since the neurologic examination is normal, this patient is not likely to have a herniated disc. Answer A Osteoarthritis of the lumbar spine and spinal stenosis is uncommon for a 17 year old. Typically pain arising from these conditions start after the 5th decade, unless there is a predisposing factor such as ankylosing spondylitis. Answers C and D Although facet tropism is the most common anatomic anomaly of the lumbar spine and can lead to degenerative changes in the lumbar spine, alone it is not the primary cause of low back pain in the above patient. Answer E

78-year-old male underwent abdominal aortic aneurysm repair with graft yesterday. He is currently in the intensive care unit. The operation was successful and postoperatively the patient is doing well. Physical exam reveals: Patient appears to be in no acute distress Vital Signs: Temperature 99.5, Heart rate 72, Respiration 16 and shallow, Blood pressure 120/60, mild incision pain approximately 4/10 controlled with PCA pump. Heart: Regular rate and rhythm. No murmurs, gallops or rubs. Lungs: Decreased respiratory excursion throughout the lung fields. Few scattered bibasilar crackles. Abdomen: Hypoactive bowel sounds, incision clean and dry with no evidence of erythema. Extremities: Strong peripheral pulses. There is trace edema in both lower extremities What is the most likely etiology of the bibasilar crackles in this patient? A) Pulmonary embolism B) Postoperative atelectasis C) Congestive heart failure D) Pneumonia E) Pericardial effusion

B This patient recently underwent major abdominal surgery. It is common for a postoperative patient to have limited chest wall motion. This typically leads to atelectasis. Atelectasis is the most common etiology of bibasilar crackles in any post-surgical patient. A pulmonary embolus is an acute event that will result in respiratory distress due to hypoxia. This patient does not have a pulmonary embolus. Answer A Although congestive heart failure could lead to bibasilar crackles, the patient has a normal respiratory rate and is not complaining of dyspnea or orthopnea. In addition, typically a patient in CHF will have an S3 gallop on auscultation of the heart. Answer C Atelectasis could lead to pneumonia, however this would be more likely if the patient had a fever or cough. Answer D This patient is not likely to have a pericardial effusion since the surgery was local to the abdominal cavity. Answer E

A 30-year-old athletic male presents to your office complaining of low back pain. He states that the pain started soon after playing baseball. The pain is localized to his low back. He denies any radiation of pain, paresthesias or weakness in the lower extremities. There is no previous history of low back pain. Physical exam reveals no focal neurologic deficits. X-rays of the lumbar spine are normal. There is marked spasm of the paraspinal musculature on the right. There appears to be a group dysfunction from L1-L5 that is markedly rotated right and sidebent left. There is a positive seated flexion test on the right, and the right sulcus is deeper. Which of the following is the most likely diagnosis? A) Lumbar spinal stenosis B) Cauda equina syndrome C) Acute lumbar strain D) Herniated nucleus pulposus E) Spondylolisthesis

C An acute lumbar strain is low back pain without focal neurological deficits. The pain may be sharp or dull, and usually localized to the low bacK, although it may radiate in a nondermatomal type of fashion. Spinal stenosis is a chronic joint disease characterized by slowly developing joint pain, usually resulting from intervertebral narrowing, foraminal encroachment and/or spinal canal compression. Often this occurs in elderly patients and the neurologic examination will reveal some deficits. Answer A Cauda equina syndrome is compression of the distal spinal cord. This will result in paresthesias and weakness as well as incontinence. Answer B A herniated nucleus pulposus (herniated disc) will often result in neurologic deficits therefore making this a less likely diagnosis. Answer D Spondylolisthesis is a forward slippage of the lumbar vertebrae. Radiographic findings in this patient are normal making this an unlikely diagnosis. Answer E

A patient tripped and fell backward on an outstretched right hand. The patient complains of pain on the lateral side of his right elbow. The physical exam reveals the right forearm prefers supination and the radial head is tender to palpation and restricted in posterior glide. The most likely diagnosis is: A) Lateral epicondylitis B) Posterior radial head C) Anterior radial head D) Adduction dysfunction of the ulna E) Abduction dysfunction of the ulna

C An anterior radial head dysfunction would cause the forearm to prefer supination and the radial head would resist posterior glide. Falling backward on a supinated forearm can cause this dysfunction. Lateral epicondylitis is a strain of the extensor muscles of the forearm near the lateral epicondyle. It is usually due to overuse of the wrist extensors and not typically associated with trauma. Answer A A posterior radial head dysfunction would cause the forearm to prefer pronation, and the radial head would resist anterior glide. Falling on a pronated forearm causes this dysfunction. Answer B An adducted dysfunction of the ulna would have a decreased carrying angle, the hand and wrist would be adducted, and the olecranon process would prefer lateral glide. Answer D An abducted ulna causes a decreased carrying angle of the arm; the olecranon process would be restricted in medial glide, and the wrist and hand would be abducted. Answer E

In a patient with a postero-Lateral disc herniation at the L3-L4 level, weakness is most likely to be present in which of the following muscles? A} Extensor hallucis longus B} Flexor hallucis longus C} Anterior tibialis D} Hamstrings E} Peroneus longus

C Anterior tibialis. The above patient has a disc herniation at the L3/L4 level. This will most likely impinge the L4 nerve root. The innervation of the anterior tibialis is from the L4 and L5 nerve roots (deep peroneal nerve). The innervation of the extensor hallucis longus is from the L5 and S1 nerve roots (deep peroneal nerve). Answer A The innervation of the flexor hallucis longus is from the S2 and S3 nerve roots (tibial nerve). Answer B The innervation of the hamstrings is from the L5 - S2 nerve roots (tibial division of sciatic nerve and peroneal division of the sciatic nerve - short head of biceps). Answer D The innervation of the peroneus longus is from the L5 - S2 nerve roots (superficial peroneal nerve). Answer E

What is the most likely composition of the mass in the KUB? A) Struvite B) Uric acid C) Calcium oxalate D) Cystine E) Bile pigment

C Approximately 80% of all renal stones are calcium oxalate and can be visualized on KUB x-ray, 5% are uric acid and 1% is cystine, the remainders are magnesium, ammonium phosphate or calcium phosphate. Also, uric acid and cystine stones are not easily visualized on a KUB.

A 62-year-old female has neck pain. On examination she has limited rotation of the neck. She is most likely to have a dysfunction at which structure? A) The sphenoid B) The occiput C} The atlas D) The axis E} C3

C As a general rule, when referring to segmental motion (or restriction) it is traditional to refer to motion (or restriction) of the segment above in a functional vertebral unit (two vertebrae). For example, when describing the motion (or restriction) of C1, it is the motion of C1 on top of C2. As the neck rotates, most of the rotation comes from the atlas rotating around the dens of the axis (Le. the AA joint - C1 on C2). So therefore, restriction with neck rotation will most likely stem from the atlas. The sphenoid bone is not directly related to rotation restriction of the neck. Answer A Due to the orientation of the occipital condyles on the atlas, this joint (the occipito-atlantal joint) typically prefers flexion and extension. Answer B The axis (C2) and C3 do have a fair amount of rotation, however most of the rotation of the neck comes from the atlas. Answers D and E

The most likely etiology of the pain is: A) Cervical rib B) Ischemic myocardium C) Hypertrophied scalene muscles D) Herniated nucleus pulposus E) Shoulder dislocation

C Based on the history, the most likely etiology is hypertrophied scalene muscles. One of the areas that causes neurovascular compression of the upper extremity is between the anterior and middle scalene. Chronic COPD can result in hypertrophy or increased tension of the scalene muscles. Although cervical ribs can cause thoracic outlet syndrome, it would have been reported on the cervical spine x-ray. Answer A Ischemic myocardium is associated with a myocardial infarction, not with thoracic outlet syndrome. Answer B Herniated nucleus pulposus is not an etiology of thoracic outlet syndrome, but is the cause of cervical radiculopathy. Answer D Shoulder dislocation is not an etiology of thoracic outlet syndrome and was ruled out by the shoulder xray report. Answer E

If the patient had reported decreased sensation on the medial aspect of the ankle, absent patellar reflex and marked weakness of ankle dorsiflexion and knee extension, what would the most likely diagnosis be? A) Spondylolisthesis B) Lumbar spinal stenosis C) Herniated nucleus pulposus D) Acute lumbar strain E) None of the above are likely diagnoses

C Herniated nucleus pulposus. A herniated disc at L4 will result in the decreased sensation, the decreased deep tendon reflexes and lower extremity weakness described. Radiographic findings on the patient were normal making spondylolisthesis unlikely. Answer A Spinal stenosis is usually due to intervertebral narrowing and degenerative joint disease. Although neurological deficits are common, spinal stenosis is usually seen in the elderly population. Answer B Neurologic exam is negative in acute lumbar strains. Answer D

Treatment of the anterior Chapman reflex point for this patient's hyperacidity would be directed just lateral to the sternum at the interspace of: A) Ribs 4 and 5 on the left side B) Ribs 5 and 6 on the right side C) Ribs 5 and 6 on the left side D) Ribs 6 and 7 on the right side E) Ribs 6 and 7 on the left side

C Chapman's reflex point for hyperacidity is located just lateral of the sternum at the interspace of ribs 5 and 6 on the left side. Overall, this is a very difficult question. Just lateral of the sternum at the interspace of ribs 4 and 5 on the left side is the Chapman's reflex point for the left lower lung. Answer A Just lateral to the sternum at the interspace of ribs 5 and 6 on the right side is the Chapman's reflex point for the liver. Answer B Just lateral to the sternum at the interspace of ribs 6 and 7 on the right side is the Chapman's reflex point for the liver and gall bladder. Answer D Just lateral to the sternum at the interspace of ribs 6 and 7 on the left side is the Chapman's reflex point for peristalsis of the stomach. Answer E

Craniosacral flexion is associated with: A) Counternutation of the sacrum around a middle transverse axis B) Caudad motion of the spinal dura C) External rotation of the frontal bone D) Increased AP diameter of the cranium E) Nausea

C Craniosacral flexion is associated with flexion of the midline bones and external rotation of the paired bones. The frontal bone is considered a paired bone because of the metopic suture. Craniosacral flexion is associated with counternutation or sacral extension about the superior transverse axis. Answer A Craniosacral extension is associated with caudad motion of the dura. Answer B Increased AP diameter of the cranium is associated with craniosacral extension. Answer D Physiologic motion of the cranium is not associated with nausea. Answer E

Which of the following biomechanical or structural abnormalities best explain why a patient with COPD has an increased AP diameter of his/her chest? A) Fibrotic changes within the diaphragm due to chronic hypoxia results in a flattened diaphragm and an increased total lung capacity. B) Continuous accessory muscle use will pull the ribs superiorly, resulting in an expanded thoracic cage. C) The destruction of elastic fibers and chronic air trapping results in permanent inhaled rib positions. D) Chronic deoxygenation of peripheral tissues results in a physiologic expansion of the thoracic cage as a compensatory mechanism. E) Exaggerated sympathetic input from visceral afferents causes broncho-aveolar expansion and results in an increased AP diameter.

C Destruction of elastic fibers results from years of smoking. (an emphysema type of COPD). Irritation of the lung parenchyma may result in an overproduction of mucus causing obstruction and air trapping (chronic bronchitis and asthma type of COPD). These two factors expand the chest cavity and can result In an Increased lung capacity, thus permanently positioning the ribs in inhalation. Patients with COPD will have impaired oxygenation, however it will not result in a fibrosis and flattening of the dIaphragm. If this were true all skeletal muscles in the body would be fibrosed. Answer A Most accessory muscles will help elevate the ribs into inhalation, however these muscles are not strong enough to overcome the natural recoil of the lungs. Answer B Chronic deoxygenation will result in a change in minute ventilation, rather than a permanent expansion of the thoracic cage. Answer D Increased sympathetic tone from visceral afferents results in bronchodilation and consequently bronchoavelolar expansion. However, this is not the primary reason for an increased AP diameter in a person with COPD. Answer E

Which of the following is the most serious neurologic complication that has been associated with shoulder dystocia? A) Amblyopia B) Klumpke's palsy C) Erb-Duchenne's palsy D) Bell's palsy E) Cerebral palsy

C Erb-Duchenne's palsy is often due to excessive traction placed on the head and neck while the shoulder is depressed. This can occur when the fetus' shoulders are having difficulty exiting the pelvis. Shoulder dystocia is not associated with amblyopia. Answer A Klumpke's palsy is any injury to the lower brachial plexus. It can be due to extreme abduction and extension of the shoulder. It is not associated with shoulder dystocia. Answer B Bell's palsy results in an injury to the facial nerve and is not associated with childbirth. Answer D Cerebral palsy is primarily a disorder of movement and posture that is due to a non-progressive lesion to an immature brain. It is not associated with shoulder dystocia. Answer E

Which one of the following statements is true? A) Abduction of the ulna is associated with an increased carrying angle and restricted adduction of the wrist B) Cubitus varus is often seen in Turners syndrome. C) Falling forward on an outstretched pronated forearm could result in a radial head somatic dysfunction that is associated with restricted forearm supination. D) A complication of a scaphoid wrist fracture is avascular necrosis of the distal fragment E) Wrist flexion is associated with ventral glide of the carpal bones

C Falling forward on an outstretched pronated forearm is a common cause of a posterior radial head somatic dysfunction. This somatic dysfunction is associated with restricted supination of the forearm. Abduction of the ulna is associated with an increased carrying angle and restricted abduction of the wrist. Answer A Cubitus valgus is sometimes seen in Turners syndrome. Answer B The vascular arrangement of the scaphoid is that the main vessel enters the distal pole and runs through to the proximal pole. Thus, proximal pole fractures of the scaphoid have a high incidence of avascular necrosis of proximal segment. Answer D Wrist flexion is associated with dorsal glide of the carpal bones. Conversely, wrist extension is associated with ventral glide of the carpal bones. Answer E

Nerve entrapment at Guyon's canal is likely to produce A) Wrist adduction weakness B) Wrist flexion weakness C) Finger abduction weakness D) Thumb abductor weakness E) Weakness with flexion of the 4th and 5th DIP's

C Guyon's (pisohamate) canal is located at the wrist. The medial border is the pisiform bone, lateral border is the hamate, roof is the flexor retinaculum and floor is the pisohamate ligament. The ulnar nerve runs through the Guyon's canal. Nerve entrapment would produce weakness in the hypothenar muscles, adductor pollicis, interossei, and 3rd and 4th lumbricals. The interossei are responsible for abduction and adduction of the fingers. Wrist adductors and wrist flexors originate in the forearm and would not be affected with entrapment at the wrist. Answer A and B The median nerve innervates the abductor pollicis brevis. The radial nerve innervates the abductor pollicis longus. Answer D Weakness with flexion of the 4th and 5th DIP's (ulnar portion of flexor digitorum profundus) would be due to injury to the ulnar nerve proximal to the wrist (most likely at the cubital tunnel). Answer E

In a 41-year-old male with back pain, you observe that thoracic segments T8 - T12 have a convexity to the right. Which of the following is the best statement regarding this group somatic dysfunction? A) T9's right transverse process is more anterior B) T12 is limited in right rotation compared to L1 C) T10's left transverse process is more caudad compared to its right D) Ribs 7 - 12 are limited in exhalation on the left E) T9 is rotated right, sidebent right

C If T8 - T12 have a convexity to the right, there is a group dysfunction that is sidebent left and rotated right. Remember group dysfunctions occur in the neutral plane and in such a case they sidebend and rotate to opposite sides. If T1 0 is sidebent left, then its left transverse process will be more caudad than its right transverse process. If T9 is rotated to the right, then its right transverse process would be more posterior. Answer A If T12 is rotated right, it will be limited in left rotation. Answer B A thoracic dysfunction does not always preclude a rib dysfunction. There is not sufficient information given to determine if a rib dysfunction is present. Answer D The question is describing a neutral dysfunction that is rotated right, sidebent left. Therefore T9 cannot be rotated right and sidebent right. Answer E

In a patient with chest wall tenderness, rib 5 has limited inhalation motion around an anteriorposterior axis. The best statement that describes the somatic dysfunction of rib 5 is? A) The shaft of rib 5 will approximate the shaft of rib 4 at the mid-clavicular line. B) Rib 5 will feel more prominent anteriorly. C) The shaft of rib 5 will approximate the shaft of rib 6 in the mid-axillary line D) Rib 5 has a pump-handle exhalation dysfunction E) The angle of rib 5 will feel more prominent

C If rib 5 has limited inhalation motion, there is an exhalation dysfunction present. Bucket handle motion occurs around an anterior-posterior axis. Even though rib 5 moves primarily in pumphandle fashion; there is still some bucket-handle motion (for a further explanation of bucket-handle dysfunctions of the upper ribs see Greenman's Principles of Manual Medicine). In a bucket-handle exhalation dysfunction the rib shaft (of rib 5) will approximate the shaft of the rib below (rib 6) in the midaxillary line. The shaft of rib 5 will approximate the shaft of rib 4 at the mid-clavicular line in a pump-handle inhalation dysfunction. Answer A Rib 5 will feel more prominent anteriorly in a pump-handle inhalation dysfunction. Answer B Rib 5 has a bucket-handle exhalation dysfunction. Pump-handle movement is around a transverse axis. Answer D The rib angle will not feel more prominent in bucket-handle dysfunctions. Rib angle changes are usually reserved for pump-handle dysfunctions. Answer E

Further examination of the patient reveals that ribs 2 - 6 lag behind with inhalation. What muscle would be used to treat this dysfunction if you used a muscle energy treatment? A) Anterior scalene B) Middle scalene C) Posterior scalene D) Pectoralis minor E) Serratus anterior

C In a group exhalation dysfunction, the key rib is the uppermost rib; treatment is directed at this key rib. The posterior scalene muscle attaches to the second rib, and is used for a muscle energy treatment of an exhalation dysfunction. The anterior and middle scalenes are used to treat an exhalation dysfunction of the 1st rib. Answers A and B Pectoralis minor is used to treat an exhalation dysfunction of ribs 3-5. Answer D Serratus anterior is used to treat an exhalation dysfunction of ribs 6-9. Answer E

In a patient with acute onset of low back pain, structural examination reveals a positive seated flexion test on the right. The left sacral base is anterior when compared to the right. The spring test is positive. Given the above information what is the most likely sacral diagnosis? A) Left sacral rotation on a right oblique axis B) Right sacral rotation on a right oblique axis C) Right sacral rotation on a left oblique axis D) Right unilateral sacral flexion E) Extended sacral base

C In a sacral torsion (or sacral rotation on an oblique axis) the seated flexion test is positive on the opposite side of the axis. In this case the right positive seated flexion test indicates a left oblique axis. A positive spring test (a.k.a. lumbosacral spring test) indicates that part of the sacral base has moved posterior. Since the left sacral base is anterior (i.e. the left sulcus is deep), then this must indicate that the right sacral base has moved posterior. Out of all the answers listed the only possible one that would correlate with the above findings is right sacral rotation on a left oblique axis. In left sacral rotation on a right oblique axis, and right sacral rotation on a right oblique axis there would be a positive seated flexion test on the left. Answers A and B In a right unilateral sacral flexion the right sulcus would be deeper, and the lumbosacral junction would spring freely. Answer D In an extended sacral base the seated flexion test would be falsely negative, and the sulci would appear symmetric. Answer E

If a right-sided psoas syndrome is suspected in this patient, which lumbar dysfunction is likely to be present? A) L1 Neutral sidebent left and rotated right B) L2 Extended, sidebent left and rotated left C) L1 Flexed, sidebent right and rotated right D) L2 Flexed, sidebent left and rotated left E) L4 Neutral sidebent right rotated left.

C In psoas syndrome a high lumbar dysfunction is likely to be present. Specifically the lumbar dysfunction will be flexed, sidebent and rotated to the side of the tight psoas. In this case, L1 would be flexed, rotated right and sidebent right.

If the above patient had a sacral torsion present, based on the above findings what is the most likely diagnosis A) Left torsion on Right oblique axis B) Right torsion on Left oblique axis C) Left torsion on Left oblique axis D) Right torsion on Right oblique axis E) Cannot determine based on the limited information

C In sacral torsions, specific L5 findings are present. L5 will influence the sacrum in the following ways: if L5 is rotated, the sacrum will rotate in the opposite way on an oblique axis. If L5 is sidebent, the sacral oblique axis is engaged on the same side. Since the patient's L5 was NSlRr, the sacrum is rotated left on a left oblique axis.

The increased sympathetic tone would cause which of the following in a COPD patient? A) Thinning of secretions B) Bronchoconstriction C) Thickening of secretions D) Decreased tissue congestion E) Vasodilatation of the pulmonary vasculature

C Increased sympathetic tone in a COPO patient will cause a thickening of pulmonary secretions via increased goblet cell production. Thinning of secretions is due to an increase in parasympathetic tone or a decrease in sympathetic tone. Answer A Increased sympathetic tone will also result in bronchodilation, not bronchoconstriction. Answer B Pulmonary vasoconstriction from increased hypersympathetic tone could result in increased tissue congestion and possibly development of pneumonia. Answer D Increased sympathetic tone will also result in vasoconstriction, not vasodilation of the pulmonary vasculature. Answer E

Plexopathy involving the medial cord of the brachial plexus will have the greatest effect on which of the following movements? A) Elbow flexion B) Elbow extension C) Finger abduction D) Shoulder abduction E) Shoulder external rotation

C Injury to the medial cord of the brachial plexus is likely to affect C8, T1 and muscles innervated by the median and ulnar nerves. The interossei muscles of the hand are responsible for finger abduction. The ulnar nerve innervates the interossei. The musculocutaneous nerve is responsible for elbow flexion. This nerve originates from the lateral cord. Answer A The radial nerve is responsible for elbow extension. This nerve originates from the posterior cord. Answer B The axillary nerve is primarily responsible for shoulder abduction. This nerve originates from the posterior cord. Answer D The teres major (axillary nerve) and infraspinatus (suprascapular nerve) are responsible for shoulder external rotation. These nerves originate from the posterior and lateral cords respectively. Answer E

A male complains of right foot pain for several weeks. He remembers having the pain before. The pain is located between the third and fourth metatarsal heads. The pain is described as 8/10 in intensity, and burning in nature. What is the most likely diagnosis? A) Hammer toes B) Claw toes C) Morton's neuroma D) Bunion E) Corns

C Morton's neuroma is a fibroneuromatous reaction between the heads of the third and fourth metatarsals. Pain is present at the forefoot at the site of the neuroma it is often accompanied with dysesthesia or burning plantar pain. A hammer toe is a condition in which the proximal interphalangeal (PIP) joint is hyperflexed. Typically there is an obvious deformity and the patient may have pain at the PIP when wearing shoes. Answer A Claw toe is a fixed flexion deformity of the PIP joints associated with hyperextension of the metatarsophalangeal articulations. All toes, especially the lesser toes, tend to be effected. Answer B A bunion, also called hallux valgus, is a lateral deviation of the proximal phalanx of the first toe associated with soft tissue changes, pain, swelling, and inflammation at the aspect of the head of the first metatarsal, which is angled medially. Answer D Soft corns are hyperkeratotic lesions found between the toes, usually the fourth and fifth toes; they are extremely painful. Hard corns are associated with hammer or claw toes. Answer E

A patient with dysmenorrhea would like osteopathic treatment for relief of her pain. Which of the following statements concerning the autonomic nervous system effects on the uterus are true? A. Parasympathetic stimulation results in constriction of the uterine fundus. B. Parasympathetic stimulation results in relaxation of the cervix. C. Parasympathetic stimulation results in relaxation of the uterine fundus. D. Sympathetic stimulation results in relaxation of the uterine fundus. E. Sympathetic stimulation results in constriction of the cervix.

C Parasympathetic stimulation of the uterus causes relaxation of the fundus and constriction of the cervix. Answers A and B Sympathetic stimulation of the uterus causes constriction of the fundus and relaxation of the cervix. Answers D, and E

If a muscle energy technique is used to treat this dysfunction, which muscle should be recruited? A) Posterior scalene B) Teres major C) Pectoralis minor D) Pectoralis major E) Serratus anterior

C Ribs 3-5 are held down in exhalation. The pectoralis minor muscle originates from the coracoid process and inserts on ribs 3-5. Activation of this muscle with muscle energy would pull the shaft of the ribs superior, correcting the dysfunction. The posterior scalene is used to treat a somatic dysfunction of rib 2. Answer A The teres major and pectoralis major are not muscles used to treat rib dysfunctions. Answers B and D The serratus anterior is used to treat a somatic dysfunction of ribs 6-9. Answer E

With deep inspiration, ribs 6-10 increase the chest wall diameter A) Anteriorly along an oblique plane B) Anteriorly around a transverse axis C) Laterally around an anterior-posterior axis D) Laterally around a transverse axis E) Laterally around a vertical axis

C Ribs 6-10 primarily move in a bucket-handle motion. Movement is around an AP axis, therefore, inhalation will increase the transverse diameter of the chest wall. Physiologic rib motion does not occur along an oblique plane. Answer A Pump-handle ribs will increase the chest wall anteriorly around a transverse axis. Answer B Rib movement around a transverse axis could not expand the ribcage laterally. Answer D Ribs do not move around a vertical axis. Answer E

Occipital or temporal restriction is least likely to affect which of the following? A) Tongue control in the newborn B) Peristalsis in the lower 2/3 of the esophagus C) Sense of smell D) Venous drainage of the cranium E) Visual accommodation

C Sense of smell is controlled by the olfactory nerve (CN I). An altered sense of smell has been associated with sphenoid, frontal and ethmoid dysfunction. Occipital or temporal restrictions can alter many physiologic functions. Poor tongue control and poor suckling in the newborn has been attributed to occipital condylar compression. Answer A Occipital or temporal bone dysfunction, especially at the occipito-temporal suture at the jugular foramen can effect cranial nerves IX, X and XI. The vagus nerve controls peristalsis of the lower 2/3 of the esophagus. Answer B The internal jugular vein drains about 85% of the blood from the cranium. Restriction of the occiput or temporal bones at or near the jugular foramen could affect venous drainage from the cranium. Answer D Temporal bone restriction has been associated with cranial nerve III dysfunction. Preganglionic parasympathetic fibers of cranial nerve III arise from the Edinger-Westphal nucleus (in the midbrain) and project to the ciliary muscle, which will control visual accommodation. Answer E

What is the accepted muscle energy treatment patient position for the above sacral diagnosis? A) Right lateral Sims position B) Right lateral recumbent with the torso turned left C) Left lateral Sims position D) Left lateral recumbent with the torso turned right E) Patient prone with right lower extremity slightly abducted

C The patient in the above question has a left on left sacral torsion. The accepted muscle energy treatment position of the patient is the left lateral Sims position. In this position the patient is lying in the left lateral recumbent position and the torso is turned to the left so the patient is lying face down. The right lateral Sims position would be used to treat a right on right sacral torsion. Answer A The right lateral recumbent position with the torso turned left would be used to treat a left on right sacral torsion. Answer B The left lateral recumbent position with the torso turned right would be used to treat a right on left sacral torsion. Answer D This position could be used to treat a sacral shear. Answer E

Which of the following is true regarding the anatomy and biomechanics of the thoracic region? A) When a patient is standing with their arms at their side, the medial border of the spine of the scapula is immediately lateral to the spinous process of T1. B) The spinous processes of the thoracic region point increasing downward, such that T1 's spinous process is approximately at the level of the transverse process of T2. C) Restriction within Sibson's fascia is likely to limit lymphatic drainage from the left lower extremity. D) The main motion of the thoracic cage is about a transverse axis. E) At quiet breathing the diaphragm bears 80 - 90% of the workload

C Sibson's fascia forms the functional cervico-thoracic diaphragm (thoracic inlet) and is especially involved in the mechanics of fluid homeostasis in the entire body. The thoracic duct, which drains the left upper extremity and bilateral lower extremities, passes through the thoracic inlet. Therefore restriction within the thoracic inlet is likely to limit lymphatic drainage of the left lower extremity. When a patient is standing with their arms at their side, the medial spine of the scapula is immediately lateral to the spinous process of T3, not T1 . Answer A Although the spinous processes point increasingly downward, the rule of three's dictates that the spinous process of T1 is located equal to the transverse processes of T1. Answer B The main motion of the thoracic spine is rotation. Rotation moves about a vertical axis. Answer D At quiet breathing the diaphragm bears 100% of the workload. During exercise the diaphragm bears 60% of the workload; and the secondary muscles of respiration are responsible for 40% of the workload during exercise. Answer E

The x-ray finding in the above patient is most consistent with? A) Anterior slippage of the L5 vertebrae on the sacrum B) A defect in the pars interarticularis C) Degenerative changes and osteophytic lipping D) Compression fracture of L5 E) Ankylosing spondylitis

C Spondylolysis is a radiological term for degenerative changes in the intervertebral discs and anklyosing of the adjacent vertebral bodies. Anterior slippage of the L5 vertebrae on the sacrum is spondylolisthesis, not spondylosis. Answer A A defect in the pars interarticularis is spondylolysis, not spondylosis. Answer B Spondylosis is not associated with compression fractures. Answer D Ankylosing spondylitis is a systemic disease that results in ankylosing of several vertebrae starting at the lumbosacral spine and proceeding cephalad. Although spondylosis can include ankylosing of adjacent vertebral bodies, a 53-year-old male with ankylosing spondylitis would have several joints involved, not just at L5/S1. Answer E

Which sacral region in the above dysfunction will spring freely? A) Left superior sulcus B) Right superior sulcus C) Left inferior lateral angle D) Right inferior lateral angle E) Sacral base

C Springing is present over the part of the sacrum that has freedom of motion anteriorly. The left inferior lateral angle moves anterior (as the right sacral base moves posterior) and thus will spring freely in a right on left sacral torsion. The left superior sulcus and the right inferior lateral angle will not spring freely because they are part of the left oblique axis of the torsion. As a general rule, motion (springing) over the parts of the oblique axis is restricted. Answers A and D The right sulcus is rotated posteriorly. Springing is restricted over the part of the sacrum that has moved posterior. Answer B In order to perform the lumbosacral spring test, the physician will spring at the lumbosacral junction (this includes the sacral base). The lumbosacral spring test is positive (meaning it does not spring) in a backward sacral torsion. Answer E

Which osteopathic treatment is indicated to treat this newborns suckling problem? A) Rib raising B) CV4 technique C) Condylar decompression D) Sphenopalatine ganglion release E) Temporal rocking

C Suckling dysfunction can be associated with condylar compression. The treatment of choice is condylar decompression.

Which of the following medications is most appropriate for initial treatment of this patient's headache? A) Butalbital B) Valproic acid C) Sumatriptan D) Propranolol E) Atenolol

C Sumatriptan (Imatrex) can be used to treat acute migranes Tension headaches are typically treated with butalbital in combination with acetominophen. Butalbital is a barbituate. Answer A Valproic acid (depakote) atenolol (tenormin) and propranolol (Inderal) can be used for migrane prophylaxis. Answers B, D and E

A 35-year-old male is in the intensive care unit after a recent motorcycle accident. He fractured C6, C7 and his left tibia. As a result, he has a spinal cord injury. He underwent surgical spine stabilization, but is awaiting surgical stabilization for his tibia fracture. Which one of the following osteopathic treatments is contraindicated in this individual? A) Thoraco-abdominal diaphragm release B) Thoracic rib raising and paraspinal inhibition C) Pedal lymphatic pump D) Myofascial release E) Counterstrain

C The above patient has a lower extremity that is awaiting surgical stabilization. The rhythmic motion of the pedal pump will exert forces at the still unstable fracture site. There are several different techniques to release the thoraco-abdominal diaphragm. Some are very gentle and can be performed on individuals with several concurrent medical or surgical conditions. Answer A Thoracic rib raising and paraspinal inhibition will not harm the recent surgical site in the cervical spine or the lower extremity fracture. Answer B Although myofascial release and counterstrain would be contraindicated in some areas for the above individual, the modalities are not specifically contraindicated and thus not the best answers. Answers D and E

A67-year-old female fell on some ice while walking into her house. She slipped backward landing on her buttock as well as the palm of her hand. She fell in such a way that her right forearm was supinated and her shoulder, elbow and wrist were extended. Based on the above mechanism of injury, which dysfunction is most likely to be seen in this patient? A) Left on Left sacral torsion B) Right on Right sacral torsion C) Right anterior radial head somatic dysfunction D) Right posterior radial head somatic dysfunction E) Right adducted ulna

C The patient has an anterior radial head somatic dysfunction. Radial head somatic dysfunction can often be caused by a fall on an outstretched arm and can result in wrist and elbow pain. Specifically, an anterior radial head somatic dysfunction is usually caused by a fall on an outstretched supinated arm. Supination of the wrist and forearm physiologically causes the radial head to glide anteriorly. A left on left torsion as well as a right on right torsion can be due to trauma, however "falling on the buttocks" is too nonspecific to indicate either dysfunction. Answers A and B A posterior radial head somatic dysfunction can be due to a fall forward on an outstretched pronated arm. Pronation of the wrist and forearm physiologically causes the radial head to glide posteriorly. At the impact of the fall, the arm is positioned with the shoulder flexed, elbow and wrist extended and the forearm pronated. Answer D Abducted ulna is not associated with trauma from falling backward on an outstretched arm. Answer E

A 45-year-old male with a history of diabetes comes to the emergency department with acute onset of low back pain. The pain started suddenly when he was sliding into home plate during a softball game earlier today. The pain is severe and radiates into his right lower extremity. He describes an electric type pain shooting into his right foot. What are some findings you would expect to see when examining him? A) Edematous, boggy tissue with palpation, and full active range of motion of the lumbar spine. Decreased sensation on the dorsum of the foot. B) Severe, sharp pain with palpation of lumbar tissues, guarded range of motion of the lumbar spine, and increased deep tendon reflexes of the right lower extremity. C) Warm tissue texture changes, hypertonic muscles, and decreased range of motion of the lumbar spine. D) Little to no edema or erythema, cool dry skin to palpation, ropy and fibrotic muscles, and decreased range of motion of the lumbar spine. E) Dull pain with palpation. Positive straight leg raising test

C The above patient has an acute somatic dysfunction of the lumbar spine. He likely has herniated lumbar discs. Acute somatic dysfunctions are associated with severe sharp pain (especially with palpation). Acute tissue texture changes are usually described as edematous, erythematous, and boggy. There is increased moisture on the overlying skin and associated hypertonic muscles. The muscle contraction and guarding will cause a limited range of motion of the involved areas. Answer A did not contain the limited range of motion in the lumbar spine that is consistent with this type of injury. Answer A An acute herniated disc results in decreased reflexes of the effected lower extremity. Answer B This patient likely has a positive straight leg raising test and decreased sensation somewhere on the foot, secondary to a herniated disc. Answers D and E included these findings however, they were coupled incorrectly with descriptors of a chronic somatic dysfunction. Chronic tissue texture changes include cool dry skin; flaccid, doughy, or mushy muscles; and firm, ropy, thickened, fibrotic interstitial tissues. Decreased range of motion is secondary to fibrotic tissues or possibly contractures. Answers D and E

Which of the following techniques best evaluates inferior motion of the clavicles at the sternum? A) Patient prone, index fingers on the anterior aspect of the clavicular heads near sternum, patient shrugs shoulders B) Patient supine, index fingers on the superior aspect of the clavicular heads near sternum, patient depresses the shoulders C) Patient supine, index fingers on the superior aspect of the clavicular heads near sternum, patient shrugs shoulders D) Patient supine, index fingers on the anterior aspect of the clavicular heads near sternum, patient protracts the shoulders E) Patient supine, index fingers on the anterior aspect of the clavicular heads near sternum, patient retracts the shoulders

C The clavicle moves in three different planes. First, it moves superior and inferior with shrugging and depressing the shoulder. second, it moves anterior and posterior with retraction and protraction of the shoulder. Third, it rotates anteriorly and posteriorly with internal and external rotation of the arm when it is abducted at 90 degrees Shrugging the shoulder causes the lateral end of the clavicle to move superior, this causes the medial end to move inferior, via a see-saw motion. This motion is easily observed with the patient in the supine position. The clavicle cannot be properly evaluated in the prone position. Answer A Patient supine, index fingers on the superior aspect of the clavicular head near sternum, patient depresses the shoulders. This will move the clavicle superior at the sternum, if no somatic dysfunction is present. Answer B Patient supine, index fingers on the anterior aspect of the clavicular head near sternum, patient protracts the shoulders. This will move the clavicle posterior at the sternum, if no somatic dysfunction is present. Answer D Patient supine, index fingers on the anterior aspect of the clavicular head near sternum, patient retracts the shoulders. This will move the clavicle anterior at the sternum, if no somatic dysfunction is present. Answer E

A patient comes to your office after "his back went out" earlier that morning. Standing postural x rays determine that there is significant sacral base unleveling. Which ligament is thought to be the first ligament to become painful in lumbosacral decompensation? A) Sacrotuberous B) Anterior sacroiliac C) Iliolumbar D) Posterior sacroiliac E) Sacrospinous

C The iliolumbar ligament is often the first ligament to become painful in lumbosacral decompensation. Sacrospinous, sacrotuberous and sacroiliac ligament may become painful in lumbosacral decompensation, however they is not the first. Answers A, B, D, and E

A 47-year-old female presents to your office with low back pain. Upon inspection of the lumbar spine, you observe a left lateral convexity. Palpation reveals fullness in the left paraspinal musculature and L2 - L5 have transverse processes posterior on the left. Given the above information the somatic dysfunction is most consistent with: A) Type II mechanics with the lumbar spine sidebent left, rotated left B) Type II mechanics with the lumbar spine sidebent right, rotated right C) Type I mechanics with the lumbar spine sidebent right, rotated left D) Type I mechanics with the lumbar spine sidebent left, rotated right E) There is not enough information given to make a correct diagnosis

C The patient has a left lateral convexity. In other words, the lumbar spine is sidebent to the right. You also note on palpation that there is left paraspinal fullness and L2 - L5 have transverse processes posterior on the left (Le. L2 - L5 is rotated to the left). When the spine sidebends and rotates in opposite directions, we call this type I mechanics. Type I mechanics occur when the spine is in the neutral position (no flexion or extension) and are typical of group dysfunctions. Type II mechanics occur when the spine is either flexed or extended, in this case sidebending and rotation would be in the same direction. Type II mechanics are typical of single vertebral dysfunctions.

The most likely pelvic diagnosis is: A) Anterior rotated innominate B) Posterior rotated innominate C) Superior innominate shear D) Inferior innominate shear E) Superior pubic shear

C The patient has a superior innominate shear. The findings in this dysfunction are: the ASIS & PSIS are more superior ipsilaterally. There is a shorter leg ipsilaterally and there is a positive standing flexion test ipsilaterally. In an anterior rotated innominate, the ASIS is more inferior ipsilaterally and the PSIS is more superior ipsilaterally. Answer A In a posterior rotated innominate, the ASIS is more superior ipsilaterally and the PSIS is more inferior ipsilaterally. Answer B In an inferior innominate shear the ASIS and PSIS are more inferior ipsilaterally. Answer D In a superior pubic shear, the ASIS's and PSIS's appear to be level and the pubic bone is superior ipsilaterally. Answer E

What is the most likely cause of this patient's scoliosis? A) Idiopathic scoliosis B) Central spasticity C) Psoas syndrome D) Spina bifida E) Muscle weakness

C The patient has some symptoms of right psoas syndrome (high lumbar dysfunction, pelvic shift to the left and left piriformis tenderpoint). Psoas syndrome can be a cause of scoliosis. Although idiopathic scoliosis accounts for most cases (80%), in this case there is a musculoskeletal cause. Answer A Central spasticity indicates an upper motor neuron origin, thus unless there was a history of cerebral or spinal cord injury this would not be the correct answer. Answer B Spina bifida occulta has not been associated with scoliosis. Answer D This patient has no history of muscle weakness. Answer E

As a sports medicine physician you are consulted on a 23 year-old female with patello- femoral tracking syndrome. In addition to medication and osteopathic treatment you decide to facilitate the patient's recovery by recommending exercises to strengthen the vastus medialis. You inform her that while in the supine position with the knee fully extended, she should tighten her quadriceps, hold for 5 seconds then relax. Which of the following statements most accurately describes this type of exercise? A) Eccentric contraction in which there is no increase in vastus medialis tension. B) Concentric contraction in which there is an increase in vastus medialis tension. C) Isometric contraction in which there is an increase in vastus medialis tension. D) Eccentric contraction in which there is a shortening of the muscle. E) Isotonic contraction in which there is an increase is vastus medialis tension

C The patient in the above question is performing a type of isometric contraction. The definition of an isometric contraction is a muscle contraction that results in an increase in tension without a change in length. Concentric and eccentric contractions are contractions in which there is an approximation (concentric) or lengthening (eccentric) of muscular origin and insertion. Tension is not related to these type of contractions. Answers A, Band D An isotonic contraction is one in which there is an approximation of the muscle's origin and insertion without an increase in muscle tension. The difference between concentric and isotonic contractions is that concentric contractions can have variable tensions. Answer E

A 34-year-old female enters your office complaining of hip pain following a fall off of a stepladder 4 days earlier. X-rays taken in the emergency department revealed no fracture, and the patient has been very uncomfortable since the fall. The pain is localized on the left, and does not radiate down the thigh. The left ASIS is cephalad, the left PSIS is caudad and the pubes are level. While performing a standing flexion test, you note that the left PSIS demonstrated greater excursion than the right. Which additional palpatory finding is most likely to be present in this patient? A) Resistance to posterior compression of the right ASIS B) Backward sacral torsion on a left axis C) Longer leg on the right D) L5 neutral sidebent left, rotated right. E) Exquisite pain upon palpation of the pubic symphysis

C The patient is has a left innominate posterior. A shorter leg on the ipsilateral side (in this case, the left leg) will be present in this dysfunction. Consequently, the contralateral leg (in this case, the right leg) will appear longer. Resistance to posterior compression is describing the ASIS compression test. This test will confirm the findings of the seated or standing flexion test. Since there is a somatic dysfunction on the left side, posterior compression on the right is likely to demonstrate adequate resiliency (i.e. it will have normal springing motion). Answer A A backward torsion on a left axis (right on left) is a specific sacral somatic dysfunction and not necessarily associated with an innominate dysfunction. Answer B L5 neutral, sidebent left, rotated right is also not necessarily associated with a left innominate posterior. Answer D Exquisite pain upon palpation of the pubic symphysis could be present with a pubic rami fracture or pubic symphysis somatic dysfunction. It is not necessarily present in a left innominate posterior. Answer E

65-year-old female presents with intermittent numbness of the first three and 1/2 digits of her right hand. Symptoms have been present for one year and were not associated with trauma. Symptoms are worsened with driving her car and typing on her computer. On examination, there is decreased sensation over the first three and 1/2 digits. Tinel's sign is positive at the wrist. There is no weakness or atrophy of the thenar muscles. She has been taking Tylenol with little relief. Her other medications include Nifedipine 10mg QD and Alendronate 70mg once weekly. The most appropriate treatment would include A) Myofascial release to the carpal tunnel followed by HVLA to the cervical spine. B) Cervical and thoracic HVLA to alter sympathetic tone in the upper extremity and splinting the wrist during aggravating activities. C) Myofascial release to the carpal tunnel, rib raising and wrist splinting D) Cervical myofascial release and steroid injection into the carpal tunnel E) Rib raising, NSAID's and referral to a hand surgeon

C The patient most likely has carpal tunnel syndrome. Conservative measures are most appropriate for initial treatment. These include wrist splints, NSAID's and OMT. OMT should be directed at correcting upper thoracic and rib somatic dysfunction and rib raising to decrease sympathetic tone. The patient is taking Fosamax (Alendronate) 70mg weekly indicating that the patient has osteoporosis. Therefore, HVLA is contraindicated. Answers A and B Although a steroid injection may be helpful, a more conservative approach such as OMT, splinting and a trial of NSAIDS should be tried first. Since this patient is elderly and diabetic, there is a higher chance of infection with joint injection. Answer D Referral to a hand surgeon is appropriate after conservative treatment has failed. Answer E

What would be the correct Jones counterstrain treatment position for this patient's tender point? A) Patient supine, hips and knees flexed and sidebent to the right B) Patient prone, hips and knees extended and sidebent to the right C) Patient supine, hips and knees flexed and sidebent to the left D) Patient prone, hips and knees extended and sidebent to the left E) Patient supine, hips and knees extended and sidebent to the left

C The psoas muscle is primarily a hip flexor, so the patient should be in the supine position, not prone (Answers B and D), with the hip and knees flexed, not extended (Answer E) and sidebent toward the affected muscle, not away (Answer A) L1 AND L5: FLEX AND STRAW L2-L4: FLEX SART

Which of the following organs would be LEAST affected by an autonomic imbalance at T4? A) Heart B) Esophagus C) Stomach D) Lungs E) Tongue

C The stomach generally receives autonomic innervation from T5-T9. Although there is some overlap, out of the other answers T4 is least likely to affect the stomach. The heart receives autonomic innervation from T1 - T5. Answer A The esophagus receives autonomic innervation from T2 - T8. Answer B The respiratory system receives autonomic innervation from T2 - T7. Answer D The tongue is a head structure and will receive autonomic innervation from T1 - T4. Answer E

A patient complains of epigastric pain that is relieved upon eating food and recurs about three hours after he eats. Upper endoscopy is performed and erosions are found in the gastric mucosa. At which spinal level would you expect to find palpatory changes related to a viscerosomatic reflex? A) C7 B) T4 C) T6 D) T10 E) T12

C This patient has peptic ulcer disease. The gastric ulcer would cause viscerosomatic changes to occur at the T6 spinal level. C7 is not associated with viscerosomatic changes related to peptic ulcer disease. Answer A T4 receives sympathetic innervation from the head and neck, heart, esophagus and lungs. Answer B T10 and T12 receive sympathetic innervation from the middle and lower GI tract. Answers D and E

Treatment of which area would most likely decrease hypersympathetic tone to this patient's heart? A) Occipito-atlantal joint B) C4 C) T2 D) T6 E) T7

C The sympathetic innervation to the heart generally stems from T1 - T5. Treatment to this area may decrease sympathetic tone in this patient. The OA (atlanto-occipital) joint has a direct influence on the function of the vagus nerve (parasympathetic innervation to the heart). Therefore treatment of the OA (atlanto-occipital) joint may alter parasympathetic tone to the heart. Answer A Manipulation of the cervical spinal segments near the cervical chain ganglia may influence sympathetic tone to head and neck structures, not the heart. Answer B Although some texts list T6 as a level for sympathetic influence to the heart, not all texts agree. However, they all do agree on T2, therefore T6 is not the best answer. Answer D T7 is not considered to have sympathetic influence to the heart. Answer E

The above patient would have a predictable reflex gangliform contraction located at A) Right proximal area of the iliotibial tract B) Right distal area of the iliotibial tract C) Left proximal area of the iliotibial tract D) Left distal area of the iliotibial tract E) The superior rami of the pubis

C The term "gangliform contraction" is the original term Frank Chapman used for his reflex points. It was not until much later that these "gangliform contractions" became "Chapman's points". The Chapman's point for the rectosigmoid colon is located on the left proximal femur at the greater trochanter in the iliotibial tract. The Chapman's point for the iIiocecal area is located on the right proximal femur at the greater trochanter within the iliotibial tract. Answer A The Chapman's point for the right half of the transverse colon is located on the right distal femur in the iliotibial tract. Answer B The Chapman's point for the left half of the transverse colon is located on the left distal femur in the iliotibial tract. Answer D The Chapman's point for the ovaries and urethra is located on the superior rami of the pubis. Answer E

24-year-old female presents for a routine physical examination. While assessing her posture from the side you observe that an imaginary plum line dropped from the ceiling to the floor would fall posterior to the apex of the coronal suture, through the external meatus, through most of the bodies of the cervical vertebrae, through the shoulder joint, through the bodies of the lumbar vertebrae, just posterior to the axis of the hip, slightly anterior to the axis of the knee joint and slightly anterior to the lateral malleolus. Which of the following best describes this patient's posture A) Military posture B) Swayback posture C) Ideal posture D) Flat back posture E) Anterior postural deviation

C This is a description of ideal posture when viewing a patient from the side. Military posture, as viewed from the side (in relation to a plumb line) is described as: head tilted slightly posteriorly, cervical curve and thoracic curve normal, anterior cervical and posterior thoracic deviation from plumb line, anterior pelvic tilt, knees extended, ankles plantar flexed. Answer A. Swayback posture as viewed from the side (in relation to a plumb line) is described as: head forward, cervical spine lordotic, thoracic spine kyphotic, decreased lordosis of lumbar spine, posterior tilt of pelvis, hip and knee joints hyper-extended. Answer B Anterior postural deviation as viewed from the side (in relation to a plumb line) is described as: entire body leans forward, deviating anteriorly from plumb line, patient's weight supported by metatarsals. Answer E Flat back posture as viewed from the side (in relation to a plumb line) is described as: head forward, cervical spine has slightly increased lordosis, thoracic spine slightly kyphotic in upper portion then flattens in lower segments, lumbar lordosis flattened, and hips and knees extended. Answer D

A 44-year-old female with severe rheumatoid arthritis comes to your office with neck pain. She has a history of chronic neck pain, however recently her pain has increased. She denies any trauma and does not recall any specific event that would cause her neck pain to increase. On examination, she has severely diminished range of motion. Palpation of her cervical paraspinals reveals areas of fibrotic tissue with some tenderness noted in the sub-occipital region. The third cervical segment is sidebent left. Neurologic examination is normal Which of the following describes an appropriate osteopathic technique for C3? A) C3 is sidebent to the left and rotated to the right against the restrictive barrier, the patient then rotates to the left against the physician's counter force for 3-5 seconds, then the physician passively rotates the cervical spine farther to the left. B) C3 is sidebent and rotated to the right against the anatomic barrier, the patient then rotates to the left against the physician's counter force for 3-5 seconds, then the physician passively rotates the cervical spine farther to the right. C) C3 is sidebent and rotated to the right against the restrictive barrier, the patient then rotates to the left against the physician's counter force for 3-5 seconds, then the physician passively rotates the cervical spine farther to the right. D) The patient's neck is sidebent to the left and rotated to the left, a high velocity, low amplitude right rotational thrust is applied to the third cervical segment. E) The patient's neck is sidebent and rotated to the right, a high velocity, low amplitude right sidebending thrust is applied to the third cervical segment.

C This is an example of direct muscle energy using post-isometric relaxation technique. Sidebending and rotation in the third cervical segment are always toward the same side (type II mechanics). When positioning a patient for a muscle energy technique, C3 should be sidebent and rotated toward the same side. Answer A C3 is side bent and rotated to the right against the restrictive (not anatomic) barrier, the patient then rotates to the left against the physician's counter force for 3-5 seconds, then the physician passively rotates the cervical spine farther to the right. Answer B Since the patient has severe rheumatoid arthritis HVLA is relatively contraindicated and therefore it is not the most appropriate technique. In addition, the patient is not correctly positioned for an HVLA technique in answers D and E.

A patient presents with right forearm and wrist pain after a fall. There are no neurologic deficits. X-ray of the cervical spine and right arm are normal. Physical exam reveals an increased carrying angle on the right, the hand and wrist are adducted and the olecranon process prefers medial glide. The most likely diagnosis is: A) Posterior radial head dysfunction B) Adduction dysfunction of the ulna C) Abduction dysfunction of the ulna D) Anterior radial head dysfunction E) Medial epicondylitis

C This is the presentation of an abduction dysfunction of the ulna. A posterior radial head dysfunction would cause the forearm to prefer pronation, and the radial head would resist anterior glide. This dysfunction is caused by falling on a pronated forearm. The carrying angle is not affected in radial head dysfunctions. Answer A An adducted dysfunction of the ulna would have a decreased carrying angle, the hand and wrist would be abducted and the olecranon process would prefer lateral glide. Answer B An anterior radial head dysfunction would cause the forearm to prefer supination and the radial head would resist posterior glide. Falling backward on a supinated forearm can cause this dysfunction. Answer D Medial epicondylitis (also known as golfer's elbow) is a strain of the flexor muscles of the forearm near the medial epicondyle. It is associated with pain at the medial epicondyle; it is not associated with a change in the carrying angle. Answer E

A patient in severe unrelenting pain from a recently herniated disc in the lumbar spine seeks treatment for his condition. The patient has a history of peptic ulcer disease and deep vein thrombosis. The patient is currently taking Coumadin (Warfarin) daily. What would be the most appropriate management of this patient? A) Non-steroidal anti-inflammatory agent, bed rest for 5 days, indirect techniques B) Non-steroidal anti-inflammatory, bed rest for 2 days, counterstrain C) Opioid analgesic, bed rest for 2 days, indirect techniques D) Acetaminophen, bed rest for 5 days, HVLA E) Opioid analgesic, bed rest for 5 days, muscle energy

C This patient is in severe pain. Usually, first line medications are non-steroidal anti inflammatory agents. However in this case, they are contraindicated because the patient is at high risk for a GI bleed (a history of peptic ulcer disease complicated by the fact that he is on Coumadin [Warfarin)). Therefore given his level of pain, an opioid is indicated. Deyo and colleagues demonstrated that bed rest for two days helped patients return to work quicker and did not worsen their back pain. In addition, prolonged bed rest in the above patient may predispose him to forming another DVT. A patient with this level of pain is well suited for gentle techniques such as indirect techniques. NSAIDS are contraindicated in this patient due to the high risk of GI bleed. Answers A and B Five days of bed rest would not benefit the patient any more than 2 days and HVLA may injure the patient. Answer D Bed rest for two days did not worsen patient's back pain and helped patients return to work quicker. Therefore this is not the best answer. Answer E

A 65-year-old female comes to your office with frequent burning substernal and epigastric pain. The pain is associated with fatty meal intake and is relieved with over the counter antacids. The pain has been present for one year but has gotten worse over the past several weeks. Based on the above diagnosis, palpatory changes are likely to arise at which of the following vertebral levels? A) C5 B) T3 C) T6 D) T10 E) T12

C This patient is likely to have a gastric ulcer or GERD. Unlike duodenal ulcers, gastric ulcers symptoms typically worsen after eating. Abnormal stimuli from the gastric mucosa will enter the spinal cord at the T5 - T9 level and facilitate these segments. This may result in abnormal efferents to somatic structures leading to palpatory changes at these levels. Since C5 is not closely associated with autonomic innervation to the upper GI tract. Answer A Efferents from T3 affects sympathetic tone in the heart, lung and head. Answer B In the Gl system T10 - T11 is associated with sympathetic tone in portions of the duodenum and the pancreas, jejunum, ileum and proximal 2/3 of transverse colon. Answer D In the Gl system T12 - L2 is associated with sympathetic tone in the distal 1/3 of the transverse colon and rectum. Answer E

The delayed development of pain can best be explained by: A) No previous history of back pain B) The force imparted by the ground transmitting to the lumbar spine immediately causing local somatic dysfunction C) Postural compensatory mechanisms D) A somatoform disorder E) The patient has a high tolerance to pain

C When somatic dysfunction occurs due to a short leg syndrome, the lumbar paraspinal musculature compensates so that the spine may remain straight and the eyes level. The lower back pain could have developed due to the overuse of these compensating dorsal muscles. There is no temporal relation to the onset of back pain after minor trauma and previous history of back pain. Answer A Although the force imparted by the ground probably did result in somatic dysfunction, this does not explain why it took two weeks for the pain to develop. The length of time from onset and the identification of a primary mechanism strongly suggest that the patient's back pain is the result of a secondary process. Answer B Although somatoform disorder (psychogenic etiology) is possible, given a primary offending cause, this is not the most likely cause of this patient's pain. Answer D If the patient has a high tolerance to pain, there is no reason why he would be experiencing pain now if the intensity of the pain is the same as it were two weeks ago. Answer E

A 19-year-old male with no history of past medical illness comes to your office with neck pain. Active neck range of motion reveals painful right rotation to 45 degrees, and painless left rotation to 90 degrees. Which of the following statements is true regarding this patient's range of motion A) Since the patient has limited right rotation he likely has limited left sidebending B) Since the patient has limited right rotation he likely has limited right sidebending C) This patient is likely to have decreased passive range of motion to the right. D) The patient is likely to have full passive range of motion with right rotation to the physiologic barrier. E) This patient demonstrated that he has full left rotation to the anatomic barrier.

C When you ask a patient to actively move a joint to its end points you are asking the patient to demonstrate active range of motion. Passive range of motion is when a physician (or any other person for that matter) ranges a joint. Any person can actively move his/her joint to its physiologic barrier (if there is no somatic dysfunction present). Any other person can move someone else's joint to its anatomic barrier (if there is no somatic dysfunction present). When there is somatic dysfunction present, that person or physician can only move the joint to the restrictive barrier. Thus, all motion is lost beyond the restrictive barrier. The question does not state which segment (or segments) is (are) restricting motion. Therefore, it cannot be determined if the dysfunctional segment follows type I or type II mechanics. Answers A and B All motion (active or passive) is lost beyond the restrictive barrier. Answer D The patient demonstrated full left rotation to the physiologic barrier, not the anatomic barrier. Answer E

A 34-year-old male truck driver presents to you with a chief complaint of moderate to severe low back pain. He gives a history of falling onto his buttocks three days prior. With the patient standing, there is a noticeable absence of the lumbar lordosis. With the patient prone, the ILA's are equal in depth. Motion testing reveals no motion at the lumbosacral junction. Which of the following most correctly describes the sacral dysfunction? A) A bilaterally flexed sacrum that has rotated around its superior transverse axis. B) A bilaterally extended sacrum that has rotated around its superior transverse axis. C) A bilaterally flexed sacrum that has rotated around its middle transverse axis D) A bilaterally extended sacrum that has rotated around its middle transverse axis E) The axis for the sacral dysfunction cannot be specified.

D A noticeable absence in the lumbar lordosis, no motion of the lumbosacral junction and symmetrical ILA's indicate a bilaterally extended sacrum. A bilateral sacral extension (a.k.a. sacral base extension or extended sacral base) somatic dysfunction occurs about a middle transverse axis. In a patient with a bilaterally flexed sacrum, the lumbosacral junction will spring easily and the patient is likely to have an increased lumbar lordosis. In addition, respiration and craniosacral motion occur about the superior transverse axis. Answers A, B, and C A bilaterally flexed or extended sacrum will have rotated around the middle transverse axis; therefore, the axis can be specified. Answer E

The sacral diagnosis is: A) Right rotation on a right oblique axis B) Right rotation on a left oblique axis C) Left rotation on a left oblique axis D) Left rotation on a right oblique axis E) Right unilateral sacral extension

D A positive lumbosacral spring test limits the choices to a backward sacral torsion and unilateral sacral extension. The left ILA is posterior and slightly inferior and the left sacral sulcus is shallow, indicating left rotation on a right oblique axis. Right rotation on a right oblique axis and left rotation on a left oblique axis would have negative lumbosacral spring tests. Answers A and C In a right rotation on a left oblique axis, the right sulcus would be shallow and the left ILA would be anterior and slightly superior. Answer B In a right unilateral sacral extension, the right sulcus would be shallow. Answer E

A 23-year-old medical student is complaining of left sided low back/sacroiliac pain. The pain started one week ago while studying for board exams. On examination, you notice he has a positive standing flexion test on the left, his left ASIS is inferior, the left PSIS is superior and the right sacral sulcus is freely mobile. Based on the information given, what is the most likely diagnosis? A) Left sacral rotation on a left oblique axis (L on L) B) Unilateral sacral flexion on the right (USFA). C) Left posterior innominate. D) Left anterior innominate. E) Left superior innominate shear

D A positive standing flexion test indicates that the patient has a somatic dysfunction of the pelvis. In this case the positive standing flexion test is on the left side, therefore the dysfunction is on the left side. An inferior ASIS and a superior PSIS on the ipsilateral side indicates an anterior innominate. With a unilateral sacral flexion on the right (USFR) and a left sacral rotation on a left oblique axis (L on L), the right sulcus is freely mobile. However the question does not mention that it is deep, nor does the question state that there is a positive seated flexion test. Therefore you cannot assume sacral dysfunction in the above question. Answer A and B With a left posterior innominate the left ASIS would be superior, and the left PSIS would be inferior. Answer C With a superior innominate shear (upslip innominate or superior innominate subluxation) both the left ASIS and the PSIS would be superior. Answer E

An 81-year-old male comes to your clinic with a complaint of right shoulder pain. He injured his shoulder while playing golf three months ago. He went to the emergency room, where the doctor told him he most likely tore his rotator cuff. He was given a sling and discharged. He has been using the sling for three months. His pain is mostly resolved, however he still has some pain with shoulder movement. On examination, inspection of the shoulder is normal without atrophy. There is tenderness at the tip of the acromion, however shoulder impingement signs are negative. Active and passive range of motion testing reveals: 60 glenohumeral abduction, 60° of forward flexion, 10° of external rotation, 20° of internal rotation of the left, extension is normal. Pain is present at the end of range of motion in all planes. Range of motion testing of the left shoulder is normal. X-rays are normal and there is no evidence of a high riding humeral head. Which of the following statements best explains this patient's decreased range of motion. A) The patient suffered a partial tear in his rotator cuff resulting in pain and thus limiting his range of motion. B) The patient suffered a complete tear in his rotator cuff resulting in limited range of motion C) Chronic impingement has resulted in inflammation of the rotator cuff tendons thus causing limited range of motion. D) Prolonged immobility of the shoulder has resulted in fibrosis of the joint capsule causing decreased range of motion. E) A tear in the anterior glenoid labrum resulted in decreased range of motion in the stated planes and preserved range of motion with extension

D Although the patient may have suffered a rotator cuff tear, it was the prolonged immobility that resulted in subsequent adhesive capsulitis. The hallmark feature of adhesive capsulitis is decreased range of motion with end range pain. Typically, adhesive capsulitis does not produce a decrease in extension range of motion. A rotator cuff tear usually does produce shoulder pain and decreased active range of motion, however usually passive range of motion will be preserved. Classically, the prolonged immobilization will result in some degree of adhesive capsulitis and this will result in decreased passive range of motion. In addition, in a complete rotator cuff tear there will be a high riding humeral head (i.e. the superior portion of the humeral head will approximate the inferior surface of the acromion) Answers A and B Chronic impingement can result in inflammation of the rotator cuff tendons, however this usually does not directly affect both passive and active range of motion. Answer C A tear in the glenoid labrum is not necessarily associated with decreased range of motion with end range pain. In addition, a torn glenoid in a particular area does not decrease range of motion in one plane and preserve motion in another plane. Answer E

Which of the following lymphatic techniques listed would be the most appropriate initial treatment in this patient? A) Liver pump B) Abdominal pump C) Classic thoracic pump D) Thoracic inlet release E) Pedal pump

D As a general rule, it is best to release any diaphragms, especially the thoracic inlet, prior to starting any pumping techniques. Once the diaphragms are free of restriction and sympathetic tone is normalized, lymphatic pumps will help return lymph through open channels.

Upon further assessment of ribs 8 - 10 the lower edge of the rib shaft is more prominent and more elevated laterally than anteriorly. There was a failure of these ribs to move during exhalation. What is the most likely diagnosis? A) Ribs 8-10 exhalation dysfunction, pump handle predominant B) Ribs 8-10 exhalation dysfunction, bucket handle predominant C) Ribs 8-10 inhalation dysfunction, pump handle predominant D) Ribs 8-10 inhalation dysfunction, bucket handle predominant E) Ribs 8-10 inhalation dysfunction, caliper motion predominant

D Bucket handle ribs 8-10 that have an inhalation dysfunction do not move caudally in exhalation and may cause pain during the exhalation phase of respiration. Upon static assessment, the lower edge of the rib shaft is more prominent and elevated laterally. Answers A and B are incorrect because the ribs are not in exhalation dysfunction. If the ribs had a pump-handle dysfunction, they would be elevated anteriorly. Answer C Laterally elevated ribs are more closely associated to bucket-handle dysfunctions, not caliper dysfunctions. Answer E

Which of the following best explains the role of the quadratus lumborum A) Inhalation causes relaxation of the quadratus lumborum and permits cephalad movement of rib 12. B) Contraction will result in posterior translation of ribs 11 and 12. C) It is the 1st structure to react to stress in the lumbosacral area. D) Contraction will increase lumbar lordosis and increase the intercostal space between the 11 th and 12th ribs. E) It plays little role in lymphatic drainage

D Contraction of the quadratus will extend, and side bend the lumbar spine as well as pull rib 12 down (caudad) thus increasing the 11th and 12th intercostal space. Inhalation results in an eccentric contraction of the quadratus lumborum resulting in cephalad movement of rib 12. The latissimus dorsi also plays an important role in the movement of rib 12. Answer A The quadratus lumborum does not attach to rib 11. Answer B Although the quadratus lumborum tenderpoints have been referred as the most often overlooked cause of low back pain, it is thought that the iliolumbar ligament is one of the first structures to react to stress in the lumbosacral spine. Answer C Treatment of the diaphragm as well as the quadratus lumborum has proven successful in helping to promote lymphatic drainage. Answer E

The most appropriate treatment for the pain would be: A) Muscle energy treatment of the first rib B) HVLA treatment of the cervical rib C) Reduction of the shoulder D) Counterstrain treatment of the scalene muscles E) Morphine and nitrates

D Counterstrain of the hypertrophied scalene would cause reduced compression on the neurovascular bundle, and is the appropriate treatment. Although the patient may have a rib 1 dysfunction, the primary etiology of this patient's symptoms is due to scalene hypertrophy. Therefore treatment of rib 1 is not the best answer. Answer A Since this patient does not have a cervical rib this would not be the best answer. In addition, a 63 year- old COPO patient may have osteopenia, which would be a contraindication for HVLA. Answer B Reduction of the shoulder is the treatment for a shoulder dislocation. Answer C Morphine and nitrates are the treatment for a myocardial infarct. Answer E

Which of the following is considered to be the most common congenital anomaly in the lumbar spine? A) Sacralization B) Lumbarization C) Facet hypertrophy D) Facet tropism E) Spina bifida occulta

D Facet tropism is a misalignment of the facet joint. According to Osteopathic Principles in Practice it is the most common anomaly of the lumbar spine. Sacralization occurs in approximately 3.5% of the population. Answer A Lumbarization occurs in <1 % of the population. Answer B Facet hypertrophy is not a congenital anomaly. It is a condition often seen with degenerative changes. Answer C Spina bifida occulta occurs in about 10% of the population. Answer E

A week later, the patient returns with low back pain. Her head is now located forward of the plum line. She has a slight increase in her cervical lordosis. Her thoracic spine is slightly kyphotic in the upper segments with a flattening of the lumbar spine. Her lumbar vertebrae are straightened and have moved posterior relative to the plum line. Which of the following best describes this patient's new posture? A) Military posture B) Swayback posture C) Posterior postural deviation D) Flat back posture E) Anterior postural deviation

D Flat back posture as viewed from the side (in relation to a plumb line) is described as: head forward, cervical spine has slightly increased lordosis, thoracic spine slightly kyphotic in upper portion then flattens in lower segments, lumbar lordosis flattened, and hips and knees extended. Posterior postural deviation as viewed from the side (in relation to a plumb line) is described as: entire body leans backward, deviating posteriorly, balance maintained by anterior thrust of pelvis and hips, and marked lordosis from mid-thoracic spine down. Answer C

While evaluating a mid thoracic somatic dysfunction, your left thumb is more anterior than your right. Your thumbs become symmetric when the patient is in flexion, however you cannot examine the patient in extension because the pain is too great. The most likely diagnosis is A) FRLSL B) ERLSL C) NRRSL D) FRRSR E) ERRSR

D Flexed, rotated right, sidebent right is the correct diagnosis. On the board exams it may be abbreviated as FRrSr. The static evaluation in the neutral position shows that the left thumb is more anterior, which means the segment is rotated right. Since the asymmetry at the segment resolved with flexion, the freedoms of motion are flexion, right rotation and left sidebending. In a FRlSl (flexed rotated left, sidebent left) somatic dysfunction your left thumb would be posterior, not anterior. Answer A In an EFlSL (extended rotated left, sidebent left) somatic dysfunction your left thumb would be posterior, not anterior, and symmetry would be restored in extension. Answer B In a NRrSl (neutral, rotated right, sidebent left) somatic dysfunction symmetry would not be restored in flexion. Answer C In an E~SR (extended, rotated right sidebent right) somatic dysfunction symmetry would be restored in extension. Answer E

Head and neck lymphatic congestion and drying of the nasopharyngeal mucosa is most likely associated with increased autonomic activity originating from the pre-ganglionic nerve fibers of which structure? A) Vagus nerve B) Inferior cervical ganglia C) Superior cervical ganglia D) Intermediolateral cells of the spinal cord at T1 - T4 E) Intermediolateral cells of the spinal cord at T5 - T7

D Hypersympathetic activity to the head and neck structures will lead to vasoconstriction producing lymphatic congestion. It will also inhibit secretion from the nasopharyngeal mucous membranes that produces dryness. Sympathetic nerve fibers arise from the intermediolateral cells of the spinal cord. Although it has nothing to do with this question, hypersympathetic activity to the respiratory epithelium (lungs) will lead to increased secretion via increased goblet cell production. The parasympathetic fibers (from the vagus nerve) will not lead to nasal and pharyngeal dryness or lymphatic congestion. Answer A Postganglionic fibers that supply the head and neck structures stem from the cervical chain ganglia, however pre-ganglionic fibers originate from the intermediolateral cell column in the spinal cord. Answers B and C The autonomic nervous system that supplies the head and neck do not originate from T5 - T7. Answer E

In a patient with a history of tension headaches, structural exam reveals the following: C5 resists lateral translation to the left. There is right-sided fullness at C2 and the right articular pillar of C2 resists anterior glide. Given the above information, what else is most likely true regarding physical examination? A) C2 is flexed B) C2 is rotated left C) The atlanto-axial joint is rotated right D) C5 is rotated left E) The atlas is rotated left

D If C5 resists lateral translation to the left. This indicates that C5 resists right sidebending. If it resists right sidebending, it is sidebent left. If it is sidebent left, it must be rotated left (C5 follows the rules of type II mechanics). Since the question did not test C2 in flexion and extension, we do not know whether C2 is flexed, extended or neutral. Answer A A common way to test for rotation in the cervical spine is to push anterior on the articular pillar of the cervical spine at each segment. If there is resistance on one side compared to the other then the segment Is rotated toward that side. In order to test rotation at C2 the practitioner would push anterior on each articular pillar. The above patient has paraspinal fullness on the right and the right articular pillar resists anterior glide, this indicates that C2 is rotated to the right. Answer B The above question does not describe the direction of the atlantoaxial joint (C1 on C2). Answers C and E

A patient complains of right leg pain for two days. The patient noticed the pain while running; the pain is located on the leg just below the knee joint on the lateral side. The right fibular head resists anterior springing; the left foot dorsiflexes more than the right. The most likely diagnosis is: A) Anterior talar dysfunction on the left B) Dorsiflexion dysfunction of the talus on the right C) Anterior fibular head dysfunction on the left D) Posterior fibular head dysfunction on the right E) Posterior fibular head dysfunction on the left

D In a posterior fibular head dysfunction the foot will prefer supination and the proximal fibular head will resist springing anteriorly. Since there is pain in the right leg, pain can be used as the indicator of laterality or sidedness of the dysfunction in the extremities. An anterior talar dysfunction is associated with resistance to posterior glide, a foot that prefers plantar flexion and has restriction to dorsiflexion. Answer A An posterior talar dysfunction is associated with resistance to anterior glide, a foot that prefers dorsiflexion and has restriction to plantar flexion. Answer B In an anterior fibular head dysfunction the foot prefers pronation. The proximal fibular head will resist springing posteriorly. In addition, the patient's history does not indicate a problem with the left foot. Answer C In a posterior fibular head dysfunction the foot will prefer supination. The proximal fibular head will resist springing anteriorly. The patient had pain in the right leg, indicating it was a right-sided dysfunction, not left. Answer E

In a patient with low back pain, L5 is flexed and sidebent right. Which of the following is true? A) You would expect L5 to rotate easily to the left, based on the laws of type II spinal mechanics. B) You would expect L5 to sidebend easily to the right, based on the laws of type I spinal mechanics C) You would expect L5 to resist right rotation in the flexed position. D) You would expect the right transverse process of L5 to become more posterior as the spine moves from flexion to extension. E) You would expect the sacrum to be rotated forward on a left oblique axis if a torsion was present.

D In the above patient L5 is flexed sidebent right and rotated right. Upon palpation of L5 transverse process, in the neutral position the right transverse process should be posterior. With flexion the asymmetry should resolve. With extension however the asymmetry will re-appear. You would expect L5 to rotate easily to the right, based on the laws of type II spinal mechanics. Answer A Since L5 is sidebent and rotated to the right, it will follow the laws governed by Type II spinal mechanics. Answer B Since L5 is sidebent and rotated to the right, there should be an ease of right rotation with the segment in the flexed position. Answer C If a sacral torsion was present, the sacrum would be rotated left on a right oblique axis (backward sacral torsion). Answer E

Which of the following studies is considered the gold standard for diagnosis of the above condition? A) Magnetic resonance imaging B) Arteriography C) Plain x-ray D) Nerve conduction studies/Electromyography E) Bone scan

D Nerve conduction studies/electromyography is considered to be the gold standard for the diagnosis of carpal tunnel syndrome. Nerve conduction studies will identify if there has been any damage to the myelin or axon of the median nerve. Electromyography will identify if this damage is severe enough to cause denervation of the distal muscles innervated by the median nerve. Other studies listed have essentially no role in the diagnosis of carpal tunnel syndrome. Answers A, B, C and E

A 42-year-old cachectic cancer patient is complaining of buttock pain that radiates to the left calf. The patient is a computer programmer who is working a lot of overtime. The patient has lost 50 Ibs, which he attributes to adverse effects of chemotherapy. His referred pain is worsened with prolonged sitting. Physical exam reveals no neurologic deficits in the lower extremities. There is a tender point located in the middle of the left buttock. The lumbar spine is symmetric and has full range of motion without pain. The most likely diagnosis is: A) Psoas syndrome B) Posterior facet syndrome C) Cauda equina syndrome D) Piriformis syndrome E) Sacroiliac joint syndrome

D Piriformis syndrome is a peripheral neuritis of the sciatic nerve. Symptoms are easily confused with those of a herniated lumbar disk or facet joint pathology. The patient often complains of hip and buttock pain radiating down the posterior thigh, possibly to the calf or foot. On physical exam there is an absence of neurologic deficits, and a characteristically exquisite tenderpoint anywhere along the piriformis muscle. Factors causing this syndrome are irritation or inflammation of the sciatic nerve, including piriformis muscle spasm, local trauma to the buttocks, repeated mechanical stressors (e.g. running) pelvic instability and excessive local pressure (e.g. sitting), especially in thin or cachectic patients. Psoas syndrome presents with low back pain, especially in the lumbar region with radiation down the posterior thigh only to the knee. The Jones tenderpoint associated with this syndrome is about one inch medial to the anterior superior iliac spine. Answer A Posterior facet syndrome presents with low back pain, which radiates to the buttocks and sometimes the calf or ankle. Physical examination generally reveals motion restriction of one or more vertebral segments, limited spinal range of motion, local paravertebral spasm and tenderness. Pain is typically exacerbated with lumbar extension. There would be no characteristic Jones tenderpoint at the piriformis muscle, and no neurologic deficit. Answer B Cauda equina syndrome is compression of the nerve roots of the cauda equina. Typical symptoms are saddle anesthesia, decreased deep tendon reflexes, decreased rectal tone, and loss of bowel and bladder control. Answer C Sacroiliac joint syndrome usually presents with pain at the sacroiliac joint area, restricted sacral motion, and associated spasm of one or more of the muscles attaching to the sacrum. Pain is typically at the SI joint itself, not in the middle of the buttock. Answer E

Which of the following is true regarding the sympathetic nervous innervation to the kidneys and ureters? A) Sympathetic innervation arises from the cell bodies in the spinal cord at the level of T8 T10. B) Increased sympathetic tone causes vasoconstriction of afferent arterioles thus increasing the glomerular filtration rate. C) Stimulating sympathetics will restore normal peristaltic waves in the ureters D) Preganglionic fibers for the kidney and upper ureters synapse in the superior mesenteric ganglion. E) Sympathetic stimulation contracts the detrusor and relaxes the trigone resulting in micturition

D Preganglionic fibers for the kidney and upper ureters synapse in the superior mesenteric ganglion, while fibers for the lower ureter synapse on the inferior mesenteric ganglion. Sympathetic innervation arises from the cell bodies in the spinal cord at the level of T1 0 - L1. Answer A Sympathetic stimulation causes vasoconstriction of afferent arterioles thus decreasing the glomerular filtration rate. Answer B The parasympathetics will maintain normal peristaltic waves in the ureters, whereas sympathetic stimulation could cause ureterospasm. Answer C Sympathetic stimulation contracts the trigone (sphincter), relaxes the detrusor (bladder wall) resulting in urinary retention. Answer E

A 45-year-old diabetic male has chronic renal insufficiency. Structural examination is likely to reveal: A) Ropy paraspinals at C3 B) Severe sharp tenderpoints at T5 and T6 C) Fibrotic paraspinals T8 and T9 D) Ropy paraspinals at T1 0 - L1 E) Fibrotic tissue texture changes at the sacrum

D Renal insufficiency has been associated with tissue texture changes at the thoracolumbar junction. Key terms that describe chronic tissue texture changes are ropy and fibrotic. The parasympathetics, which supply the kidney and upper ureter, originate from the vagus nerve and are influenced by the occiput, C1 and C2, not C3. Although it is not known how the parasympathetics affect the kidney, they do maintain normal peristaltic waves in the ureters. Answer A T5 and T6 are not segmentally related to the kidney. Answer B Sympathetic fibers arising from T8 - T9 do not supply the kidney. Answer C Parasympathetics from the pelvic splanchnic supply the lower ureter and bladder. Answer E

Treatment should be directed at one which of the following: A) Psoas muscle B) Posterior facet capsule C) Sacroiliac joint D) Piriformis muscle E) Surgical decompression of the lumbar spinE

D Since the patient has piriformis syndrome caused by repeated irritation of the piriformis muscle due to his prolonged sitting. Treatment should be directed at the piriformis muscle for maximum effectiveness. The psoas muscle would be indicated for psoas syndrome. Answer A Treatment directed at the posterior facet capsule would be appropriate for posterior facet syndrome. Answer B Treatment for sacroiliac joint would be indicated for sacroiliac joint syndrome. Answer C Immediate surgical decompression of the lumbar spine is the appropriate management of cauda equina syndrome. Answer E

You decide that this patient may benefit further from a heel lift. Which statement best describes an appropriate heel lift therapy protocol? A) A 1/8" heel lift should be placed under the right foot and increased 1/8" every two weeks until a total height of 3/4" is achieved. B) A 1/8" heel lift should be placed under the right foot and increased 1/16" every two weeks until a total height of 3/8" is achieved. C) A 1/8" heel lift should be placed under the right foot and increased 1/8" in two weeks for a total height of 1/2". D) A 1/8" heel lift should be placed under the right foot and increased 1/8" in two weeks for a total height of 1/4". E) A V2" heel lift should be placed under the right foot and the patient should return in two weeks for follow-up heel lift evaluation.

D The "flexible" (Le. not elderly or "fragile") patient should begin with 1/8" (-3.2mm) heel lift and increase 1/8" every two weeks. The final lift height should be 1/2 - 3/4 of the measured leg length discrepancy, unless there was a recent sudden cause of the discrepancy (I.e. hip fracture or hip prosthesis). Since, the initial discrepancy was 12mm, an acceptable value for the final height would be 6.4mm (1/4") to 9mm (-3/8"). 3/4" is approximately 19.2mm which is much higher than the original height. Answer A Although 3/8" is an acceptable final height, since the patient is not considered "fragile", the lift should be increased 1/8" every 2 weeks, after 1/4" is achieved additional height should be added to the outside of the shoe. Answer B Since the maximum heel lift should be 1/2 to 3/4 of the total height, 1/2" (12.8mm) is too high. In addition, a maximum of 1/4" may be applied to the inside of the shoe. If > 1,4" is needed, then this must be applied to the outside of the shoe. Answer C Since there was not a sudden cause of the leg length discrepancy (Le. hip fracture or hip prosthesis, the final lift height should be 1/2 - 3/4 of the measured leg length discrepancy. Answer E

Treatment to which structure will result in the greatest reduction of excessive autonomic tone in the upper extremity A) The occiput B) C3 C) C7 D) T3 E) T9

D The above patient has carpal tunnel syndrome (CTS). This condition is due to a compression of the median nerve, usually at the transverse carpal ligament. However, cervical or rib dysfunction as well as anterior scalene hypertonicity may further complicate CTS. Ipsilateral upper thoracic and rib dysfunction will result in increased upper extremity sympathetic tone producing arm and hand symptoms. Decreasing sympathetic tone is an integral part of OMT treatment for CTS. The upper thoracic segments (specifically T2 - T8) supply sympathetics to the upper extremity. Sympathetic innervation arises from the thoracic and lumbar regions. Treatment of the occiput, C3, and C7 will not alter the sympathetic tone in the upper extremity. Answers A, B and C T9 is a segment that generally receives sympathetic fibers from structures below the diaphragm. Answer E

An 85-year-old male with a history of severe cervical degenerative joint disease comes to your office. He states that his neck pain started several years ago and has gotten progressively worse. The pain radiates into the upper extremity and fingertips. Pain is worsened with cervical spine extension. What are some of the findings you would expect to see when examining the cervical spine? A) Boggy, edematous paraspinal musculature with full range of motion of the cervical spine. B) Warm and moist skin texture with some erythema. Sharp, painful paraspinal musculature, and decreased range of motion of the cervical spine. C) Hypertonic paraspinals with severe, sharp pain with palpation. And a compensated spinal curve in the thoracic spine. D) Dry skin, and a moderate amount of paraspinal tenderness with ropy and fibrotic cervical musculature. E) Decreased range of motion of the cervical spine with an uncompensated spinal curve.

D The above patient has degenerative joint disease of his cervical spine. The condition has been present for several years and has been getting progressively worse. When you examine this patient you would expect to find chronic changes. There are some key phrases used to describe chronic somatic dysfunctions they are: cool dry skin, absent/decreased edema or erythema, flaccid, ropy and fibrotic tissues, and pain that may be burning, achy or dull in nature. Boggy and edematous paraspinal musculature is associated with acute somatic dysfunctions. In addition, severe cervical degenerative joint disease is usually associated with a decreased range of motion. Answer A Warm and moist skin texture with some erythema is associated with acute somatic dysfunctions. Answer B Hypertonic paraspinals with severe, sharp pain upon palpation are associated with acute somatic dysfunctions. Answer C An uncompensated spinal curve is associated with acute somatic dysfunctions. Chronic somatic dysfunctions are associated with compensated spinal curves. If a somatic dysfunction produces a spinal curve, over time other areas of the spine will tend to compensate with opposite spinal curves in attempt to straighten the body. Answer E

On examination, there is no evidence of deformity of the forearm or wrist. Upon palpation, he has exquisite pain between the extensor pollicis longus and abductor pollicis longus tendons at the radiocarpal joint. There is mild edema at the radiocarpal joint. He has decreased flexion and extension at the wrist as well as decreased forearm supination. There is a tender point located at the musculotendonous insertion of the lateral epicondyle. X-rays of the wrist and elbow are normal without evidence of fracture Which of the following techniques is contraindicated in this patient? A) Muscle energy to the upper extremity to increase supination B) Counterstrain to the tenderpoint C) Thoracic inlet release D) HVLA to the right wrist to improve range of motion E) Posterior axillary fold technique

D The anatomic snuff box is located at the radiocarpal joint and is bordered by the extensor pollicis longus and abductor pollicis longus tendons. Since the patient has pain at the anatomic snuff box with some edema and decreased range of motion at the wrist, he may have a fracture of the scaphoid bone. HVLA is contraindicated in areas of the fracture. Muscle energy to the upper extremity to increase supination is not contraindicated because this technique can be modified so that no stress is applied to the radiocarpal joint. A cross extensor reflex muscle energy could also be used. This would require treating the left upper extremity in order to achieve a therapeutic effect in the right upper extremity. Answer A Counterstrain is not contraindicated because this technique can also be modified so that no stress is applied to the radiocarpal joint. Answer B Thoracic inlet release and posterior axillary fold techniques are indicated in this scenario because both techniques will help open lymphatic channels and help decrease the edema in this person's right upper extremity. Answers C and E

A patient with left shoulder pain has the following findings: - The left clavicle at the sternum is more cephalad than the right. - The right sternoclavicular head of the clavicle moves inferiorly when the patient shrugs his shoulders while the left does not. What is the most likely diagnosis? A) Left anterior rotation somatic dysfunction of the sternoclavicular joint B) Left anterior somatic dysfunction of the sternoclavicular joint C) Left inferior somatic dysfunction of the sternoclavicular joint D) Left superior somatic dysfunction of the sternoclavicular joint E) Left posterior somatic dysfunction of the sternoclavicular joint

D The clavicle moves in three different directions. First, it moves superior and inferior with shrugging and depressing the shoulder. Second, it moves anterior and posterior with retraction and protraction of the shoulder. Third, it rotates anteriorly and posteriorly with internal and external rotation of the arm when it is abducted at 90°. Shrugging the shoulder will cause the lateral end of the clavicle to move superior; this will cause the medial end to move inferior. If it does not, this indicates that a somatic dysfunction is present. Since you name somatic dysfunctions in the direction of freer motion, the clavicle has a superior somatic dysfunction at the sternum.

An 81-year-old male with atherosclerotic heart disease was admitted to the hospital with an exacerbation of COPD. He eventually developed pneumonia and went into acute respiratory failure. He was intubated and received assisted mechanical ventilation for 2 weeks. He was extubated and transferred to a general medical floor where he spent two additional weeks. Today, he arrives at a skilled nursing facility where you are the covering house physician. On your admission history and physical exam, the patient complains of low back pain. On examination, you notice that this severely debilitated male has generalized muscle atrophy and requires maximal assistance with bed mobility. He is unable to tolerate sitting or standing. You conclude that his back pain is due to his prolonged bedridden state. Which of the following treatment protocols is best suited for this patient? A) Low velocity moderate amplitude techniques followed by gentle isometric exercises to strengthen the lumbar spine B) Myofascial release of the sacrum followed by isotonic exercises with low resistance to strengthen the lumbar spine C) Rib raising to help with respiration followed by muscle energy to the pelvis and lumbar somatic dysfunctions D) Indirect treatments to the lumbar fascia followed by direct gentle artiCUlatory techniques E) Myofascial release to the thoraco-Iumbar junction followed by gentle high velocity low amplitude treatment

D The elderly patient in the above question is recovering from severe pulmonary complications from his COPD. He has been bedridden for several weeks and he has coronary disease. As noted in the Foundations of Osteopathic Medicine elderly patients and hospitalized patients typically respond better with indirect techniques or gentle direct techniques, such as rib raising. This patient could not tolerate isometric exercise due to his low vitality. Answer A The patient will not be able to tolerate isotonic exercises with low resistance, even though myofascial release of the sacrum may help this patient. Answer B Although rib raising will help the patient's respiratory status, this patient will not be able to tolerate muscle energy because of his low Vitality. Answer C Although this patient may tolerate myofascial release, he has been bedridden for several weeks. Thus, his bone mineral density may be decreased to the level of osteoporosis. High velocity low amplitude techniques are thus contraindicated. Answer E

Which of the following is true regarding the cervical spine? A) Hypertrophic changes at the C4/C5 facet joint and degenerative changes at the corresponding joint of Luschka may cause C4 nerve root entrapment B) The synovium of the uncovertebral joint is continuous with that of the facet joint. C) The main motion of the OA joint is rotation D) The joints of Luschka play an important role in cervical sidebending E) Restriction within the sternocleidomastoid will decrease neck rotation to the contralateral side

D The joints of Luschka (also called the uncovertebral joints) are only located in the cervical spine. Most authors agree that they are not true synovial joints, however they play an important role in cervical motion, especially sidebending. They also serve to protect nerve roots from disc herniation. Since the C5 nerve root exits between the C4 and C5 vertebrae, degenerative changes of the joint of Luschka and hypertrophy of the facet joint may cause C5 nerve root entrapment, not the C4 nerve root. Answer A If the uncovertebral joint (a.k.a. the joint of Luschka) was a true synovial joint (again most authors think that it is not) its synovium would not be continuous with that of the facet joint. This is because the facet joints and uncovertebral joints are located on opposite sides of the intervertebral foramen. Answer B The main motion of the OA joint is flexion and extension. Approximately half of the flexion and extension of the C-spine stems from the OA joint. Answer C Restriction within the sternocleidomastoid muscle will decrease rotation to the ipsilateral side. Answer E

Which of the following soft tissue dysfunctions is the most likely cause of this innominate dysfunction? A) Tight quadriceps B) Tight hip adductors C) Tight hip abductors D) Tight hamstrings E) Tight piriformis

D The most common cause of a left innominate posterior is tight hamstrings. The hamstrings attach to the ischial tuberosity and contraction can cause the innominate to rotate posteriorly. The most common cause of a left innominate anterior is tight quadriceps. The quadriceps have attachments to the AIlS and contraction can cause the innominate to rotate anteriorly. Answer A Hip adductors are attached to the pubic bones and femur. Tight hip adductors can cause an inferior pubic shear. Answer B Tight hip abductors and piriformis tightness has not been specifically linked with innominate dysfunction. Answers C and E

A 35-year-old female who is obese complains of left foot pain. The pain is worse in the morning when she gets out of bed or if she stands at work for a long period of time. The pain is located on the bottom of her foot at the anterior-medial aspect of the calcaneus. The most likely diagnosis is: A) Pes planus B) Pes cavus C) Tarsal tunnel syndrome D) Plantar fasciitis E) Morton neuroma

D The most common cause of foot pain in outpatient medicine is plantar fasciitis, which results from constant strain on the plantar fascia at its insertion into the medial tubercle of the calcaneus. The most common causes are obesity, prolonged standing, and improper footwear. The pain is worse in the morning, but usually subsides after a few minutes of ambulation. Diagnosis is confirmed by palpation over the plantar fascia on the medial heel. Pes planus is a flattened longitudinal arch; it is not typically associated with pain. Answer A Pes cavus is an exaggerated height of the longitudinal arch; it also is not typically associated with pain. Answer B Tarsal tunnel syndrome is a complex of symptoms resulting from compression of the posterior tibial nerve or the plantar nerve in the tarsal tunnel. Symptoms include pain and paresthesias of the sole of the foot. Answer C Morton's neuroma is a fibroneuromatous reaction between the heads of the third and fourth metatarsals. Pain is present at the forefoot at the site of the neuroma. Answer E

A 35-year-old man comes to your office with back pain. He has no previous history of back pain. He denies recent heavy lifting, however he remembers that he misjudged the distance when stepping off a curb two weeks ago. At first he states that walking "did not feel right", but now, he has developed moderate lower back pain. On examination, he has a positive standing and seated flexion tests on the left. His left ASIS and PSIS are superior when compared to the right. His left sacral sulcus is deep and his left ILA is slightly posterior and markedly inferior. Springing is restricted at the left ILA. The most likely sacral diagnosis is: A) Right sacral rotation on a right oblique axis B) Left sacral rotation on a right oblique axis C) Left sacral rotation on a left oblique axis D) Unilateral sacral flexion on the left E) Unilateral sacral extension on the left

D The patient has a unilateral sacral flexion on the left. The findings for this dysfunction are: left sulcus deeper, left ILA significantly inferior, left ILA slightly posterior. Positive seated flexion test on the left. Motion (springing) at the left sulcus is present. Motion (springing) at the left ILA is restricted. In a right sacral rotation on a right oblique axis, the right ILA would be posterior and slightly inferior. Answer A In a left sacral rotation on a right oblique axis, the left sulcus would be shallow. Answer B In a left sacral rotation on a left oblique axis, the right sulcus would be deep. Answer C In a unilateral sacral extension on the left, the left sulcus is shallow and the left ILA is significantly superior and slightly anterior. Answer E

Which of the follOWing best explains the most appropriate course of action for this patient's hip click? A) Weekly osteopathic manipulation to decrease myofascial restrictions and optimize femoral head position in the acetabulum. B) Monthly radiographs to determine the position of the femur in the acetabulum. C) Monthly ultrasounds of the hip to determine the position of the femoral head in the acetabulum D) Pavlik harness with follow-up CT scan to determine the adequacy of reduction and position. E) Rigid casting with the hips in frog leg position.

D The patient in the above case has a positive Ortolani's sign. This is indicative of congenital dysplasia of the hip. Treatment for this involves the application of a fundamental osteopathic precept: growth and maturation of the skeleton follows function. Many orthopedists today use the Pavlik harness to hold a dislocatable hip in place. This brace allows the child to flex the hip and kick while the hip in a physiologic position. Osteopathic treatment may be used in addition to the above treatment. However, alone it is not considered to be the most appropriate course of action. Answer A Monthly radiological testing and observation is not an adequate course of action. As the child gets older, especially as the child begins to ambulate, it becomes more difficult to obtain adequate reduction without surgery. Answers B and C Rigid casting is an older form of treatment. Placing the hips in the "frog" position may put added stress on the hip while it is reduced and is generally not considered to be the optimal position. Answer E

In a patient with neck and upper thoracic pain, you notice that the fifth cervical vertebrae is extended and rotated right. Ribs 1-5 on the left lag behind with deep inspiration. The thoracic vertebrae, T1 - T5, have a lateral convexity to the right. The transverse processes of these vertebrae are posterior on the right, except for T3, which has a posterior transverse process on the left. Which of the following statements is true regarding diagnosis and treatment? A) The above patient has' an exhalation dysfunction, and initial treatment should be directed toward rib 1. B) T3 is sidebent left, rotated right and the patient would be asked to rotate their torso to the right when correcting this somatic dysfunction using a muscle energy (direct) technique. C) The above patient has an inhalation dysfunction and an HVLA thrust can be directed at the rib angle of rib 5 to correct this dysfunction. D) C5 will resist translation to the right while the head is in the flexed position. Therefore, the head should be placed in the flexed, sidebent left and rotated left position when positioning for a direct muscle energy technique. E) T1 - T5 are neutral, sidebent left rotated right. These vertebrae should be placed in right sidebending and left rotation when employing a direct type of treatment.

D The patient in the above question has the following dysfunctions 1) an exhalation dysfunction of ribs 1 - 5, 2) T1 - T5 neutral sidebent left and rotated right, except for T3 which is sidebent left, rotated left (sagittal component of T3 is not mentioned), . 3) C5 extended rotated right and sidebent right. Translation to the right will induce left sidebending. Since C5 is sidebent right, it will resist left sidebending. When using a direct muscle energy technique, C5 should be positioned against the barrier. C5 then must be flexed rotated left and sidebent left. Even though the patient has a group exhalation dysfunction, initial treatment should be directed at the thoracic spine. As a general rule, the somatic dysfunction of the thoracic vertebrae is treated before treating rib dysfunctions. Answer A Based on the above findings T3 is sidebent left, rotated left. Answer B The above patient has an exhalation dysfunction of ribs 1-5, because these ribs lag behind with inhalation. Answer C T1, T2, T4 and T5 are sidebent left rotated right (not T3). Answer E

A 52-year-old male with atrial fibrillation comes to your office complaining of new onset right shoulder pain. The pain started 24 hours ago during his golf tournament. The patient recalls that the pain came about immediately following a golf swing. The paint which is greatest at the tip of the right acromion does not radiate. He denies any numbness or tingling in his upper extremity. There is full passive range of motion. At rest with his arm by his side, the patient has little pain. However the pain becomes moderate to severe when he attempts to hold his arm above his head. X-rays of his shoulder show no fracture. His medications include Coumadin (Warfarin) and hydrochlorothiazide You instruct your patient to abduct his arms and then slowly lower them. When he attempts to comply, you notice that he is unable to slowly lower the right arm smoothly. What disorder does this suggest? A) Adhesive capsulitis B) Bicipital tendonitis C) Osteoarthritis of the glenohumeral joint D) Rotator cuff tear E) Thoracic outlet syndrome

D The question describes a positive drop arm test. A rotator cuff tear is often associated with trauma and will result in sharp pain at the tip of the acromion, pain with active abduction and a positive drop arm test. Restricted passive and active range of motion is characteristic of adhesive capsulitis. Answer A Bicipital tendonitis will result in pain at the bicipital groove and increased pain with shoulder and elbow flexion. Answer B OA is characterized by a gradual onset of pain and limited passive range of motion. Answer C TOS is due to compression of neurovascular bundle. This typically results in pain radiating into the UE. Answer E

Which of the following statements is true regarding the biomechanics of the lower extremity? A) Anterior glide of the talus provides talocrural stability B) The head of the femur will glide posterior with external rotation of the hip C) Patella femoral syndrome has been associated with an increase of the femoral neck angle. D) Restriction within the subtalar joint will limit internal and external rotation of the leg while the foot is fixed E) Somatic dysfunctions of the ankle most commonly occur when the foot is dorsiflexed

D The subtalar joint acts mostly as a shock absorber and allows internal and external rotation of the leg while the foot is fixed. Restriction within this joint will limit this motion. The talocrural joint (a.k.a. tibiotalar joint) is composed of the articulation of the trochlea of the talus and the ankle mortise. Posterior glide of the talus (dorsiflexion) provides stability to this joint. This joint is relatively unstable in plantar flexion (anterior glide of the talus), and allows the greatest degree of inversion/eversion in this position. Answer A The head of the femur will glide anterior with external rotation of the hip, and posterior with internal rotation of the hip. Answer B It is believed that an increase in the Q angle is associated with patellofemoral syndrome (a.k.a. lateral patella femoral tracking syndrome). Changes in this angle can result in genu valgum or genu varum. Changes in the femoral neck angle are associated with coxa valga and coxa vara. Answer C Somatic dysfunctions of the ankle most commonly occur when the foot is supinated because of the relative instability of the ankle in this position. Answer E

Treatment of which cranial bone would most effectively improve this patient's symptoms? A) Frontal B) Occipital C) Parietal D) Temporal E) Maxilla

D The treatment of choice for acute otitis media would be to address the underlying eustachian tube dysfunction. Eustachian tube dysfunction has been associated with cranial somatic dysfunction, especially internal rotation of the temporal bone; therefore, treatment of the temporal bone would be the most effective. Although treatment of other cranial bones would help improve all cranial functions, none would be as effective as treating the temporal bone in a case of otitis media. Answers A, B, C, and E

In a patient with an anterior fibular head somatic dysfunction, which of the following statements is true A) The above type of dysfunction does not easily respond to treatment B) The somatic dysfunction will increase the amount of passive plantar flexion C) It can result in foot drop D) The tibio-fibular synovial articulation will resist posterior glide E) It is often seen in common ankle sprains

D There are 2 tibio-fibular articulations. The proximal articulation is synovial and the distal articulation is syndesmotic. In an anterior fibular head dysfunction, the proximal tibio-fibular articulation will be displaced anteriorly and will resist posterior glide. A fibular head somatic dysfunction responds well to manipulative treatments. HVLA, muscle energy and indirect methods are commonly used techniques. Answer A A fibular head anterior is associated with an externally rotated talus causing the foot to appear more dorsiflexed and everted. Increased motion in these planes is seen as a result. Answer B The common peroneal nerve courses posteriorly to the fibular head. Posterior fibular head dysfunction can result in compression of this nerve causing foot drop. Answer C The most common ankle sprain occurs with the foot in supination (plantar flexion, inversion and internal rotation). These ankle sprains are associated with posterior fibular head somatic dysfunctions. Answer E

A 48-year-old male complains of low back pain with radiation to the right knee for 3 days. The patient was shoveling snow for several hours, and then was unable to straighten up. The patient has no previous history of low back pain. Physical examination reveals the patient to be in a forward flexed and leaning to the left posture. There are no neurological deficits. X-rays of the lumbar spine are negative for fracture. The following findings were found: L1 FRLSL L5 FRLSL (+) Lumbosacral spring test Pelvis shifted to the right Tenderpoint 1 inch medial to the left ASIS (+) standing flexion test on the right Right ASIS lower than the left Shallow sacral sulcus on the left ILA posterior and slightly inferior on the left The primary cause of this patient's low back pain is: A) Piriformis syndrome B) Spondylolysis C) Herniated nucleus pulposus D) Psoas syndrome E) Lumbar stenosis

D This is a classic presentation of psoas syndrome. Typical signs and symptoms include: a high lumbar somatic dysfunction, pelvic shift to the opposite side of the psoas spasm and a contralateral piriformis spasm. The contralateral piriformis muscle spasm may irritate the sciatic nerve causing radiation of pain down the thigh, but not usually past the knee. Piriformis syndrome is a peripheral neuritis of the sciatic nerve usually due to an irritation or inflammation of the piriformis muscle. Answer A Spondylolysis is a fracture of the pars interarticularis without anterior displacement. This typically is seen as a "collar" on the scotty dog in oblique views of the lumbar spine. Answer B A herniated nucleus pulposus would likely cause a radiating pain below the knee and into the foot. Answer C Lumbar stenosis is usually characterized as back pain with radiation into the lower leg or legs. It can be accompanied with paresthesias and pain worsens with lumbar extension. Answer E

A patient in a car accident, who was not wearing a seat belt, hit his forehead on the steering wheel and the back of his head on the head rest. The patient went to the emergency room and was discharged. X-ray and CT scan were negative. The patient has a constant dull headache. Cranial sacral examination reveals decreased amplitude to the point where it is barely detectable. The most likely craniosacral strain pattern is A) Torsion B) Sidebending and rotation C) Vertical strain D) Lateral strain E) Compression

E A compression strain can be cause by a force to the back of the head, to the front of the head, or from a circumferential compression such as childbirth. It results in a little to no palpable C.R.I.

Which of the following dysfunctions has been most associated with a vertical downward force on the sacrum at the SI joint. A) Innominate anterior B) Innominate posterior C) Innominate inferior shear D) Forward sacral torsion E) Unilateral sacral flexion

E A unilateral sacral dysfunction has been associated with trauma at the S1 joint. The downward force at the sacrum or an upward force from the leg results in a shearing effect at the S1 joint. This force causes the sacrum to be pushed caudad, while the innominate is pushed cephalad. Due to the configuration of the SI joint, the sacral base will move anterior as well as caudad and the ILA will move posterior as well as caudad. An innominate anterior is often due to tight quadriceps such as the rectus femoris. Answer A An innominate posterior is often due to tight hamstrings. Answer B An inferior innominate shear has been associated with a vertical downward force on the innominate, not the sacrum. Answer C Forward sacral torsions have not been associated with a specific type of trauma. Answer D

Which of the following organs would be least affected by an autonomic imbalance of the upper lumbar spine? A) Ureters B) Urinary bladder C) Descending colon D) Penis E) Adrenal medulla

E Adrenal medulla. The adrenal medulla receives sympathetic innervation from T10. The ureters receive sympathetic innervation from T10-L2. Answer A The urinary bladder receives sympathetic innervation from T11-L2. Answer B The descending colon receives sympathetic innervation from T12-L2. Answer C The penis receives sympathetic innervation from T11-L2. Answer D

Which of the following is the correct treatment position for the above tenderpoint? A) Patient prone, right knee and hip extended, rotated to contralateral side B) Patient prone, knees and extended, rotated to ipsilateral side C) Patient left lateral recumbent position, right knee flexed, hips extended D) Patient supine, left knee and hip flexed, sidebent and rotated to ipsilateral side E) Patient supine, both knees flexed and hips flexed to about 90 degrees

E An anterior L5 somatic dysfunction is treated with the patient in the supine position. The knees and hips flexed. Jones reports flexion of both hips to about 135 degrees. Yates reports flexion of both thighs to 60 - 90 degrees with sidebending and rotation away.

The patient is found to have a tenderpoint 1 cm lateral to the pubic symphysis on the right superior pubic ramus. This tenderpoint correlates to somatic dysfunction of which vertebra? A) L1 B) L2 C) L3 D) L4 E) L5

E Anterior lumbar tender points are located around the pelvis. LS is located approximately 1 cm lateral to the pubic symphysis on the superior rami. L2, L3 and L4 tender points are located near the AlIS. Answers B, C and 0 L1 tender point is located medial to the ASIS. Answer A

A patient complains of bloody diarrhea. A colonoscopy reveals the mucosa of the rectosigmoid region is edematous, friable, and with numerous erosions. At which spinal level would you expect to find palpatory changes related to a viscerosomatic reflex? A) C3-T5 B) T1-T4 C) T5-T9 D) T10-T11 E) T12-L2

E Pathology in the rectosigmoid area would cause a viscerosomatic change at the T12-L2 spinal level. C3 - C5 spinal segments are not involved in a viscero-somatic reflex in the rectosigmoid region. Answer A T1-T4 receives sympathetic innervation from the head and neck. Answer B T5-T9 receives sympathetic innervation from the upper GI tract. Answer C T10-T12 receives sympathetic innervation from the middle GI tract. Answer D

A 68-year-old man is brought to your office for a postural evaluation. The sagittal plane evaluation reveals that the patient's entire body leans anteriorly away from a plumb line that hangs from the ceiling to the floor in the exam room. The patient's weight is supported mainly by the metatarsals of his feet. Your postural assessment of this patient would be: A) Military posture B) Swayback posture C) Posterior postural deviation D) Flat back posture E) Anterior postural deviation

E Anterior postural deviation as viewed from the sagittal plane (in relation to a plumb line) is described as: entire body leans forward, deviating anteriorly from plumb line, patient's weight supported by metatarsals. Military posture, as viewed from the sagittal plane (in relation to a plumb line) is described as: head tilted slightly posteriorly, cervical curve and thoracic curve normal, anterior cervical and posterior thoracic deviation from plumb line, anterior pelvic tilt, knees extended, ankles plantar flexed. Answer A Swayback posture as viewed from the sagittal plane (in relation to a plumb line) is described as: head forward, cervical spine lordotic, thoracic spine kyphotic, decreased lordosis of lumbar spine, posterior tilt of pelvis, hip and knee joints hyperextended. Answer B Posterior postural deviation as viewed from the sagittal plane (in relation to a plumb line) is described as: entire body leans backward, deviating posteriorly, balance maintained by anterior thrust of pelvis and hips, and marked lordosis from mid-thoracic spine down. Answer C Flat back posture as viewed from the sagittal plane (in relation to a plumb line) is described as: head forward, cervical spine has slightly increased lordosis, thoracic spine slightly kyphotic in upper portion then flattens in lower segments, lumbar lordosis flattened, and hips and knees extended. Answer D

The most appropriate osteopathic treatment for this patient's tenderness is: A) HVLA B) Sacral rocking C) Still technique D) Cranial sacral E) Counterstrain

E Counterstrain would be the most appropriate and effective treatment for this patient, especially directed at the piriformis muscle. The patient is 42 years old, cachectic, has recently undergone chemotherapy, and is possibly osteopenic. Gentle treatment would therefore be preferred. High velocity low amplitude would be contraindicated in a patient who may be osteopenic (see above). Answer A The purpose of sacral rocking is to relax the muscles of the lumbosacral junction. It typically is indicated in patients with tight lumbar paraspinals. Although sacral rocking would not be entirely inappropriate, it is not the best answer. Answer B Answer C has no known techniques specific for the piriformis that would be as effective, and therefore would not be the most appropriate treatment. Craniosacral treatment would be a gentle treatment, but it would not be the most appropriate treatment because it would not directly release the key somatic dysfunction of the piriformis muscle spasm. Answer D

A 21-year-old male is in the intensive care unit following a motor vehicle accident. He has sustained bilateral tibia/fibula fractures and several rib fractures. He underwent closed reduction/external fixation of his tib/fib fractures 24 hours ago. He has been extubated since the surgery and is complaining of severe pain at the surgical site as well as at the chest wall. Ultrasound on the lower extremities is negative for deep venous thrombosis. Which one of the following is the best statement regarding osteopathic treatment for this patient? A) This patient should receive osteopathic manipulation daily until he is transferred out of the unit. B) This patient should receive direct active forms of treatment such as muscle energy. C) This patient should receive one extensive session of osteopathic treatment then followed-up in two weeks. D) Osteopathic treatment is contraindicated in this patient due to the acute nature of the illness. E) This patient should receive indirect myofascial release and diaphragm release to improve lymphatic return.

E Diaphragm release techniques are not contraindicated in this patient. In addition, this patient could also tolerate some indirect myofascial release techniques. Due to the severity of this person's illness he would not be able to tolerate daily OMT, nor would he be able to tolerate direct active forms of OMT. Answers A and B Osteopathic treatment for those individuals in the intensive care unit should also be limited in dosage. Usually these individuals cannot tolerate extensive osteopathic treatment sessions. Answer C Although this patient is in critical condition he would be able to tolerate some mild forms of OMT. Answer D

34-year-old female comes into your office complaining of mild left-sided thoracic pain. The pain started about a week after the patient began driving her new sports car that has very low riding seats. The pain is non-radiating and worsens with inhalation. X-rays and EKG reveal no abnormalities. Your structural exam reveals that ribs 3-5 on the left are more caudad and lag behind during inhalation. Tender points are noted on ribs 3-5 in the left mid axillary line. Which of the following most correctly describes the somatic dysfunction? A) Type I thoracic dysfunction B) Type II thoracic dysfunction C) Right sidebending thoracic dysfunction D) Left rib 3-5 inhalation dysfunction E) Left rib 3-5 exhalation dysfunction

E Exhalation dysfunctions are characterized by one or a group of ribs held down. These ribs typically lag behind in inhalation. Pain usually increases with inhalation. Also anterior riB tender points (in the mid axillary line) are often associated with anteriorly depressed ribs. Although most rib dysfunctions are due to thoracic dysfunctions, the case does not include a thoracic dysfunction, therefore it cannot be assumed that the patient has one. Answers A, Band C Inhalation dysfunctions are characterized by one or a group of ribs held up. These ribs typically lag behind in exhalation. Pain usually increases with exhalation. Also, posterior rib tender points are often associated with posteriorly depressed ribs. Answer D

Radiation of pain into this patient's right knee is most directly related to: A) Psoas spasm B) Spondylosis C) Disc herniation D) Spondylolisthesis E) Piriformis muscle spasm

E Psoas syndrome could result in a contralateral piriformis muscle spasm. This may irritate the sciatic nerve and may radiate pain to the knee. Although the patient certainly does have psoas syndrome on the left side, it is not most directly related to the pain radiating into the right knee. Answer A Since the x-ray findings are normal, the patient is not likely to have spondylolysis, therefore it is not likely the cause of his pain. Answer B Although disc herniation will typically cause radiation of pain, this is not likely in this patient because herniated discs usually result in a neurologic deficit. Answer C Since the lumbar x-rays are negative, this patient does not have spondylolisthesis. Answer D

You are consulted to see a severely debilitated 87-year-old male with complaints of mid thoracic pain. He was in the intensive care unit for 3 weeks and was transferred to a medical/surgical bed yesterday. He has a history of coronary artery disease, congestive heart failure and prostate cancer with vertebral metastasis. His back pain is localized to the mid thoracic region and seems to be musculoskeletal in nature. Which osteopathic manipulative techniques would be best suited to relieve this patient's symptoms? A) Muscle energy treatment with the spine extended B) Thoraco-abdominal diaphragm release C) High velocity low amplitude D) Pedal (Dalrymple) pump E) Direct myofascial release

E Hospitalized patients typically respond better with indirect techniques or gentle direct techniques. Often these types of patients cannot withstand aggressive treatment. There are a variety of direct myofascial techniques that a physician can do that will not interfere with his fragile medical status. Muscle energy (regardless of the position of the spine) is contraindicated in patients with low vitality such as patients in the intensive care unit. Answer A Techniques geared toward improving lymphatic return are relatively contraindicated in patients with advanced stages of cancer. In addition, the thoracoabdominal diaphragm release and pedal pump are less likely to decrease this patient's mid-thoracic pain since it seems to be musculoskeletal in nature and not directly related to lymphatic congestion. Answers Band D High velocity low amplitude is contraindicated in patients with bone metastasis because this may cause a pathologic fracture to the spine. Answer C

You are consulted to see a severely debilitated 87-year-old male with complaints of mid thoracic pain. He was in the intensive care unit for 3 weeks and was transferred to a medical/surgical bed yesterday. He has a history of coronary artery disease, congestive heart failure and prostate cancer with vertebral metastasis. His back pain is localized to the mid thoracic region and seems to be musculoskeletal in nature. Which osteopathic manipulative techniques would be best suited to relieve this patient's symptoms? A) Muscle energy treatment with the spine extended B) Thoraco-abdominal diaphragm release C) High velocity low amplitude D) Pedal (Dalrymple) pump E) Direct myofascial release

E Hospitalized patients typically respond better with indirect techniques or gentle direct techniques. Often these types of patients cannot withstand aggressive treatment. There are a variety of direct myofascial techniques that a physician can do that will not interfere with his fragile medical status. Muscle energy (regardless of the position of the spine) is contraindicated in patients with low vitality such as patients in the intensive care unit. Answer A Techniques geared toward improving lymphatic return are relatively contraindicated in patients with advanced stages of cancer. In addition, the thoracoabdominal diaphragm release and pedal pump are less likely to decrease this patient's mid-thoracic pain since it seems to be musculoskeletal in nature and not directly related to lymphatic congestion. Answers B and D High velocity low amplitude is contraindicated in patients with bone metastasis because this may cause a pathologic fracture to the spine. Answer C

Evaluation of ribs 5-7 on the right reveal that the ribs are depressed anteriorly and the inferior edges of the posterior rib angles are more prominent. The ribs move caudally during exhalation but do not move cephalad upon inhalation. What is the most likely diagnosis? A) Ribs 5-7 inhalation dysfunction, pump handle predominant B) Ribs 5-7 inhalation dysfunction, bucket handle predominant C) Ribs 5-7 inhalation dysfunction, caliper motion predominant D) Ribs 5-7 exhalation dysfunction, bucket handle predominant E) Ribs 5-7 exhalation dysfunction, pump handle predominant

E In exhalation dysfunctions, the rib moves caudad on expiration and is restricted on inspiration. In pump-handle dysfunctions the findings are concentrated on the anterior part of the rib in the mid-clavicular line and the rib angle. In inhalation dysfunctions, the rib moves cephalad on inspiration and it is restricted on expiration. Answers A, B, and C In bucket handle dysfunctions the findings are concentrated on the lateral part of the rib in the midaxillary line. Answers Band D

Which of the following is the best statement regarding anatomy and biomechanics of the lumbar spine? A) The lumbar spine moves easiest about a sagittal axis B) Lumbarization increased the flexibility of the lumbosacral unit thus decreasing the likelihood of degenerative joint disease. C) The S1 nerve root will exit between the L5 and S1 vertebrae. D) Central disc herniations are more common than posterior-lateral disc herniations. E) Radicular symptoms from a herniated disc will often worsen when the lumbar spine is flexed.

E Lumbar flexion will push the nucleus pulposus posterior and stretch the nerve roots. This will usually worsen radicular symptoms. The lumbar spine moves easiest through flexion and extension. This is motion about a transverse axis and in a sagittal plane. Answer A Lumbarization will alter the structure-function relationship of the lumbosacral junction, leading to early disc degeneration. Answer B In the thoracic and lumbar regions the nerve root will exit the intervertebral foramina below its corresponding segment. For example the L5 nerve root will exit between L5 and S1. Answer C The posterior longitudinal ligament narrows as it approaches the sacrum. This results in a posterolateral weakness making disc herniations more common in this area. Large central disc herniations that impinge S2-S4 can result in cauda equina syndrome, but this is uncommon. Answer D

A 17-year-old female complains of anterior knee pain that gets worse when she runs, jumps or climbs stairs. Physical exam reveals pain is reproducible on squatting and when the knee is flexed. There is point tenderness on the undersurface of the patella and there is some patellar crepitus The most likely diagnosis is: A) Osgood-Schlatter disease B) Baker's cyst C) Patellar tendon tendonitis D) Housemaid's knee E) Patello-femoral syndrome

E Patella-femoral syndrome (also known as lateral femoral patella tracking syndrome) is among the most common causes of knee complaints in primary care medicine, particularly among adolescents and young adults. The patient's chief complaint is anterior knee pain that is exacerbated with running, jumping and climbing stairs. Osgood-Schlotter's is a traction apophysitis of the tibial tubercle. Answer A. Pain and inflammation are located at the tibial tubercle. Housemaid's knee, Answer D, also has enlargement and pain at the tibial tuberosity, which is caused by constant kneeling on hard surfaces. Both of these conditions are responsible for anterior knee pain, but at a very localized area, and do not account for the point tenderness on the posterior aspect of the patella or the patellar crepitus. Baker's cyst is usually found in the popliteal fossa of the knee and does not result in anterior knee pain. These cysts can be associated with degeneration of the posterior horn of the medial meniscus. Answer B Patellar tendonitis is an inflammation of the patellar tendon usually caused by overuse. It results in pain at the insertion into the patella. Patellar crepitus is not an associated finding. Answer C

Which of the following regarding the thoracic vertebrae is true? A) The upper thoracic vertebrae have larger vertebral bodies than the lower thoracic vertebrae. B) T4, T5, and T6 have spinous processes that project the most inferiorly so that the spinous process of T5 is level with the transverse process of T6. C) T10, T11, and T12 have spinous processes that project horizontally, so that the spinous process of T1 0 is level with the transverse process of T1 O. D) The accessory vertebral vein courses through the foramen transversarium of the T1 vertebrae. E) The inferior demifacet of the vertebral body of T3 articulates with rib 4.

E Rib 4 will articulate with the transverse process and superior demifacet (on the vertebral body) of T4 and the inferior demifacet of T3. Vertebral bodies get increasingly larger from the cervical spine to the lumbar spine. Answer A. Based on the "rules of 3's." The first 3 thoracic vertebrae have spinous processes that project horizontally. The following set of 3 (T4, T5, T6), project more inferiorly, and the next set of 3 (T7, T8, and T9), project most inferiorly, and the last set of 3 (T10, T11, T12), project in such a way that T10 follows the rules of T7 - T9, T11 follows the rules of T4 - T6, and T12 follows the rules of T1 - T3. Answers B and C The vertebral artery courses through the foramen transversarium of C1 - C6. T1 does not have a foramen in its transverse process. Answer D

Which of the following is true regarding goals of manipulation in a patient with asthma? A) Bronchodilate by stimulating the parasympathetic nervous system. B) Normalize the phrenic nerve with a CV4 technique. C) Encourage perfusion of lung parenchyma by stimulating the sympathetic nervous system D) Decrease the thoraco-abdominal pressure gradient by redoming the diaphragm E) Improve ventilation and perfusion (V/Q) function of the lungs with rib raising

E Rib raising will increase costal wall motion thus increasing ventilation. It will also normalize sympathetic tone thus increasing tissue perfusion. Stimulating the sympathetic nervous system would cause bronchodilation; stimulating the parasympathetic nervous system would cause bronchoconstriction. Answer A Cranial manipulation may normalize the vagus nerve, not the phrenic nerve. Answer B Decreasing the sympathetic nervous system will encourage perfusion of the lung parenchyma. Answer C Redoming the diaphragm will increase the thoraco-abdominal pressure gradient and improve lymphatic return. Answer D

A patient complains of chest pain, especially when she exhales. The pain is localized to ribs 8-10 on the left These ribs are characterized as: A) Bifid ribs B) Atypical ribs C) True ribs D) Floating ribs E) False ribs

E Ribs 8-10 are classified as false ribs. False ribs are those ribs that do not directly attach to the sternum. Bifid ribs occur in 1-2% of the population, but this abnormality is present in 8.4% of Somoans. The condition is usually unilateral, and appears as an 8th true rib. Answer A The atypical ribs are 1, 2, 11, and 12. Sometimes rib 10 is considered atypical. Answer B True ribs directly attach to the sternum. The true ribs are ribs 1-7. Answer C The floating ribs are ribs 11 and 12. Answer D

In a patient with neck pain, the atlas is rotated to the left. Given this information what else would you expect to find on this patient? A) The OA joint will resist translation to the left. B) C2 will resist anterior glide on the right C) C2 will resist right sidebending D) The lower cervical segments will be rotated to the right to compensate for the somatic dysfunction of this upper segment E) With the patient's neck flexed to 45 degrees, he will be able to turn his head further left than right.

E Rotation of the atlas (on the axis) is the AA joint. Flexing the patient's head will lock out rotation of C2 - C7 and will isolate rotation to the AA joint. Since the AA joint is rotated to the left, the patient should be able to turn his head further left than right. The AA joint will not necessarily affect motion of the occiput on the atlas (OA joint) or C2 on C3. Answers A, Band C Motion at the AA joint will not necessarily cause compensation or somatic dysfunction of the lower cervical units. Answer D

The patient's tender point is from spasm of which muscle? A) Psoas B) Gluteus maximus C) Gluteus medius D) Gluteus minimus E) Piriformis

E The Jones tender point for the piriformis is located 1/2 way between the inferior lateral angle of the sacrum and the greater trochanter within the belly of the piriformis muscle. The Jones tenderpoint for the psoas is about one inch medial to the ASIS. Answer A Answers B, C, and D are all located in the upper half of the buttock, distributed from just lateral of the ASIS following around and below the iliac crest. They are not located in the middle of the buttock

An 18-year-old male suffered a closed head injury from a motor vehicle accident 21 days ago. He was recently admitted to a rehabilitation hospital where he has been making moderate progress until two days ago when he developed increasing confusion and urinary incontinence. Based on these symptoms, what craniosacral techniques would this patient most likely benefit from? A) Occipital condylar decompression to stimulate the reticular activating system B) Venous sinus technique to improve drainage and decrease any residual cranial edema C) CV4 technique to enhance arousal D) Sacral rocking to improve bladder control E) All cranial treatment is absolutely contraindicated in this patient.

E The above patient has developed symptoms of a post-traumatic hydrocephalus (PTH). PTH may present as dementia, ataxia and urinary incontinence. Craniosacral therapy is absolutely contraindicated in situations with increased intracranial pressure. Craniosacral therapy is relatively contraindicated in patients who have had a traumatic brain injury

A patient complains of neck pain. After a thorough exam you find that there are several muscles in spasm. The most painful muscle originates from the transverse processes of the cervical vertebra and inserts onto rib 1. Which muscle is it? A) Platysma B) Longus colli C) Posterior scalene D) Sternocleidomastoid E) Anterior scalene

E The anterior scalene originates from the posterior tubercle of the transverse processes of C3-C6 and inserts onto rib 1. Platysma attaches to the fascia and skin of the pectoralis major and the deltoid inferiorly. It attaches to the mandible superiorly. Answer A Longus colli is found in three parts of the anterior surface of the vertebral column between the atlas and the body of C3. Answer B The posterior scalene originates from the posterior tubercles of the transverse processes of C4-C6. Its insertion is the external border of the second rib. Answer C The sternocleidomastoid muscle originates from the mastoid process and the lateral half of the nuchal line of the occipital bone; it inserts at the manubrium and the medial clavicle. Answer D

A 19-year-old volleyball player dislocated her shoulder while spiking a volleyball several weeks ago. She immediately was taken to the ER where it was re-Iocated. She was started in a rehabilitation program working with her trainer and team physician. Since the injury, her pain has diminished however she still has some persistent weakness with arm abduction. On examination, strength testing is 5/5 except for arm abduction, which is 2/5. Muscle stretch reflexes in the upper extremities are normal and symmetric. X-rays are normal. What is the most likely etiology of her persistent weakness? A) Injury to the dorsal scapular nerve B) Injury to the inferior glenohumeral ligament C) Complete tear of the supraspinatus tendon D) Incomplete rupture of the deltoid at its insertion site E) Injury to the axillary nerve

E The axillary nerve courses around the posterior aspect of the humeral head. The anterior-inferior dislocation can stretch the axillary nerve thus injuring it. Approximately 10% - 18% of shoulder dislocations will result in injury to the axillary nerve. Typical muscle stretch reflexes (deep tendon reflexes) that are tested in the upper extremity are biceps, triceps and brachioradialis reflexes. Injury to the axillary nerve will not affect these reflexes. The dorsal scapular nerve innervates the rhomboids and is not susceptible to injury with shoulder dislocation. Answer A Although anterior/inferior shoulder dislocations typically disrupt the inferior glenohumeral ligament, it is not the most likely cause of arm abduction weakness. Answer B Although complete tears of the supraspinatus tendon may lead to arm abduction weakness, shoulder dislocations do not typically result in complete supraspinatus tears. Answer C Shoulder dislocations are not likely to rupture the deltoid muscle. Answer D

A 17-year-old high school gymnast comes to your office for a high school physical. Throughout her school career she has been having intermittent back pain. The back pain is localized to the right thoraco-Lumbar region and worsens with spinal extension. On examination, her right shoulder is lower than her left, and her right iliac crest is higher. Her right leg is 1.2cm shorter than the left. L2 is flexed, rotated right and sidebent right. Her lumbar spine appears to be sidebent to the right and L5 is rotated left. She has full range of motion and reverses her scoliotic curve on spinal motion testing. Pelvis shifts easily to the left and resistance is present with right pelvic shift. There is a tenderpoints in the center of her left buttocks. On standing postural x-ray of the spine and pelvis the scoliotic curve measures 140 and the left femoral head is 12mm caudad compared to the right. A spina bifida occulta is noted on x-rays without evidence of spondylolysis or spondylolisthesis. Which of the following most accurately describes this patient's scoliosis? A) A functional dextroscoliosis B) A structural dextroscoliosis C) A structurallevoscoliosis D) A structural kyphoscoliosis E) A functional levoscoliosis

E The lumbar spine is sidebent to the right, therefore the patient has a levoscoliosis (for clarification see fig 5.1). Since the patient is able to reverse her scoliosis on range of motion testing, she has a functional scoliotic curve. A dextroscoliosis describes a spine that is sidebent to the left. Answers A and B A structural curve is a spinal curve that is relatively fixed and inflexible. A structural curve will not correct with sidebending in the opposite direction. Answers B, C and D A kyphoscoliosis is a scoliosis that has resulted in an increase in kyphosis. Answer D

What area of the thoracic vertebra is most commonly palpated for evaluation of a patient's thoracic somatic dysfunction? A) Spinous process B) Superior costal facet C) Pedicle D) Transverse costal facet E) Transverse processes

E The most accepted area of the thoracic vertebra for palpatory evaluation of the thoracic spine is the transverse process. The spinous process could be used, but convention dictates that it is the transverse processes. Answer A Answers B, C, and D would be ineffective for diagnosis and are not readily palpable

A 63-year-old-male with COPD is complaining of right shoulder and arm pain. The patient has noticed that the pain gets worse during exacerbation of his COPD, especially, when he has difficulty breathing. Physical examination reveals bilateral rhonchi with decreased breath sounds. Weakness and paresthesias of the right arm and a diminished right radial pulse occur when the right arm is abducted, extended and the patient turns his head to the right side. There is hypertrophy of respiratory accessory muscles and no lymphadenopathy noted. EKG showed a right axis deviation, low voltage complexes, and poor Rwave progression. Chest, cervical spine and right shoulder X-rays are negative. The most likely diagnosis is: A) Myocardial infarction B) Cervical disk herniation C) Cervical radiculopathy D) Pancoast's tumor E) Thoracic outlet syndrome

E The patient has thoracic outlet syndrome. The patient has a classic presentation of this syndrome. This is characterized by weakness and paresthesias with a decreased radial pulse when the arm is abducted, extended when the patient takes a deep breath and turns his head towards the effected shoulder (Adson's test) Myocardial infarction is unlikely since acute changes, such as ST elevation or depression are not present in the EKG. The EKG changes in the question are associated with cor pulmonale. COPD is the most common cause of cor pulmonale. Answer A Cervical disc herniation and radiculopathy would result in weakness and paresthesias in the upper extremity, however it will not result in a decreased peripheral pulse. In addition, C-spine x-rays are negative; therefore radiculopathy from cervical stenosis is unlikely. Answers B and C Pancoast's tumor is located at the pulmonary apex. The tumor may cause shoulder and arm pain with paresthesia, however, the chest x-ray would show a density in the right upper lobe. Answer D

A 54-year-old alcoholic male patient complains of tender bumps at the distal palmar crease of his hand. Physical exam reveals subcutaneous nodules and palmar fascia contracture of the ring finger. The most likely diagnosis is A) DeQuervain's disease B) Heberden node C) Syphilis D) Bouchard's node E) Dupuytren's contracture

E This patient has Dupuytren's contracture. This condition is characterized by contracture of the palmar fascia and nodule formation. There appears to be a genetic predisposition and is frequently found in alcoholics. DeQuervain's disease is a tendonitis of the abductor pollicis longus, extensor pollicis longus or extensor pollicis brevis tendons. Answer A Herberden's nodes are cartilaginous and bony enlargements of the distal interphalangeal joints of the fingers in degenerative joint diseases. It is most commonly seen in osteoarthritis. Answer B Secondary stage of syphilis presents with a rash on the palms of the hands and feet, however, this patient has nodules, not a rash. Answer C Bouchard's nodes are cartilaginous and bony enlargements of the proximal interphalangeal joints of the fingers in degenerative joint diseases. It is most commonly seen in rheumatoid arthritis. Answer D

Evaluation of a patient with hip and groin pain reveals a positive standing and seated flexion test on the right. The left ASIS appears more caudad and the left PSIS is more cephalad when compared to the right. The lumbosacral junction springs freely. The right sacral base is more anterior when compared to the left. What of the following is the best answer? A) This patient has a posterior rotated innominate on the left B) The above patient is likely to have tight quadriceps on the right C) This patient has a right shortened psoas D) ASIS compression of the pelvis on the right will demonstrate adequate resiliency E) This patient is likely to have an apparent short leg on the right

E The patient in the above question has positive standing and seated flexion tests (SFT & SeFT) on the right (Le. the patient has a pelvic somatic dysfunction on the right). Since the left ASIS is caudad and the left PSIS is cephalad, the left innominate has rotated anteriorly. However, since the SFT indicates a somatic dysfunction on the right, the right innominate must have rotated posteriorly. This typically results in a shorter leg ipsilaterally (on the right). Tight quadriceps is associated with an anteriorly rotated innominate. Tight hamstrings is associated with a posteriorly rotated innominate. Answer B Although innominate dysfunctions can be associated with a shortened psoas (psoas syndrome), not all innominate dysfunctions are caused by a shortened psoas. In addition, findings that are common in psoas syndrome are a tenderpoint medial to the ASIS, high lumbar somatic dysfunction and a positive Thomas test. Answer C The ASIS compression test identifies somatic dysfunction in the SI joint. Since there is somatic dysfunction in the right SI joint, there will be resisted compression at the right ASIS. If there was no SI dysfunction on the right, there will be adequate compression (adequate resiliency) on this side. Answer D

Which of the following statements accurately describes the best course of treatment? A) Weekly muscle energy to the scapula stabilizers and continued use of the shoulder sling. B) Avoiding overhead activities and myofascial release to the shoulder musculature. C) Referral to a surgical specialist for evaluation of a rotator cuff tear. D) This patient should begin progressive resistive exercises to strengthen his rotator cuff E) This patient may benefit from Spencer techniques to free up restrictions in the glenohumeral joint followed by gentle strengthening of the rotator cuff.

E The patient in the above question may have suffered a rotator cuff tear and now has subsequent adhesive capsulitis. The best treatment for this is gentle strengthening of the rotator cuff and stretching of the joint capsule. Stretching the joint capsule is easily done with Spencer techniques. Continued immobilization of the shoulder with use of a sling may actually worsen this patient's condition. Answer A Since this patient does not have any evidence of impingement of the shoulder, he would be cleared to do overhead activities. Overhead activities would be encouraged in this case to help stretch the joint capsule. Answer B This patient most likely does not have a complete rotator cuff tear since x-rays are negative for a high riding humeral head, thus conservative management is preferred. In addition, based on this patient's age conservative management may be considered even in a complete tear. Answer C Progressive resistive exercises are usually started after the patient has achieved full range of motion. Answer D

A 24-year-old is complaining of buttock pain, proctalgia fugax, and dyspareunia. The patient remembers that the problem began after the birth of her 3-year-old son. The pain has been getting progressively worse. The patient feels her marriage is good and she still loves her husband, however she admits that she is often sad and cries a lot. The physical examination reveals external hemorrhoids, tenderness at the tip of the coccyx, and a tender point V2 way between the inferior lateral angle of the sacrum and the greater trochanter in the right buttocks. The most likely diagnosis is: A) Post partum depression B) Piriformis syndrome C) Cauda equina syndrome D) Sacroiliac joint syndrome E) Coccygodynia

E This is a classic presentation of chronic coccygodynia. Coccygodynia is used to describe persistent pain at the coccyx. The etiologies are many, some of which include infection, osteomyelitis, fractures, postnatal injuries, and other types of trauma. Symptoms include proctalgia fugax (rectal pain from levator ani spasm), pain upon sitting, pain with bowel movements, constipation, and dyspareunia. Physical exam reveals a characteristic tenderness at the tip of the coccyx and spasm of adjacent muscles. Postpartum depression is unlikely, since her child was born three years ago. Answer A Although the patient has a piriformis tender point, her main complaint is tenderness at the tip of the coccyx. In addition, piriformis syndrome is often associated with pain that radiates down the thigh. Answer B Cauda equina syndrome is compression of the cauda equina nerve roots. It presents with saddle anesthesia, decreased deep tendon reflexes, decreased rectal tone, and loss of bowel and bladder control. Answer C Sacroiliac joint syndrome usually presents with pain at the sacroiliac joint area, restricted sacral motion, and associated spasm of one or more of the muscles attaching to the sacrum. Answer D

63-year-old female fell off a chair and landed on her buttocks. She complains of severe, unrelenting low back pain. There is point tenderness to percussion of the spinous process of L3. A lumbar spine X-ray confirms your diagnosis. The most likely etiology of the pain is: A) Tumor B) Herniated nucleus pulposus C) Cauda equina syndrome D) Spinal stenosis E) Compression fracture

E This is the classic presentation of a compression fracture. The key findings are spinous process point tenderness to percussion with confirmation on X-ray. X-rays primarily identify bony and some soft tissue problems. A tumor would usually be painless, not result from trauma, and it is unlikely to be visible on X-ray unless it obscures a structure or displaces it. Answer A Herniated nucleus pulposus and cauda equina syndrome would not be visible on X-ray, and do not fit the case history. Answers B and C Spinal stenosis would be visible on X-ray, but pain is not usually localized to the spinous process, or elicited with percussion of the spinous process. Answer D

A 6-month-old male presents with 101-degree fever, runny nose, vomiting, and diarrhea. Physical exam reveals decreased cone of light and erythema of the right tympanic membrane. Chest x-ray is negative for infiltrates, urinalysis is negative for infection, blood and cultures are negative. Which of the following lymphatic techniques would be the most effective adjunctive therapy A) Liver pump B) Spleen pump C) Abdominal pump D) Thoracic pump E) Galbreath Technique

E This patient has an acute otitis media, and Galbreath's technique is indicated as appropriate adjunctive therapy. Galbreath's technique is a soft tissue manipulation technique used to increase drainage of middle ear structures via the Eustachian tube. Answers A, B, C, and D are effective lymphatic techniques, but would not directly affect the ear, and hence would not be the most effective adjunctive therapy.

A 35-year-old female comes to your office with an insidious onset of vague abdominal discomfort. She reports a loss of appetite and a 15-pound weight loss. She denies any melena or hematochezia. On physical exam no abdominal masses are palpable and bowel sounds are present in all quadrants. Stool guiac testing is negative for blood. No axillary or inguinal lymph nodes are palpable, however she does have a palpable lymph node at the periumbilical region. The most likely diagnosis is: A) Peritonitis B) Pancreatitis C) Ectopic pregnancy D) Rectosigmoid cancer E) Gastric adenocarcinoma

E This patient most likely has a gastric adenocarcinoma with metastatic spread to the periumbilical lymph node (Sister Mary Joseph's nodes). Gastric cancers can cause upper abdominal discomfort varying in intensity from a vague, postprandial fullness to a severe, steady pain. Anorexia, often with slight nausea is common. Peritonitis and pancreatitis usually presents with acute abdominal or pelvic tenderness. It will not typically result in a palpable periumbilical lymph node. Answers A and B Ectopic pregnancy usually presents with abdominal pain, amenorrhea, and vaginal bleeding. It also does not result in a palpable periumbilical lymph node. Answer C Rectosigmoid cancer usually causes GI bleeding, this will result in a positive stool guaiac. Classically, colon cancer does not result in a palpable periumbilical lymph node. Answer D

A patient with a left-sided paresis from a stroke ambulates by leaning to the right side, advancing the left leg in a circumduction pattern while the left foot drags on the floor.

In this type of gait, the leg is extended at the knee with plantar flexion at the ankle. The patient leans to the stronger side (in this case the right side) and advances the leg by circumducting it. Since the ankle is plantar flexed the foot tends to drag against the floor

A 56-year-old-male is observed to shift his body to the right side when he walks. The patient has a positive Trendelenburg sign when standing on the right foot.

This gait is characterized by a shift of the body toward the side of the weak gluteus medius. In this case the person's right gluteus medius muscle is weak. He compensates by shifting his body weight toward the weak side. By doing this he prevents the left side of his pelvis from dropping.

A 70-year-old-male with rigidity, resting tremor, bradykinesia has a slow walking speed and his feet do not clear the ground

This is typically described as short, flat-footed shuffling steps in which the foot does not clear the ground. In Parkinsonism, rigidity, tremor, paucity of movement, shuffling with haste, and difficulty in starting, stopping, or turning are also seen.


Conjuntos de estudio relacionados

Vocabulary and Analytical Reasoning II

View Set

Uppers, Downers, and All Arounders 8th ed. Chapter 2, Uppers, Downers, All Arounders Ch 2, Uppers Downers All Arounders: Chapter 1, Uppers,Downers,All Arounders chapter 1 Ouiz, Uppers Downers and All Arounders chapter 3, Chapter 3 - Uppers, upper dow...

View Set

WHA Study Guide for the Neolithic Revolution Test

View Set

Mental Health Chapter 18 Evolve Questions (Eating and Feeding Disorders)

View Set